STUDY MODE - Practice Exam 2 Flashcards

1
Q

A client tells a clinician that she’s having a “lot of problems” with food. She states that she’s been on numerous diets over the years and, even when she’s achieved her desired weight, she always gains it back. She says that she has been evaluated for medical problems such as hypothyroidism, but all of the tests have been negative. When determining if the client’s symptoms meet the criteria for Binge Eating Disorder or Bulimia Nervosa, the clinician should keep in mind that:
Select one:

A. binges are more severe in Binge Eating Disorder than in Bulimia Nervosa.
B. a disturbance in body image is not characteristic of Bulimia Nervosa.
C. compensatory behavior is not a requirement for Binge Eating Disorder.
D. a sense of a lack of control over eating during binges is not characteristic of Binge Eating Disorder.

A

Answer C is correct: Compensatory behavior is a diagnostic criterion for Bulimia Nervosa but not for Binge Eating Disorder and, therefore, differentiates the two disorders.

Answer A: Binges can be severe in both disorders and, therefore, is not a distinguishing characteristic.

Answer B: The disturbance in body image associated with Bulimia Nervosa involves an excessive emphasis on body shape and weight in self-evaluation.

Answer D: A sense of a lack of control is characteristic of both disorders.

The correct answer is: compensatory behavior is not a requirement for Binge Eating Disorder.

How well did you know this?
1
Not at all
2
3
4
5
Perfectly
2
Q

Research investigating the effectiveness of treatments for Conduct Disorder has demonstrated that:
Select one:

A. boot camps and “tough love” policies are more effective than parent management training and multisystems therapy.
B. boot camps and “tough love” policies are as effective as parent management training and multisystems therapy.
C. boot camps and “tough love” policies are less effective than parent management training and multisystems therapy.
D. boot camps and “tough love” policies are effective only when they are combined with parent management training or multisystems therapy.

A

The research has clearly demonstrated that long-term multimodal or multisystems treatments - e.g., parent management training, functional family therapy, and multisystems therapy - are most effective for children and adolescents with Conduct Disorder.

a. Incorrect See explanation for response c.
b. Incorrect See explanation for response c.
c. CORRECT Boot camps and other inoculation techniques have not been found to be effective and, in some cases, may actually be injurious. Therefore, they are not recommended as a stand-alone treatment or as elements of a multimodal intervention. See, e.g., AACAP, Practice parameters for the assessment and treatment of children and adolescents with Conduct Disorder, Journal of the American Academy of Child and Adolescent Psychiatry, 36(10), 122S-139S, 1997.
d. Incorrect See explanation for response c.

The correct answer is: boot camps and “tough love” policies are less effective than parent management training and multisystems therapy.

How well did you know this?
1
Not at all
2
3
4
5
Perfectly
3
Q

As described in the DSM-5, the characteristic symptoms of Substance Use Disorder can be categorized in terms of which of the following?
Select one:

A. impaired control, social impairment, risky use, and pharmacological criteria
B. functional impairment, denial of consequences, impaired control, and pharmacological criteria
C. impaired control, tolerance, withdrawal, and functional impairment
D. functional impairment, tolerance and withdrawal, risky use, and marked distress

A

Answer A is correct: The DSM-5 categorizes the indicators of problematic substance use in terms of the four categories listed in this answer. Impaired control refers to the tendency of the individual to use more of the substance than intended, inability to cut down on substance use, craving for the substance, and tendency to spend a great deal of time obtaining, using, or recovering from the effects of the substance. Social impairment refers to difficulties with social, occupational, and/or recreational activities due to the substance use. Risky use refers to use of the substance in situations that are potentially physically hazardous or despite knowing that physical or psychological problems may be exacerbated by substance use. Pharmacological criteria are tolerance and withdrawal.

Answers B, C, and D: See explanation for answer A.

The correct answer is: impaired control, social impairment, risky use, and pharmacological criteria

How well did you know this?
1
Not at all
2
3
4
5
Perfectly
4
Q

A 58-year old woman is brought to the mental health clinic where you work by her husband. She seems agitated and, during your interview with her, she paces back and forth and engages in frequent hand-wringing. The woman says she’s been feeling very sad and hopeless, and she also tells you that she thinks her next door neighbor has tried to poison her food several times because she has been so unfriendly toward him. The woman’s husband says that his wife has been gradually losing interest in her usual activities over the past several weeks but that her symptoms became worse a few days ago. Based on this information, the most likely diagnosis is:
Select one:

A. Major Depressive Disorder.
B. Schizoaffective Disorder.
C. Bipolar II Disorder.
D. Delusional Disorder.

A

The woman’s pervasive depressed mood and common associated symptoms (agitation, impaired memory) suggest a major depressive episode.

a. CORRECT Delusions are not uncommon in Major Depressive Disorder and often involve a belief that one is being persecuted because of immorality or a personal inadequacy.
b. Incorrect In Schizoaffective Disorder there are periods of at least two weeks duration in which there have been delusions or hallucinations in the absence of a mood disturbance.
c. Incorrect Bipolar II Disorder requires a history of hypomanic and major depressive episodes. The woman’s symptoms are not suggestive of hypomania.
d. Incorrect A diagnosis of Delusional Disorder requires the presence of delusions that are not due to Major Depressive Disorder or other disorder. This woman’s delusion seems to be part of her depression.

The correct answer is: Major Depressive Disorder.

How well did you know this?
1
Not at all
2
3
4
5
Perfectly
5
Q

The use of Haloperidol or other dopamine blocking agent for the treatment of Tourette’s Disorder:
Select one:

A. is uncommon since these drugs are rarely effective for alleviating tics.
B. can be problematic in many cases because of the severe side effects of these drugs.
C. is contraindicated because these drugs exacerbate tics in most patients.
D. should be considered only in the presence of psychotic symptoms.

A

Antipsychotic drugs are associated with a number of unpleasant side effects.

a. Incorrect Antipsychotic drugs are effective in about 80% of cases of Tourette’s Disorder.
b. CORRECT About 50% of people with Tourette’s Disorder who take Haloperidol or similar drug develop intolerable side effects.
c. Incorrect This is not true.
d. Incorrect This is not true. Antipsychotic drugs are useful for alleviating tics in most cases.

The correct answer is: can be problematic in many cases because of the severe side effects of these drugs.

How well did you know this?
1
Not at all
2
3
4
5
Perfectly
6
Q

Research investigating the usefulness of phototherapy as a treatment for seasonal affective disorder has generally found that it is:
Select one:

A. substantially less effective than antidepressant drugs.
B. an effective treatment and produces results comparable to those produced by antidepressant drugs.
C. effective only when administered in conjunction with a low- to moderate-dose of an antidepressant drug.
D. effective only for individuals who have atypical symptoms.

A

Many of the studies investigating the effectiveness of phototherapy (light therapy) for seasonal affective disorder (SAD) have been criticized on methodological grounds. However, the general consensus is that phototherapy is an effective treatment for this disorder.

a. Incorrect The studies suggest that phototherapy is as effective as antidepressants for reducing the symptoms of SAD.
b. CORRECT Of the answers given, this one best summarizes the existing research. See, e.g., R. N. Golden et al., The efficacy of light therapy in the treatment of mood disorders: A review and meta-analysis of the evidence, The American Journal of Psychiatry, 2005, 162, 656-662.
c. Incorrect The research has not found this to be true.

d. Incorrect The research has not found this to be true.
The correct answer is: an effective treatment and produces results comparable to those produced by antidepressant drugs.

How well did you know this?
1
Not at all
2
3
4
5
Perfectly
7
Q

A DSM-5 diagnosis of Posttraumatic Stress Disorder or Acute Stress Disorder requires the development of characteristic symptoms following exposure to:
Select one:

A. actual or threatened death, serious injury, or sexual violence.
B. actual or threatened death, serious injury, or a threat to the physical integrity of self or others.
C. one or more severe psychosocial stressors.
D. a threat to the physical integrity of self or others.

A

Answer A is correct: This answer (exposure to actual or threatened death, serious injury, or sexual violence) accurately describes the requirement for a DSM-5 diagnosis of PTSD or Acute Stress Disorder.

The correct answer is: actual or threatened death, serious injury, or sexual violence.

How well did you know this?
1
Not at all
2
3
4
5
Perfectly
8
Q

Psychological autopsies suggest that about ____% of people who complete suicide had one or more mental disorders at the time of their death.
Select one:

A. 25
B. 55
C. 70
D. 90

A

Psychological autopsy (PA) was originally developed as a method for identifying the mode of death in equivocal cases (i.e., homicide, suicide, natural causes, or accident) but has become the primary method for identifying the proximate cause of completed suicide.

Answer D is correct: Based on their comprehensive review of PA studies, J. T. O. Cavanagh and colleagues concluded that that about 90% of people who commit suicide had at least one diagnosable mental disorder, and this conclusion has been confirmed by a number of subsequent studies (Psychology autopsy studies of suicide: A systematic review. Psychological Medicine, 33, 395-405, 2003).

The correct answer is: 90

How well did you know this?
1
Not at all
2
3
4
5
Perfectly
9
Q

The intense “sleep attack” associated with Narcolepsy is often accompanied by:
Select one:

A. cataplexy.
B. dyskinesia.
C. ataxia.
D. muscular rigidity.

A

Narcolepsy involves frequent intense periods of irresistible sleep.

a. CORRECT Sleep attacks in Narcolepsy may include cataplexy, which is a sudden loss of muscle tone.
b. Incorrect See explanation for response a.
c. Incorrect See explanation for response a.

d. Incorrect See explanation for response a.
The correct answer is: cataplexy.

How well did you know this?
1
Not at all
2
3
4
5
Perfectly
10
Q

Two weeks after witnessing the murder of a co-worker, Mrs. Cee develops a number of symptoms including a sense of detachment, flashbacks of the event, sleep problems, impaired concentration, a depressed mood, and an unwillingness to talk about the murder. She has had these symptoms for seven days. Based on these symptoms, the most likely diagnosis is:
Select one:

A. PTSD.
B. Brief Psychotic Disorder.
C. Acute Stress Disorder.
D. Conversion Disorder.

A

Mrs. Cee’s symptoms followed (and were apparently the result of) exposure to a traumatic event that is outside the range of normal human experience.

a. Incorrect See explanation for response c.
b. Incorrect See explanation for response c.
c. CORRECT Because the duration of the woman’s symptoms are less than one month (the minimum duration required for PTSD), Acute Stress Disorder is the most likely diagnosis.

d. Incorrect See explanation for response c.
The correct answer is: Acute Stress Disorder.

How well did you know this?
1
Not at all
2
3
4
5
Perfectly
11
Q

The presence of which of the following would suggest that the appropriate diagnosis is Social Anxiety Disorder rather than Agoraphobia?
Select one:

A. fear of humiliation or embarrassment in social situations
B. increased anxiety in feared situations when accompanied by a trusted companion
C. anxiety is perceived as excessive and temporarily controllable
D. an absence of the physiological symptoms associated with a panic attack

A

Social Anxiety Disorder (also known as Social Phobia) and Agoraphobia are sometimes difficult to distinguish because they share several symptoms.

a. Incorrect Fear of humiliation or embarrassment in social situations is characteristic of both disorders (although the fear is not necessarily limited to social situations in Agoraphobia).
b. CORRECT The effects of having a companion in anxiety-arousing situations can help distinguish the two disorders. In Agoraphobia, the presence of a trusted companion often alleviates anxiety. In Social Anxiety Disorder, a companion can actually increase anxiety.
c. Incorrect These symptoms are not useful for distinguishing between Social Anxiety Disorder and Agoraphobia.
d. Incorrect Both disorders may or may not involve panic attacks.

The correct answer is: increased anxiety in feared situations when accompanied by a trusted companion

How well did you know this?
1
Not at all
2
3
4
5
Perfectly
12
Q

The diagnosis of Cyclothymic Disorder requires a minimum duration of symptoms for:
Select one:

A. one year in adults and eight months in children and adolescents.
B. one year in adolescents and adults and six months in children.
C. two years in adults and one year in children and adolescents.
D. two years in adolescents and adults and one year in children.

A

You’re likely to encounter one or two questions on the licensing exam that require you to know the duration requirements for a specific diagnosis. To prepare for these questions, focus on questions in the AATBS practice tests that address this issue.

a. Incorrect See explanation for response c.
b. Incorrect See explanation for response c.
c. CORRECT For the exam, you want to know that the minimum duration of symptoms for Cyclothymic Disorder is two years for adults and one year for children and adolescents.

d. Incorrect See explanation for response c.
The correct answer is: two years in adults and one year in children and adolescents.

How well did you know this?
1
Not at all
2
3
4
5
Perfectly
13
Q

Depression has been linked to:
Select one:

A. decreased REM latency and increased REM density.
B. decreased REM latency and decreased REM density.
C. increased REM latency and increased REM density.
D. increased REM latency and decreased REM density.

A

Depression has been linked to a number of alterations in sleep quantity and quality.

a. CORRECT The research has consistently shown that depression (especially endogenous depression) is associated with decreased sleep continuity, reduced slow-wave sleep, shortened REM latency (earlier onset of REM sleep), and increased REM density (increased frequency of rapid eye movements).
b. Incorrect See explanation above.
c. Incorrect See explanation above.

d. Incorrect See explanation above.
The correct answer is: decreased REM latency and increased REM density.

How well did you know this?
1
Not at all
2
3
4
5
Perfectly
14
Q

Compared to adolescents and adults without ADHD, those with ADHD:
Select one:

A. are at an equal or reduced risk for substance use and substance use disorders.
B. are at a higher risk for substance use disorders only when they have a comorbid internalizing disorder.
C. are at a higher risk for substance use disorders only when they received psychostimulant medication as a treatment for ADHD in childhood.
D. are at a higher risk for substance use disorders with the risk being greatest for those with comorbid conduct problems.

A

Although the research confirms an association between ADHD and substance use and abuse in adolescents and adults, the studies have also found that several factors mediate this relationship.

a. Incorrect The research on this issue is not entirely consistent, but a number of studies have found that adolescents and adults with ADHD (especially adults in their 20s) have higher rates of substance use and substance use disorders than do those without ADHD.
b. Incorrect A higher risk for substance abuse individuals with ADHD has been established for comorbid externalizing (conduct/antisocial) disorders, not for comorbid internalizing disorders.
c. Incorrect In fact, there is evidence that children with ADHD who receive psychostimulant medication are at lower risk for developing a substance use disorder than are those with ADHD who do not receive medication. See, e.g., J. Biederman et al., Pharmacotherapy of attention deficit/hyperactivity disorder reduces risk for substance use disorders, Pediatrics, 104, 293-294, 1999.
d. CORRECT There is evidence that the link between ADHD and substance abuse is mediated by conduct problems - i.e., that the risk for substance abuse is substantially greater for those individuals with comorbid conduct disturbances. See, e.g., J. J. McGough et al., Psychiatric comorbidity in adult attention deficit hyperactivity disorder: Findings from multiplex families, American Journal of Psychiatry, 162, 1621-1627, 2005.

The correct answer is: are at a higher risk for substance use disorders with the risk being greatest for those with comorbid conduct problems.

How well did you know this?
1
Not at all
2
3
4
5
Perfectly
15
Q

A new client tells the clinician that she’s been feeling “down in the dumps” and “completely worthless” for the last five or six days. She also says she’s having trouble sleeping, has lost her appetite, and feels anxious “all the time.” The client denies having previous mood problems and states that there are no particular stressors in her life. The client’s symptoms are most suggestive of which of the following DSM-5 diagnoses?
Select one:

A. Major Depressive Disorder
B. Other Specified Depressive Disorder
C. Other Specified Anxiety Disorder
D. Major Depressive Episode, single episode

A

Answer B is correct: This client has symptoms of Major Depressive Disorder, but they have persisted for less than two weeks which is the minimum duration required for the diagnosis. Consequently, a diagnosis of Other Specified Depressive Disorder with the reason “short-duration depressive episode” is the best fit for her symptoms.

Answer A: A diagnosis of Major Depressive Disorder requires a duration of symptoms of at least two weeks.

Answer C: Although the client is experiencing anxiety, her symptoms do not meet the criteria for an anxiety disorder. Also, the DSM-5 states that “anxious distress has been noted as a prominent feature of both bipolar and major depressive disorder” (p. 184), and the client is exhibiting several symptoms that are typical of depression.

Answer D: Major depressive episode is not a DSM-5 diagnosis but, instead, is an essential feature of Major Depressive Disorder and Bipolar II Disorder and a possible feature of Bipolar I Disorder.

The correct answer is: Other Specified Depressive Disorder

How well did you know this?
1
Not at all
2
3
4
5
Perfectly
16
Q

On a measure that assesses the “Big Five” personality factors, a high score on neuroticism, a low score on agreeableness, and a low score on conscientiousness are most likely to be obtained by a person with ____________ Personality Disorder.
Select one:

A. Obsessive-Compulsive
B. Narcissistic
C. Schizoid
D. Antisocial

A

Answer D is correct: The match between the Big Five personality traits and DSM’s personality disorders is summarized in T. Widiger et al., A description of the DSM-III-R and DSM-IV personality disorders with the five-factor model of personality, in P. T. Costa and T. A. Widiger (eds.), Personality disorders and the five-factor model of personality, Washington, DC, American Psychological Association, 1994. This is the pattern reported by these investigators for individuals with Antisocial Personality Disorder.

Answer A: A person with Obsessive-Compulsive Personality Disorder is likely to obtain a high score on some aspects of neuroticism, a low score on some aspects of agreeableness, and a high score on conscientiousness.

Answer B: A person with Narcissistic Personality Disorder would obtain a high score on some aspects of neuroticism, a low score on agreeableness, and a high score on some aspects of conscientiousness.

Answer C: A person with Schizoid Personality Disorder would obtain a low score on some aspects of neuroticism, a high score on one aspect of agreeableness (compliance), and a low score on one aspect of conscientiousness (achievement striving).

The correct answer is: Antisocial

How well did you know this?
1
Not at all
2
3
4
5
Perfectly
17
Q

A DSM-5 diagnosis of Schizophrenia requires the presence of two or more active-phase symptoms during a one-month period with at least one symptom being ____________ plus continuous signs of disturbance for least six months.
Select one:

A. negative symptoms
B. disorganized speech or disorganized behavior
C. delusions or hallucinations
D. delusions, hallucinations, or disorganized speech

A

Answer D is correct: The DSM-5 specifies that at least one of the active-phase symptoms must be delusions, hallucinations, or disorganized speech.

The correct answer is: delusions, hallucinations, or disorganized speech

How well did you know this?
1
Not at all
2
3
4
5
Perfectly
18
Q

“In periods of crisis, such as loss, abandonment, separation, failure, and loneliness, the individual can temporarily fall back on his internal world.” This statement reflects the basic assumptions of which of the following approaches?
Select one:

A. Gestalt therapy
B. reality therapy
C. object relations theory
D. cognitive-behavioral therapy

A

This statement was made by Otto Kernberg (Object relations theory and clinical psychoanalysis, New York: Jason-Aronson, 1976).

a. Incorrect See explanation for response c.
b. Incorrect See explanation for response c.
c. CORRECT Kernberg’s statement reflects the object relations view that early social relations impact future social situations.
d. Incorrect See explanation for response c.

The correct answer is: object relations theory

How well did you know this?
1
Not at all
2
3
4
5
Perfectly
19
Q

Recent research suggests that higher rates of Schizophrenia among African-Americans is most likely attributable to:
Select one:

A. heredity.
B. family dynamics.
C. socioeconomic factors.
D. misdiagnosis of hallucinations and delusions.

A

Studies in both the United States and England have found that members of certain minority groups (especially groups devalued by the dominant culture) are more likely to receive a diagnosis of Schizophrenia.

a. Incorrect See explanation for response d.
b. Incorrect See explanation for response d.
c. Incorrect See explanation for response d.
d. CORRECT Research suggests that the discrepancy in rates of Schizophrenia is due to misdiagnosis rather than to actual differences in the prevalence of the disorder. A contributing factor to misdiagnosis may be that African-Americans have a higher incidence of hallucinations and delusions that are actually due to other disorders such as mania, depression, and alcohol abuse.

The correct answer is: misdiagnosis of hallucinations and delusions.

How well did you know this?
1
Not at all
2
3
4
5
Perfectly
20
Q

The characteristic symptoms of Tobacco Withdrawal include all of the following except:
Select one:

A. irritability or anger.
B. hypersomnia.
C. impaired concentration.
D. increased appetite.

A

Answer B is correct: As described in the DSM-5, Tobacco Withdrawal is characterized by the development of at least four characteristic symptoms within 24 hours of abrupt cessation or reduction in the use of tobacco - i.e., irritability or anger, anxiety, impaired concentration, increased appetite, restlessness, depressed mood, and/or insomnia.

Answers A, C, and D: See explanation for answer B.
The correct answer is: hypersomnia.

How well did you know this?
1
Not at all
2
3
4
5
Perfectly
21
Q

The presence of which of the following symptoms would be more suggestive of a diagnosis of Caffeine-Induced Anxiety Disorder than Caffeine Intoxication?
Select one:

A. tachycardia or cardiac arrhythmia
B. psychomotor agitation
C. panic attacks
D. rambling flow of thoughts and speech

A

The differential diagnosis of Caffeine-Induced Anxiety Disorder and Caffeine Intoxication can be difficult due the overlap between the symptoms for the two disorders.

a. Incorrect A diagnosis of Caffeine Intoxication requires the presence of at least five characteristic symptoms that develop during or shortly after the consumption of caffeine. Tachycardia or cardiac arrhythmia is one of the characteristic signs of Caffeine Intoxication.
b. Incorrect Psychomotor agitation is a symptom of Caffeine Intoxication.
c. CORRECT Caffeine-Induced Anxiety Disorder would be the more likely diagnosis when the patient’s symptoms of anxiety are in excess of those associated with Caffeine Intoxication - e.g., when the patient has signs of Generalized Anxiety Disorder, panic attacks, or obsessive-compulsive symptoms.
d. Incorrect Rambling flow of thoughts and speech are also characteristic of Caffeine Intoxication.

The correct answer is: panic attacks

How well did you know this?
1
Not at all
2
3
4
5
Perfectly
22
Q

The core feature of the DSM-5 diagnosis of Gender Dysphoria is which of the following?
Select one:

A. persistent preference for cross-gender roles
B. strong desire to be treated as the opposite gender
C. incongruence between assigned and experienced/expressed gender
D. cross-gender identification

A

Answer C is correct: Incongruence between assigned gender and experienced/expressed gender is identified as the core feature of Gender Dysphoria in the DSM-5.

Answer A: A persistent preference for cross-gender roles in make-believe or fantasy play is one of the possible symptoms for children with Gender Dysphoria, but it is not the core feature.

Answer B: The DSM-5 lists “a strong desire to be treated as the other gender (or some alternative gender different from one’s assigned gender)” (p. 452) as one of the possible symptoms of Gender Dysphoria for adolescents or adults.

Answer D: Cross-gender identification is not identified as the core feature of this disorder in the DSM-5.

The correct answer is: incongruence between assigned and experienced/expressed gender

How well did you know this?
1
Not at all
2
3
4
5
Perfectly
23
Q

In comparison to bilateral ECT, unilateral ECT to the right (nondominant) hemisphere:
Select one:

A. produces similar levels of anterograde and retrograde amnesia.
B. produces greater anterograde amnesia for nonverbal tasks only.
C. produces greater anterograde and retrograde amnesia for both verbal and nonverbal tasks.
D. produces less anterograde amnesia for both verbal and nonverbal tasks.

A

Memory loss is considered one of the primary negative side-effects of ECT. There is evidence, however, that memory loss with right unilateral ECT is less severe than with bilateral ECT.

a. Incorrect See explanation for response d.
b. Incorrect See explanation for response d.
c. Incorrect See explanation for response d.
d. CORRECT Right unilateral ECT not only produces less retrograde amnesia but also less anterograde amnesia for verbal and nonverbal tasks.

The correct answer is: produces less anterograde amnesia for both verbal and nonverbal tasks.

How well did you know this?
1
Not at all
2
3
4
5
Perfectly
24
Q

The intentional production of a physical symptom for the specific purpose of obtaining financial benefits is characteristic of which of the following?
Select one:

A. Malingering
B. Factitious Disorder
C. Conversion Disorder
D. Somatic Symptom Disorder

A

Answer A is correct: A characteristic that distinguishes Malingering from other disorders with somatic symptoms is that the goal of Malingering is to obtain an external incentive.

Answer B: Although Factitious Disorder involves the intentional production of symptoms, its goal is not to obtain an external reward.

Answers C and D: Conversion Disorder and Somatic Symptom Disorder both involve somatic symptoms, but the goal is not to obtain an external incentive.

The correct answer is: Malingering

How well did you know this?
1
Not at all
2
3
4
5
Perfectly
25
Q

A child with Oppositional Defiant Disorder is least likely to exhibit which of the following?
Select one:

A. frequent conflicts with his/her parents
B. a high degree of physical aggression toward peers
C. frequently blames others for his/her mistakes
D. often loses temper

A

Answer B is correct: Oppositional Defiant Disorder is characterized by a pattern of angry/irritable mood, argumentative/defiant behavior, or vindictiveness. Although children with this disorder may be verbally aggressive, a high degree of physical aggression is not characteristic and may be a sign of comorbid Conduct Disorder.

Answers A, C, and D: See explanation for answer B.

The correct answer is: a high degree of physical aggression toward peers

How well did you know this?
1
Not at all
2
3
4
5
Perfectly
26
Q

Which of the following is not a DSM-5 diagnostic criterion for Conversion Disorder?
Select one:

A. one or more symptoms that involve an alteration in sensory or motor functioning
B. evidence of incompatibility between the symptoms and known medical conditions
C. symptoms are not better explained by a medical condition or other mental disorder
D. evidence that the symptoms are related to a psychological need or conflict.

A

Answer D is correct: This was a requirement for a diagnosis of Conversion Disorders in previous versions of the DSM, but it is not a diagnostic criterion in DSM-5.

Answers A, B, and C: These answers describe the DSM-5’s diagnostic criteria for Conversion Disorder.

The correct answer is: evidence that the symptoms are related to a psychological need or conflict.

How well did you know this?
1
Not at all
2
3
4
5
Perfectly
27
Q

Which of the following is NOT included as an Alcohol-Related Disorder in the DSM-5?
Select one:

A. Alcohol Dependence
B. Alcohol Intoxication
C. Alcohol Withdrawal
D. Alcohol-Induced Major Neurocognitive Disorder

A

Answer A is correct: Alcohol Dependence was a separate diagnosis in previous versions of the DSM but was replaced in the DSM-5 by Alcohol Use Disorder.

Answer B: Alcohol Intoxication is included as a diagnosis in the DSM-5 and involves maladaptive behavioral and psychological changes with at least one characteristic symptom (e.g., slurred speech; unsteady gait; nystagmus; impaired attention or memory).

Answer C: The DSM-5 describes Alcohol Withdrawal as involving the development of at least two characteristic symptoms within several hours to a few days following cessation or reduction of alcohol consumption (e.g., autonomic hyperactivity, hand tremor, insomnia, transient illusions or hallucinations, seizures).

Answer D: Alcohol-Induced Major Neurocognitive Disorder is described in the DSM-5 as involving a significant decline in one or more cognitive domains that interferes with independence in everyday activities.

The correct answer is: Alcohol Dependence

How well did you know this?
1
Not at all
2
3
4
5
Perfectly
28
Q

In its early stages, Alzheimer’s disease shares several characteristics with Korsakoff’s syndrome. Specifically, both disorders are characterized by:
Select one:

A. anterograde amnesia that affects declarative memories but not procedural memories.
B. anterograde amnesia that affects both declarative and procedural memories.
C. retrograde amnesia for recent and remote declarative (but not procedural) memories.
D. retrograde amnesia for recent (but not remote) declarative and procedural memories.

A

Alzheimer’s disease and Korsakoff’s syndrome share several characteristics.

a. CORRECT In Alzheimer’s disease, anterograde amnesia for declarative knowledge is often the first neurological finding. It is also characteristic of Korsakoff’s syndrome.
b. Incorrect Procedural memory is left relatively intact in both disorders, and confabulation is characteristic of Korsakoff’s syndrome but not Alzheimer’s disease.
c. Incorrect In Alzheimer’s disease, there is a “flat” retrograde amnesia that emerges once the disorder has progressed to moderate levels of impairment (i.e., the amnesia affects both remote and recent memories). In Korsakoff’s syndrome, the retrograde amnesia affects recent memories while remote memories remain intact. See, e.g., N. Butters and D. C. Delis, Clinical assessment of memory disorders in amnesia and dementia, Annual Review of Psychology, 46, 493-523, 1995.

d. Incorrect See explanation for response c.
The correct answer is: anterograde amnesia that affects declarative memories but not procedural memories.

How well did you know this?
1
Not at all
2
3
4
5
Perfectly
29
Q

Research examining the relationship between mild depression and autobiographical memory has found that, in comparison to the memories of people who are nondysphoric, the memories of people who are dysphoric are:
Select one:

A. less detailed and more negative.
B. more detailed and more negative.
C. less detailed and more neutral.
D. more detailed and more neutral.

A

Not surprisingly, the studies have linked depression to an increased memory for negative events.

a. CORRECT People who are dysphoric are also likely to have autobiographical memories that are less detailed than those of people who are nondysphoric. See, e.g., J. M. G. Williams & J. Scott, Autobiographical memory in depression, Psychological Medicine, 18,689-695, 1986.
b. Incorrect See explanation above.
c. Incorrect See explanation above.
d. Incorrect See explanation above.

The correct answer is: less detailed and more negative.

How well did you know this?
1
Not at all
2
3
4
5
Perfectly
30
Q

For a diagnosis of Separation Anxiety Disorder, the DSM-5 requires symptoms of fear, anxiety, or avoidance that have lasted for at least __________ in children and adolescents or about __________ in adults.
Select one:

A. one week; six months
B. four weeks; six months
C. six weeks; eight months
D. two months; eight months

A

Answer B is correct: The DSM-5 specifies a minimum duration of symptoms for Separation Anxiety Disorder of “4 weeks in children and adolescents and typically 6 months in adults” (p. 191).

Answers A, C, and D: See explanation above.

The correct answer is: four weeks; six months

How well did you know this?
1
Not at all
2
3
4
5
Perfectly
31
Q

An emphasis on contact, awareness, and experimentation is central to which of the following?
Select one:

A. Gestalt therapy
B. solution-focused therapy
C. Adlerian therapy
D. rational-emotive therapy

A

This is one of those questions you may have been able to answer correctly even if you had incomplete information - i.e., if you knew that the goal of Gestalt therapy is to achieve conscious awareness, you would have been able to identify response a as the correct answer.

a. CORRECT Contact, conscious awareness, and experimentation are key targets in Gestalt therapy. Contact refers to being in touch with what is happening in the here-and-now; awareness refers to focused attention in situations requiring it; and experimentation is the act of trying something new in order to increase understanding.
b. Incorrect See explanation for response a.
c. Incorrect See explanation for response a.
d. Incorrect See explanation for response a.

The correct answer is: Gestalt therapy

How well did you know this?
1
Not at all
2
3
4
5
Perfectly
32
Q

Sue and Sue (1990) describe the worldview of counselors and their clients in terms of two dimensions - locus of control and locus of responsibility - and propose that members of minority groups are becoming increasingly more likely to exhibit which of the following worldviews?
Select one:

A. internal locus of control/internal locus of responsibility
B. external locus of control/internal locus of responsibility
C. internal locus of control/external locus of responsibility
D. external locus of control/external locus of responsibility

A

D. W. Sue and D. Sue (Counseling the culturally different, New York, John Wiley, 1990) propose that an understanding of different worldviews and their impact adds to the effectiveness of cross-cultural counseling.

a. Incorrect- An internal locus of control/internal locus of responsibility is characteristic of the Westernized approach to counseling, white counselors, and white middle-class clients.
b. Incorrect- An external locus of control/internal locus of responsibility is characteristic of “marginal” individuals who have been oppressed by the dominant group.
c. CORRECT- Sue and Sue propose that minority group members are becoming increasingly aware of their own ethnic identity and the impact of racism on their lives - i.e., they are adopting a worldview that reflects an internal locus of control and an external locus of responsibility.
d. Incorrect- An external locus of control/external locus of responsibility is characteristic of a “placater” who adopts a passive role.

The correct answer is: internal locus of control/external locus of responsibility

How well did you know this?
1
Not at all
2
3
4
5
Perfectly
33
Q

You are conducting a therapy group and you are now in the middle stage. You ask group members to shift from answering direct questions to talking freely about themselves. The group members show resistance to this. You should:
Select one:

A. allow them to express their resistance.
B. explain to them the developmental process of the group.
C. tell them that resistance is not allowed.
D. go back to asking direct questions.

A

In group therapy, resistance can take several forms and occurs most often in the early middle stage of therapy when members try to establish their preferred position in the group.

a. CORRECT By definition, resistance is a covert effort to undermine change; thus, it is best handled by being brought out into the open. Discussions should focus on the here-and-now and deal with what the resistance implies.
b. Incorrect While the developmental process of the group might be of interest to some members, this approach would do little, if anything, to deal with resistance.
c. Incorrect This strategy would probably engender more intense resistance.
d. Incorrect Capitulation to resistance amounts to reinforcement and is likely to increase its occurrence.

The correct answer is: allow them to express their resistance.

How well did you know this?
1
Not at all
2
3
4
5
Perfectly
34
Q

For members of minority groups, long-term exposure to racial oppression is most likely to result in:
Select one:

A. downplaying the importance of ethnicity and adopting a “culture-free” identity.
B. focusing on satisfying physiological needs and acquiring basic survival skills.
C. attempting to earn acceptance by acquiring material goods and other signs of status.
D. becoming socially withdrawn and isolated.

A

The impact of long-term racial oppression has been discussed by J. Landrum and V. Batts (Helping blacks cope with and overcome the personal effects of racism, paper presented at the annual meeting of the American Psychological Association, Los Angeles, 1985).

a. Incorrect According to Landrum and Batts, it is more likely that an individual who has been oppressed will identify with the dominant culture or, alternatively, reject the dominant culture and develop a separate ethnic identity.
b. Incorrect This is not one of the outcomes identified by Landrum and Batts.
c. CORRECT One response to racism is internalized oppression, which can be manifested in several ways including denying the impact of race and racism and attempting to earn acceptance of the dominant group through material possessions and educational or occupational status.
d. Incorrect This has not been identified as a common response to oppression.

The correct answer is: attempting to earn acceptance by acquiring material goods and other signs of status.

How well did you know this?
1
Not at all
2
3
4
5
Perfectly
35
Q

For Hispanic/Latino American families, the strongest family bond is most often between:
Select one:

A. husband and wife.
B. parent and child.
C. brother and sister.
D. grandparent and grandchild.

A

There is, of course, considerable variability in Hispanic/Latino American families, but, traditionally, family structure and relationships have had a predictable pattern.

a. Incorrect See explanation for response b.
b. CORRECT In traditional Hispanic/Latino and Hispanic/Latino American families, the parent-child relationship (and especially the mother-son and father-son relationship) is stronger than the husband-wife and other relationships listed in the answers to this question. See, e.g., C. Falicov, Latino families in therapy, New York, Guilford, 1998.
c. Incorrect See explanation above.
d. Incorrect See explanation above.

The correct answer is: parent and child.

How well did you know this?
1
Not at all
2
3
4
5
Perfectly
36
Q

During the first session with a new client, a solution-focused therapist will:
Select one:

A. conduct a thorough screening and history-taking.
B. help the client identify and describe therapy goals in concrete, positive terms.
C. educate the client about the causes of his/her presenting problem and alternative techniques for alleviating the problem.
D. identify the client’s stage of change.

A

As its name implies, solution-focused therapy focuses on solutions to problems rather than on the problems themselves.

a. Incorrect See explanation for response b.
b. CORRECT Stating goals in positive concrete (behavioral) terms helps the client recognize when he/she is accomplishing his/her goals and has been found to be a good predictor of outcome in solution-focused therapy.
c. Incorrect See explanation for response b.
d. Incorrect See explanation for response b.

The correct answer is: help the client identify and describe therapy goals in concrete, positive terms.

How well did you know this?
1
Not at all
2
3
4
5
Perfectly
37
Q

In schools, psychological consultants are often hired to help teachers or other school personnel resolve problems related to specific individual students. This is referred to as:
Select one:

A. consultee-centered case consultation.
B. client-centered case consultation.
C. consultee-centered administrative consultation.
D. client-centered administrative consultation.

A

In this situation, a consultant is working with a consultee to resolve a problem with a particular client (case).

a. Incorrect In consultee-centered consultation, the focus is on the consultee’s skills, abilities, and objectivity so that he/she can work more effectively with a particular group of clients in the future.
b. CORRECT The question describes client-centered case consultation, which is the most common form of consultation in schools and elsewhere.
c. Incorrect This type of consultation focuses on helping consultees (administrators) develop the skills they need to more effectively develop and implement programs.
d. Incorrect This is a “made-up” term.

The correct answer is: client-centered case consultation.

How well did you know this?
1
Not at all
2
3
4
5
Perfectly
38
Q

A client is acting in a passive-aggressive way toward a psychology intern who, in turn, becomes unconsciously passive-aggressive toward her supervisor. This is an example of which of the following?
Select one:

A. transference
B. countertransference
C. over-involvement
D. parallel process

A

In this situation, the intern’s behavior with the counselor is “mirroring” the client’s behavior with the intern.

a. Incorrect See explanation for response d.
b. Incorrect See explanation for response d.
c. Incorrect See explanation for response d.
d. CORRECT This type of mirroring is referred to as parallel process and is conceptualized as a reflection of the transference/countertransference process (which makes responses a and b only partially correct).

The correct answer is: parallel process

How well did you know this?
1
Not at all
2
3
4
5
Perfectly
39
Q

An African American adult in which stage of Cross’s (1991) Black Racial Identity Development Model denigrates all aspects of White culture while idealizing all aspects of African American culture?
Select one:

A. internalization
B. encounter
C. immersion-emersion
D. reintegration

A

Cross distinguishes between four stages of identity development: pre-encounter, encounter, immersion-emersion, and internalization.

a. Incorrect A person in the internalization stage has adopted an African American worldview but is flexible in his/her attitudes toward Whites.
b. Incorrect During the encounter stage, the individual becomes increasingly aware of his/her racial heritage.
c. CORRECT During the immersion-emersion stage, the individual withdraws from the dominant (White) culture and becomes immersed in African American culture.
d. Incorrect Reintegration is not one of the stages included in Cross’s model.

The correct answer is: immersion-emersion

How well did you know this?
1
Not at all
2
3
4
5
Perfectly
40
Q

For Salvador Minuchin, “psychosomatic families” (e.g., those in which asthma, diabetes, or anorexia threaten the life of one child) are most likely to be characterized by which of the following?
Select one:

A. frequent intense open conflicts between family members
B. weak boundaries between family members and limited opportunities for individual autonomy
C. family roles that are inflexible and stereotyped
D. marked emotional distance between the husband and wife who are both emotionally immature

A

Minuchin is associated with structural family therapy, which emphasizes the nature of the family structure including the boundaries between family members.

a. Incorrect In fact, according to Minuchin, psychosomatic families are characterized by a consistent avoidance of conflict.
b. CORRECT According to Minuchin, diffuse (weak) boundaries are common in psychosomatic families.
c. Incorrect Ackerman is more associated with the importance of roles in family dysfunction.
d. Incorrect This describes “emotional divorce,” which is a term used by Bowen.

The correct answer is: weak boundaries between family members and limited opportunities for individual autonomy

How well did you know this?
1
Not at all
2
3
4
5
Perfectly
41
Q

From the perspective of Cross’s (1991, 2001) Black Racial Identity Development Model, change in the direction of greater racial or ethnic awareness is:
Select one:

A. a natural consequence of a person’s developmental level.
B. the result of exposure to events that increased the salience of race.
C. the result of peer influence.
D. one of the possible resolutions of an identity crisis.

A

Cross’s model distinguishes between four stages of identity development: pre-encounter, encounter, immersion-emersion, and internalization.

a. Incorrect See explanation for response b.
b. CORRECT The stages of Cross’s model are associated with different levels of race salience. For example, during the pre-encounter stage, race has low salience; but then exposure to one or more race-related events increases the salience of race and the individual progresses into the encounter stage.
c. Incorrect See explanation for response b.
d. Incorrect See explanation for response b.

The correct answer is: the result of exposure to events that increased the salience of race.

How well did you know this?
1
Not at all
2
3
4
5
Perfectly
42
Q

Which of the following terms is least descriptive of the Lockean tradition underlying the Western approach to individual psychotherapy?
Select one:

A. linear-cause effect
B. reductionistic
C. individualistic
D. relativistic

A

Most individual therapies - e.g., Freudian psychoanalysis, Jung’s analytical psychology, Adler’s individual psychology, and Rogerian therapy - reflect a Lockean approach, which emphasizes linear cause-effect relationships, value-free science, and a deterministic and reductionistic perspective. See, e.g., D. Becvar and R. Becvar, Family therapy: A Systemic integration, Boston: Allyn and Bacon, 1993.

a. Incorrect This is characteristic of the Lockean tradition.
b. Incorrect This is also characteristic of the Lockean tradition.
c. Incorrect The Lockean tradition emphasizes individualism over collectivism.
d. CORRECT Relativism is more characteristic of non-western and systems approaches. The Lockean tradition reflects an absolutist approach.

The correct answer is: relativistic

How well did you know this?
1
Not at all
2
3
4
5
Perfectly
43
Q
Gregory Herek (1992) attributes violence against gays and lesbians to cultural and psychological:
Select one:

A. heterosexism.
B. homophobia.
C. paranoia.
D. bigotry.

A

In his discussion of violence against gays and lesbians, G. Herek distinguishes between cultural and psychological heterosexism [Psychological heterosexism and anti-gay violence: The social psychology of bigotry. In G. M. Herek & K. T. Berrill, Hate crimes: Confronting violence against lesbians and gay men (pp. 149-169), Newbury Park, Sage, 1992].

a. CORRECT As defined by Herek, heterosexism is an “ideological system that denies, denigrates, and stigmatizes among nonheterosexual forms of behavior, identity, relationships, or community” (p. 150). He views violence against gays and lesbians to be due to a combination of psychological (individual) and cultural heterosexism.
b. Incorrect See explanation above.
c. Incorrect See explanation above.
d. Incorrect See explanation above.

The correct answer is: heterosexism.

How well did you know this?
1
Not at all
2
3
4
5
Perfectly
44
Q

For Carl Jung, transference is:
Select one:

A. a form of projection.
B. sexual “acting out.”
C. symbolization.
D. a fantasy.

A

For Jung, transference is a specific form of the more general process of projection.

a. CORRECT Jung believed that, when transference occurs in therapy, elements of the client’s personal and collective (transpersonal) unconscious are projected onto the therapist.
b. Incorrect Jung disagreed with Freud that all transferences have a sexual nature.
c. Incorrect Although symbols are an important part of Jung’s theory, he did not describe transference as “symbolization.”
d. Incorrect Jung did not describe transference as a form of fantasy.

The correct answer is: a form of projection.

How well did you know this?
1
Not at all
2
3
4
5
Perfectly
45
Q

In terms of group therapy, premature termination has been found to depend most on the members’:
Select one:

A. locus of control.
B. expectations.
C. psychological mindedness.
D. diagnosis

A

One of the most consistent findings of the research on group therapy is that people who terminate prematurely from group treatment are those who have unrealistic expectations and unfavorable attitudes.

a. Incorrect See explanation for response b.
b. CORRECT Because of the relationship between unrealistic expectations and premature termination, experts suggest that, to ensure the success of a therapy group, members should be selected through prescreening or should be given some type of pregroup training.
c. Incorrect See explanation for response b.
d. Incorrect See explanation for response b.

The correct answer is: expectations.

How well did you know this?
1
Not at all
2
3
4
5
Perfectly
46
Q

For an extended family systems (Bowenian) therapist, when a family exhibits a high degree of fusion but one member is more differentiated than the others, the likely approach would be to:
Select one:

A. work with the least differentiated family members.
B. work with the most differentiated family member.
C. work with all members of the extended family as a group.
D. work with all family members in individual therapy until they reach a similar level of differentiation.

A

According to Murray Bowen, a high degree of fusion (emotional “stuck togetherness”) is the source of family dysfunction. The goal of Bowen’s form of family therapy, therefore, is to help family members become less fused, or more differentiated (individuated). In contrast to many other family therapists, Bowen’s form of therapy typically does not involve seeing all of the family members.

a. Incorrect Followers of Bowen would likely work first with the most differentiated family member.
b. CORRECT Bowen often worked with the most differentiated family member on the assumption that, when that member changed in a positive way, his/her change would motivate other family members to move toward greater differentiation.
c. Incorrect In contrast to many other family therapists, Bowen typically worked with the two most significant adult family members, usually the parents, or the parent who was the most differentiated, even when the “identified patient” was a child.
d. Incorrect This is not a technique used by Bowen.

The correct answer is: work with the most differentiated family member.

How well did you know this?
1
Not at all
2
3
4
5
Perfectly
47
Q

Nonsexist therapy, in comparison to feminist therapy, places greater emphasis on:
Select one:

A. the impact of sex roles and sexism.
B. modification of personal behavior.
C. treating men and women the same.
D. the egalitarian nature of the therapeutic relationship.

A

To a large degree, nonsexist therapy and feminist therapy overlap.

a. Incorrect This is characteristic of both forms of therapy.
b. CORRECT Of the answers given, this is the best choice. While feminist therapy produces change in personal behavior, that change is more consistently framed within the sociopolitical arena in which it occurs. In contrast, nonsexist therapy, while recognizing sociopolitical influences, also looks at personal change separately.
c. Incorrect Both types of therapy advocate that men and women be treated as equals but not necessarily the same.
d. Incorrect This is an explicit goal of feminist therapy and, perhaps to a lesser degree, a characteristic of nonsexist therapy.

The correct answer is: modification of personal behavior.

How well did you know this?
1
Not at all
2
3
4
5
Perfectly
48
Q

Which of the following best describes the results of research on client and provider satisfaction with the use of videoconferencing and other forms of telehealth to deliver mental health services?
Select one:

A. Clients express high levels of satisfaction with telehealth but providers express low levels of satisfaction.
B. Providers express high levels of satisfaction with telehealth but clients express low levels of satisfaction.
C. Clients and providers both express satisfaction with telehealth and most clients express a stronger preference for telehealth than for in-person services.
D. Clients and providers both express satisfaction with telehealth but some providers also express concerns about its effectiveness.

A

Research on telehealth (which, in the context of mental health, is also known as telepsychology, telepsychiatry, and telemental health) has generally found that satisfaction ratings are high among both clients and providers.

a. Incorrect See explanation for response d.
b. Incorrect See explanation for response d.
c. Incorrect Although clients are satisfied with telehealth, they express similar levels of preference for telehealth and in-person services.
d. CORRECT The research has focused primarily on clients and has found that they generally express high levels of satisfaction with telehealth. Studies looking at providers has confirmed that they also tend to be satisfied but that there is greater variability in their satisfaction ratings due to concerns about its effectiveness. See, e.g., J. Monnier, R. G. Knapp, and B. C. Frueh, Recent advances in telepsychiatry: An updated review, Psychiatric Services, 54, 1604-1609, 2003.

The correct answer is: Clients and providers both express satisfaction with telehealth but some providers also express concerns about its effectiveness.

How well did you know this?
1
Not at all
2
3
4
5
Perfectly
49
Q

As defined by Wrenn (1985), a “culturally encapsulated” counselor:
Select one:

A. has had little or no exposure to individuals from different cultures.
B. disregards cultural differences and his/her own cultural biases.
C. prioritizes culture as an etiological factor in mental illness.
D. recognizes that universal principles of behavior do not apply to everyone.

A

For the exam, you want to be familiar with the characteristics of a culturally encapsulated counselor. Additional information about these characteristics is provided in the Clinical Psychology chapter of the written study materials.

a. Incorrect See explanation for response b.
b. CORRECT According to Wrenn, a culturally encapsulated counselor disregards cultural differences and defines everyone’s reality according to his/her own cultural assumptions and stereotypes.
c. Incorrect See explanation for response b.
d. Incorrect See explanation for response b.

The correct answer is: disregards cultural differences and his/her own cultural biases.

How well did you know this?
1
Not at all
2
3
4
5
Perfectly
50
Q

Although the various models of homosexual (gay/lesbian) identity development differ somewhat in terms of the number and nature of the stages they propose, most (or all) describe a person in the initial stage as:
Select one:

A. revealing to another person that he/she is not sexually attracted to people of the opposite gender.
B. actively denying his/her sexual attraction toward same-gender individuals.
C. recognizing that he/she is different from same-gender peers.
D. first realizing that he/she is sexually attracted to people of the same gender.

A

Being familiar with Troiden’s (1988) homosexual identity development model would have helped you recognize the correct answer to this question.

a. Incorrect See explanation for response c.
b. Incorrect See explanation for response c.
c. CORRECT Troiden’s initial stage is the sensitization stage and occurs prior to puberty. During this stage, the individual feels socially different from same-gender peers. For example, a girl may say that, unlike her female friends, she’s not interested in boys or that she’s more independent or aggressive than other girls; while a boy might say that he’s less interested in sports and more interested in arts than his male peers.
d. Incorrect See explanation for response c.

The correct answer is: recognizing that he/she is different from same-gender peers.

How well did you know this?
1
Not at all
2
3
4
5
Perfectly
51
Q

A therapy client’s negative transference is most likely to be manifested in which of the following ways?
Select one:

A. The client will criticize the therapist and be pessimistic about the outcomes of therapy.
B. The client will be more willing to talk about anxiety-arousing events.
C. The client will say whatever comes to his/her mind.
D. The client will express unrealistic positive expectations for the outcomes of therapy.

A

The interpretation of a client’s transferences is an important part of psychodynamic psychotherapy.

a. CORRECT Negative transference involves the projection of negative emotions onto the therapist and may manifest itself several ways including making direct, negative comments about the effectiveness of therapy or being late for or missing appointments.
b. Incorrect Transference is a form of resistance and, consequently, is not likely to increase a client’s willingness to talk about anxiety-arousing events.
c. Incorrect This describes the psychodynamic technique of free association and is not how a client would be likely to manifest transference.
d. Incorrect Unrealistic positive expectations for the outcomes of therapy are more likely to be a manifestation of positive transference.

The correct answer is: The client will criticize the therapist and be pessimistic about the outcomes of therapy.

How well did you know this?
1
Not at all
2
3
4
5
Perfectly
52
Q

Structural family therapy entails three overlapping steps or stages. Which of the following techniques is/are most useful during the first step?
Select one:

A. reframing and relabeling
B. tracking and mimesis
C. constructing a family map
D. enactment

A

The three steps in structural family therapy are joining, evaluating/diagnosing, and restructuring.

a. Incorrect Reframing and relabeling are restructuring techniques.
b. CORRECT Tracking (identifying and using the family’s values, life themes, etc.) and mimesis (adopting the family’s affective and behavioral style) are methods used to join the family system. Joining is the initial step in structural family therapy.
c. Incorrect A family map is used to facilitate the structural diagnosis of the family.
d. Incorrect Enactment is a technique used to facilitate diagnosis and restructuring of the family.

The correct answer is: tracking and mimesis

How well did you know this?
1
Not at all
2
3
4
5
Perfectly
53
Q

A client who is a member of a minority group expresses negative feelings about her own culture and positive feelings toward the dominant (Anglo) culture. From the perspective of Atkinson, Morten, and Sue’s (1993) Racial/Cultural Identity Development Model, this client is in which stage?
Select one:

A. conformity
B. dissonance
C. immersion
D. denial

A

The Racial/Cultural Identity Development Model distinguishes between five stages: conformity; dissonance; resistance and immersion; introspection; and integrative awareness.

a. CORRECT This stage is characterized by a preference for the dominant culture over one’s own culture.
b. Incorrect This stage is characterized by cultural conflict and confusion.
c. Incorrect The immersion stage is marked by an active rejection of the dominant (Anglo) society and acceptance of one’s own culture.
d. Incorrect Denial is not one of the stages of this model.

The correct answer is: conformity

How well did you know this?
1
Not at all
2
3
4
5
Perfectly
54
Q

A therapist tells a client who wants to quit smoking that he should keep his only pack of cigarettes in a place that is accessible only by walking for more than one mile. This is an example of which of the following paradoxical techniques?
Select one:

A. reframing
B. symptom prescription
C. ordeal
D. restraining

A

In this situation, smoking becomes self-punishing because of the amount of work (walking one mile) it will take in order to smoke.

a. Incorrect Reframing involves helping the client see a behavior in a different (and usually more positive) way.
b. Incorrect Symptom prescription (aka prescribing the symptom) involves instructing the client to engage in the undesirable behavior in order to help the client recognize that the behavior is under his/her control.
c. CORRECT Haley (1984) described several types of ordeals. One type involves having the individual perform an unpleasant or inconvenient act in order to engage in the target behavior - which is what this question describes.
d. Incorrect Restraining involves encouraging the client not to change in order to elicit resistance and thereby facilitate change.

The correct answer is: ordeal

How well did you know this?
1
Not at all
2
3
4
5
Perfectly
55
Q

Dr. Wilson has been treating a heterosexual couple for sexual problems for three weeks. His first intervention was to instruct the couple to use the squeeze technique. When this technique had beneficial results, he then instructed the couple to attempt intercourse using the female-superior position. Based on this information, you can conclude that:
Select one:

A. the husband met the diagnostic criteria for Premature Ejaculation.
B. the husband met the diagnostic criteria for Erectile Disorder.
C. the wife met the diagnostic criteria for Sexual Arousal Disorder.
D. the wife met the diagnostic criteria for Vaginismus.

A

Standard treatments for the sexual dysfunctions are based primarily on the work of Masters and Johnson (1970).

a. CORRECT Knowing that the squeeze technique is a treatment for Premature Ejaculation would have enabled you to identify this as the correct response. Note that the woman-superior (woman-on-top) position is used for this disorder as well as several other sexual dysfunctions.
b. Incorrect See explanation for response a.
c. Incorrect See explanation for response a.
d. Incorrect See explanation for response a.

The correct answer is: the husband met the diagnostic criteria for Premature Ejaculation.

How well did you know this?
1
Not at all
2
3
4
5
Perfectly
56
Q

Harry Stack Sullivan’s interpersonal approach to psychiatry identifies ____________ as the primary cause of psychopathology.
Select one:

A. unconscious conflicts
B. excessive anxiety
C. obstacles to self-actualization
D. avoidance of ultimate concerns

A

Sullivan believed that human behavior is motivated by two needs - the need for satisfaction and the need for security. The need for satisfaction is fulfilled by things that meet the individual’s biological needs (food, water, shelter, etc.), while the need for security is fulfilled by gratifying experiences with others.

a. Incorrect See explanation for response b.
b. CORRECT Sullivan emphasized the role of anxiety in personality development and psychopathology. As described by Sullivan, excessive anxiety is the result of interpersonal insecurity that can be traced to problems in interpersonal relationships (especially during infancy and childhood) and is the basis for most psychiatric problems.
c. Incorrect See explanation for response b.
d. Incorrect See explanation for response b.

The correct answer is: excessive anxiety

How well did you know this?
1
Not at all
2
3
4
5
Perfectly
57
Q

Dr. Wright, a licensed psychologist, has been seeing Aimee Ames in individual therapy for nearly six months. Aimee has just decided to divorce her husband and asks Dr. Wright if he will conduct custody evaluations of her three children. As an ethical psychologist, Dr. Wright:
Select one:

A. should conduct the custody evaluations only if he obtains the consent of Mr. Ames.
B. should conduct the custody evaluations only if he also evaluates Mr. Ames.
C. should conduct the custody evaluations only if he determines that his professional relationship with Mrs. Ames will not interfere with his objectivity.
D. should not conduct the custody evaluations.

A

This issue is addressed in Paragraph 7 of the Guidelines for Child Custody Evaluations in Family Law Proceedings (APA, 2010).

a. Incorrect See explanation for response d.
b. Incorrect See explanation for response d.
c. Incorrect See explanation for response d.
d. CORRECT This response is most consistent with Paragraph 7 of the Guidelines, which states that “psychologists strive to avoid conflicts of interest and multiple relationships in conducting evaluations.”

The correct answer is: should not conduct the custody evaluations.

How well did you know this?
1
Not at all
2
3
4
5
Perfectly
58
Q

During her first therapy session, a client tells you that she wants to pay for her sessions in cash and that she doesn’t want you to keep a record of her sessions or payments. If you agree to this arrangement:
Select one:

A. you are acting ethically as long as you ascertain that her reason for making this request is valid.
B. you are acting ethically since it is up to you to decide what to include in a client’s records.
C. you are acting ethically and legally as long as you report her payments as income on your income tax forms.
D. you are acting unethically and possibly illegally.

A

Record keeping is addressed in laws, organizational regulations, and ethical guidelines.

a. Incorrect See explanation for response d.
b. Incorrect See explanation for response d.
c. Incorrect See explanation for response d.
d. CORRECT This answer is most consistent with legal and ethical requirements. For example, Guideline 2 of the APA’s (2007) Record Keeping Guidelines states that a “psychologist strives to maintain accurate, current, and pertinent records of professional services as appropriate to the circumstances and may be required by the psychologist’s jurisdiction”; and Standard 6.01 of the APA’s (2002) Ethic Code presents reasons why it is important for psychologists to create and maintain adequate records.

The correct answer is: you are acting unethically and possibly illegally.

How well did you know this?
1
Not at all
2
3
4
5
Perfectly
59
Q

You are asked to assist with a single-parent therapy group. The therapist leading the group is also conducting a study designed to evaluate the psychological problems of parents who are raising their children alone. You become aware that the therapist has not informed group participants that they do not have to be included as subjects in the research project. Your first action should be to:
Select one:

A. immediately discontinue your participation in the group.
B. advise participants that they may withdraw from the group.
C. advise the therapist that he is acting unethically.
D. report the therapist to the local ethics committee.

A

APA’s Ethics Code requires that research participants be informed that they may withdraw at any time from the research. Thus, the therapist is acting unethically and appropriate action should be taken.

a. Incorrect See explanation for response c.
b. Incorrect See explanation for response c.
c. CORRECT In terms of response to an ethical violation, the Ethics Code recommends that a psychologist first attempt an informal resolution by discussing the violation with the violator. Thus, this response is most consistent with the provisions of the Ethics Code.
d. Incorrect See explanation for response c.

The correct answer is: advise the therapist that he is acting unethically.

How well did you know this?
1
Not at all
2
3
4
5
Perfectly
60
Q

A licensed psychologist who provides individual and group therapy to adolescents and adults learns that she is HIV+. In terms of ethical requirements, the psychologist:
Select one:

A. should refrain from initiating any professional activities that might be adversely affected by her medical condition.
B. should obtain supervision to ensure that her medical condition does not impair her ability to provide effective services to clients.
C. should inform her clients of her medical condition “as early as is feasible.”
D. is not obligated to take any special actions or precautions in this situation.

A

Standard 2.06 of the Ethics Code applies to this situation.

a. CORRECT Standard 2.06 requires psychologists to “refrain from initiating an activity when they know or should know there is a substantial likelihood that their personal problems will prevent them from performing their work-related activities in a competent manner.” Personal problems include emotional, social, health-related and other personal issues.
b. Incorrect Although the psychologist should “take appropriate measures” if she believes her condition may interfere with the performance of her work-related duties, supervision may not be the best course of action.
c. Incorrect It is not necessary for the psychologist to inform her clients of her condition and, in some cases, may not be in the best interests of the clients to do so.
d. Incorrect See explanation for response a.

The correct answer is: should refrain from initiating any professional activities that might be adversely affected by her medical condition.

How well did you know this?
1
Not at all
2
3
4
5
Perfectly
61
Q

The brother of a former client of yours calls you requesting that you release the client’s therapy records to him. The client died several months prior to this request. As an ethical psychologist, you:
Select one:

A. release the records to him since he is a family member.
B. release the records to him since requirements for maintaining confidentiality do not extend beyond a client’s death.
C. release the records only after obtaining a waiver from the executor or administrator of the client’s estate.
D. refuse to release the records under any circumstance.

A

Although this situation is not explicity addressed by the Ethics Code, Standard 4.05(a) states that “psychologists may disclose information with the appropriate consent of the organizational client, the individual client/patient, or another legally authorized person on behalf of the client/patient unless prohibited by law. (Also, keep in mind that, on the exam, it’s always best to choose the most “conservative” response - i.e., the response that best protects the client’s welfare.)

a. Incorrect See explanation for response c.
b. Incorrect See explanation for response c.
c. CORRECT This response is most consistent with ethical requirements as well as with state laws. For example, a Massachusetts statute relevant to the practice of psychology states that “all communications … shall be deemed to be treated as confidential in perpetuity” (251 CMR 1.11). And, in jurisdictions that have similar laws, the records of a deceased client cannot (usually) be released to a family member or other person without a court order or release from the executor or administrator of the client’s estate.
d. Incorrect See explanation for response c.

The correct answer is: release the records only after obtaining a waiver from the executor or administrator of the client’s estate.

How well did you know this?
1
Not at all
2
3
4
5
Perfectly
62
Q

The Ethics Code requires that a psychologist discuss the issue of fees and billing policies with a new client:
Select one:

A. prior to the first consultation.
B. during the first session.
C. when the psychologist deems it appropriate.
D. as early as is feasible.

A

This issue is addressed in Standard 6.04(a): Fees and Financial Arrangements.

a. Incorrect See explanation for response d.
b. Incorrect See explanation for response d.
c. Incorrect See explanation for response d.
d. CORRECT “As early as is feasible” is the language used in the Ethics Code and, therefore, is the best answer to this question.

The correct answer is: as early as is feasible.

How well did you know this?
1
Not at all
2
3
4
5
Perfectly
63
Q

The best predictor of whether a client will become sexually involved with a therapist is:
Select one:

A. the client’s level of physical attractiveness.
B. the client’s history of sexual abuse.
C. the therapist’s professional experience and status.
D. the therapist’s past history with regard to sexual involvement with a client.

A

Once again, past behavior is the best predictor of future behavior which narrows your choices down to responses b and d.

a. Incorrect The client’s physical attractiveness does not appear to be a factor in sexual misconduct among therapists.
b. Incorrect Although certain client behaviors and background factors may place them at higher risk for sexual involvement with a therapist, client characteristics are not as good as the therapist’s history at predicting whether or not the client will become sexually involved with his/her therapist.
c. Incorrect The therapist’s experience and status have not been found to correlate highly with sexual misconduct.
d. CORRECT The research shows that recidivism rates for sexual misconduct among therapists are quite high, leading to the conclusion that a therapist’s history with regard to sexual involvement with clients is the best predictor of future involvement.

The correct answer is: the therapist’s past history with regard to sexual involvement with a client.

How well did you know this?
1
Not at all
2
3
4
5
Perfectly
64
Q

According to the APA’s General Guidelines for Providers of Psychological Services, a “user” of clinical and counseling psychological services includes:
Select one:

A. direct users of services only (“patients” and “clients”).
B. direct users and third party purchasers of such services.
C. direct users, third party purchasers, and sanctioners (people who have legitimate concern related to the provision of psychological services).
D. direct users, third party purchasers, sanctioners, and public/private facilities and organizations receiving psychological services.

A

According to the General Guidelines, a “user” of clinical and counseling psychological services includes direct users, third party purchasers, sanctioners, and public/private facilities and organizations receiving psychological services.

a. Incorrect The users of psychological services are not limited to direct users only.
b. Incorrect The users of psychological services are not limited to direct users and third party purchasers only.
c. Incorrect The users of psychological services are not limited to direct users, third party purchasers, and sanctioners only.
d. CORRECT All of these are defined as users of psychological services in the General Guidelines.

The correct answer is: direct users, third party purchasers, sanctioners, and public/private facilities and organizations receiving psychological services.

How well did you know this?
1
Not at all
2
3
4
5
Perfectly
65
Q

The APA’s (2007)Record Keeping Guidelines recommends that, in the absence of superseding laws or regulations, psychologists should retain:
Select one:

A. the full record of an adult client until seven years after termination of therapy.
B. the full record of an adult client until 12 years after termination of therapy.
C. the full record of an adult client until three years after termination and a summary of the record for at least four additional years.
D. the full record of an adult client until five years after termination and a summary of the record for at least seven additional years.

A

Paragraph 7 of the 2007 version of the APA’s Record Keeping Guidelines specifies that a “psychologist strives to be aware of applicable laws and regulations and to retain records for the period required by legal, regulatory, institutional, and ethical requirements.”

a. CORRECT In comments on the application of Paragraph 7, the Guidelines states that, in the absence of superseding laws or institutional regulations, “psychologists may consider retaining full records until 7 years after the last date of service delivery for adults or until 3 years after a minor reaches the age of majority, whichever is later. In some circumstances, the psychologist may wish to keep records for a longer period, weighing the risks associated with obsolete or outdated information, or privacy loss, versus potential benefits associated with preserving the records.” This answer is most consistent with these recommendations.
b. Incorrect See explanation for response a.
c. Incorrect See explanation for response a.
d. Incorrect See explanation for response a.

The correct answer is: the full record of an adult client until seven years after termination of therapy.

How well did you know this?
1
Not at all
2
3
4
5
Perfectly
66
Q

A client reveals to you that he and a friend robbed a convenience store six months ago. He says he knows that what he did was wrong and will never do it again. He also says that he notices that you “make notes” while he is talking, and he asks that you not keep any written record of the robbery. You should:
Select one:

A. agree to exclude information about the robbery from his file.
B. determine whether the robbery is clinically relevant before deciding whether or not to include information about it in his file.
C. inform the client that you are legally required to report the matter to the authorities before you actually do so.
D. reassure the client that anything he tells you is confidential and will not be revealed without his consent.

A

Psychologists are generally not required to report criminal conduct unless it is conduct explicitly addressed by the law (e.g., child abuse, danger to others).

a. Incorrect Omitting client information from the record at the client’s request is not necessarily in the best interests of the client or the therapist.
b. CORRECT This is the best response of those given. A similar situation is discussed by R. Woody, who notes that client records are for the benefit of both the client and the therapist and that, if the client “communicates information that is relevant to the services provided, it should be entered into the record” (Legally safe mental health practice: Psycholegal questions and answers, Madison, CT, Psychosocial Press, 1997, p. 70).
c. Incorrect See explanation above.
d. Incorrect It is important to keep in mind that it is impossible to guarantee that any client information will never be revealed under any circumstance.

The correct answer is: determine whether the robbery is clinically relevant before deciding whether or not to include information about it in his file.

How well did you know this?
1
Not at all
2
3
4
5
Perfectly
67
Q

When conducting group therapy, you should remind group members:
Select one:

A. that they are legally bound to maintain confidentiality.
B. that, since they are not legally bound to maintain confidentiality, they should not expect other members to do so.
C. that, as members of a therapy group, they are ethically required to maintain confidentiality.
D. of the importance of maintaining the confidentiality of statements made during group sessions.

A

Confidentiality is an important issue in therapy but poses special problems in multi-client situations.

a. Incorrect Legally, privilege is waived in many states if a third party (e.g., other group member) is present at the time the information is revealed. Also, other group members are not governed by privilege laws. Consequently, there is no legal requirement for confidentiality by group members.
b. Incorrect This would NOT be a particularly useful approach in helping to ensure that clients do maintain the confidentiality of other group members.
c. Incorrect Group members are not bound by professional codes regarding confidentiality.
d. CORRECT Of the responses given, this is the best approach and is most consistent with Standard 10.03 of the Ethics Code.

The correct answer is: of the importance of maintaining the confidentiality of statements made during group sessions.

How well did you know this?
1
Not at all
2
3
4
5
Perfectly
68
Q

With regard to informed consents in research, APA’sEthics Code:
Select one:

A. requires psychologists to provide potential research participants with specific information in “reasonably understandable language” prior to consenting to participate.
B. requires psychologists to “ensure that potential research participants understand” the information they have been given prior to consenting to participate.
C. requires psychologists to provide potential research participants with specific information prior to consenting to participate.
D. requires psychologists to provide potential research participants with specific information in “reasonably understandable language” and to ensure that they understand the information prior to consenting to participate.

A

With certain exceptions, psychologists must obtain informed consents from individuals prior to their participation in a research study.

a. CORRECT The issue of language and informed consent is addressed in Standard 3.10 of the Ethics Code. It outlines the specific information that psychologists must give potential participants, therapy clients, and others as part of the informed consent process and states that the information must be supplied in “reasonably understandable language.” The Code does not, however, explictly state that psychologists must ensure that participants understand the information.
b. Incorrect See explanation for response a.
c. Incorrect See explanation for response a.
d. Incorrect See explanation for response a.

The correct answer is: requires psychologists to provide potential research participants with specific information in “reasonably understandable language” prior to consenting to participate.

How well did you know this?
1
Not at all
2
3
4
5
Perfectly
69
Q

Dr. Goodman has been dating Billy Bob for seven weeks when she realizes that he is the brother of a client she has been seeing in therapy for nearly three months. Dr. Goodman’s best course of action would be to:
Select one:

A. refer the client to another therapist immediately.
B. stop seeing Billy Bob immediately.
C. discuss the matter with the client as soon as possible.
D. do nothing until it becomes evident that the situation is creating a conflict.

A

The APA’s Ethics Code warns against becoming involved in multiple relationships. However, it does not directly address this particular situation.

a. Incorrect See explanation for response c.
b. Incorrect See explanation for response c.
c. CORRECT This response is most consistent with the spirit of Standard 3.05(b) of the Ethics Code which states, “If a psychologist finds that, due to unforeseen factors, a potentially harmful multiple relationship has arisen, the psychologist takes reasonable steps to resolve it with due regard for the best interests of the affected person and maximal compliance with the Ethics Code.” It may be necessary, eventually, to refer the client to another therapist or end the relationship with Billy Bob, but neither of these would be the best initial action.
d. Incorrect See explanation for response c.

The correct answer is: discuss the matter with the client as soon as possible.

How well did you know this?
1
Not at all
2
3
4
5
Perfectly
70
Q

You assessed a defendant who is accused of rape using a single standardized personality inventory. The defendant’s profile does not seem to be consistent with the nature of the crime. You should:
Select one:

A. testify as an expert witness that you believe the defendant is not guilty.
B. testify as a character witness.
C. should not disclose the results of this personality inventory.
D. should not base a conclusion on the results of a single assessment profile.

A

According to Standard 9.01(a) of the APA’s Ethics Code, “psychologists base the opinions contained in their recommendations, reports, and diagnostic or evaluative statements, including forensic testimony, on information and techniques sufficient to substantiate their findings.” Consequently, you should not draw a conclusion in this situation based solely on the results of a single assessment test.

a. Incorrect See explanation above.
b. Incorrect See explanation above.
c. Incorrect See explanation above.
d. CORRECT See explanation above.

The correct answer is: should not base a conclusion on the results of a single assessment profile.

How well did you know this?
1
Not at all
2
3
4
5
Perfectly
71
Q

According to the APA’s Guidelines for Psychological Services to Ethnic, Linguistic, and Culturally Diverse Populations, when a client belonging to a different ethnic or cultural group comes to therapy, a therapist is best advised to:
Select one:

A. treat the client like any other person unless there is reason to do otherwise.
B. alter the diagnostic and treatment processes to coincide with the client’s ethnic or cultural background.
C. recognize that ethnic and cultural forces can have an impact on both the therapist’s and the client’s psychological processes.
D. refer the client to another therapist from the same ethnic or cultural background whenever it is feasible to do so.

A

The Guidelines for Psychological Services to Ethnic, Linguistic, and Culturally Diverse Populations addresses several issues related to assessment, intervention, and research with clients from different ethnic and cultural backgrounds.

a. Incorrect Although there are times when this would be the appropriate course of action (e.g., when the client exhibits a high degree of acculturation into the “mainstream” culture), it is not a recommended strategy.
b. Incorrect This may be appropriate in some situations, but not all of them.
c. CORRECT Paragraph 3a states that “Psychologists recognize ethnicity and culture as significant parameters underlying psychological processes,” and this refers not only to the processes of the client but also to those of the therapist (e.g., therapists must be aware of their own prejudices and stereotypes when working with clients from different ethnic and cultural groups).
d. Incorrect This may be appropriate in some situations but is not a general requirement.

The correct answer is: recognize that ethnic and cultural forces can have an impact on both the therapist’s and the client’s psychological processes.

How well did you know this?
1
Not at all
2
3
4
5
Perfectly
72
Q

Responsibility for the validity of information provided by an automated test scoring and interpretation service lies with:
Select one:

A. the test scoring and interpretation service.
B. the psychologist who is in charge of the test scoring and interpretation service.
C. the psychologist who uses the test scoring and interpretation service.
D. the test scoring and interpretation service and the psychologist who uses the service.

A

This issue is addressed in Standard 9.09(c) of the Ethics Code.

a. Incorrect See explanation for response c.
b. Incorrect See explanation for response c.
c. CORRECT According to the Code, “psychologists retain responsibility for the appropriate application, interpretation, and use of assessment instruments, whether they score and interpret such tests themselves or use automated or other services.”
d. Incorrect See explanation for response c.

The correct answer is: the psychologist who uses the test scoring and interpretation service.

How well did you know this?
1
Not at all
2
3
4
5
Perfectly
73
Q

A new client of yours, who is homosexual, tells you that he overheard his previous therapist make derogatory remarks about his sexual orientation to a co-worker. Your best course of action would be to:
Select one:

A. discuss his option of filing a complaint against the psychologist with the Ethics Committee.
B. file a complaint against the psychologist with the Ethics Committee yourself.
C. call the psychologist and tell him that your client overheard his remarks.
D. provide the client with support and remind him that there are many prejudiced people in the world.

A

This is a difficult question that requires a careful reading of the responses.

a. CORRECT This is usually an acceptable course of action when a client reveals that a previous therapist has acted unethically. Note that responses b and c can be eliminated because they do not address the issue of the client’s confidentiality: Before talking to the psychologist yourself or contacting the Ethics Committee, you would want to get a waiver from the client.
b. Incorrect See explanation for response a.
c. Incorrect See explanation for response a.
d. Incorrect See explanation for response a.

The correct answer is: discuss his option of filing a complaint against the psychologist with the Ethics Committee.

How well did you know this?
1
Not at all
2
3
4
5
Perfectly
74
Q

You are starting a new program for children of recently divorced parents. You tell the editor of the local newspaper (who has just divorced her husband) that her two children can attend the program for free if she puts an article about the program in the paper. According to the Ethics Code, this is:
Select one:

A. unethical because it constitutes coercion.
B. unethical because psychologists are prohibited from compensating a newspaper employee for publicity in a news item.
C. unethical only if you influence the content of the article.
D. ethical as long as the arrangement doesn’t compromise your objectivity when working with the editor’s children.

A

This issue is addressed in Standard 5.02(b) of the APA’s Ethics Code.

a. Incorrect See explanation for response b.
b. CORRECT The Code prohibits psychologists from compensating employees of the press for providing publicity about the psychologist or his/her work in a news item.
c. Incorrect See explanation for response b.
d. Incorrect See explanation for response b.

The correct answer is: unethical because psychologists are prohibited from compensating a newspaper employee for publicity in a news item.

How well did you know this?
1
Not at all
2
3
4
5
Perfectly
75
Q

Providing therapy to a former sex partner:
Select one:

A. is not directly addressed by the Ethics Code but is covered by the prohibition against multiple relationships.
B. is permissible only when it is not clinically contraindicated and the chance of exploitation is minimal.
C. is permissible under “unusual circumstances” when at least two years have passed since the sexual relationship ended.
D. is explicitly prohibited by the Code.

A

Standard 10.07 of the Ethics Code addresses the issue of therapy with former sexual partners.

a. Incorrect See explanation for response d.
b. Incorrect See explanation for response d.
c. Incorrect If you picked this response, you’ve mixed up the situation asked about in this question with becoming involved sexually with a former client.
d. CORRECT According to the Code, “Psychologists do not accept as therapy clients/patients persons with whom they have engaged in sexual intimacies.” There are no exceptions to this prohibition.

The correct answer is: is explicitly prohibited by the Code.

How well did you know this?
1
Not at all
2
3
4
5
Perfectly
76
Q

You are meeting with the parents of a teenage daughter who is suffering from an eating disorder. All of their children have attended this first session. You should:
Select one:

A. obtain a release of information from all family members.
B. clarify at the outset your relationship with each family member.
C. refer the daughter who is the identified patient for individual therapy.
D. include only the parents and identified patient in family therapy

A

According to Standard 10.02(a) of the APA’s Ethics Code, “When psychologists agree to provide services to several persons who have a relationship (such as spouses, significant others, or parents and children), they take reasonable steps to clarify at the outset (1) which of the individuals are clients/patients and (2) the relationship the psychologist will have with each person. This clarification includes the psychologist’s role and the probable uses of the services provided or the information obtained.”

a. Incorrect See explanation above.
b. CORRECT See explanation above.
c. Incorrect See explanation above.
d. Incorrect See explanation above.

The correct answer is: clarify at the outset your relationship with each family member.

How well did you know this?
1
Not at all
2
3
4
5
Perfectly
77
Q

A psychologist acts as both a consultant to the court and a fact witness for the defense in a criminal case. This:
Select one:

A. represents a clear conflict of interest and is prohibited by APA guidelines.
B. represents a “multiple relationship” and is explicitly prohibited by APA guidelines.
C. may be acceptable as long as the psychologist clarifies her roles with the court and the defense.
D. may be acceptable as long as the psychologist gets written permission from the court to act as a fact witness.

A

Dual (multiple) relationships in forensic settings are addressed in the Ethics Code and the Specialty Guidelines for Forensic Psychology.

a. Incorrect Although a conflict of interest should always be avoided, this particular situation does not necessarily constitute a conflict of interest.
b. Incorrect This situation may be described as a “multiple relationship,” but it is not one that is explicity prohibited by APA guidelines.
c. CORRECT Accepting dual roles (e.g., acting as a consultant for one party and a fact witness for another) may be acceptable as long as the psychologist clarifies his/her roles with both parties and acts in a way that does not compromise his/her judgment and objectivity.
d. Incorrect See explanation for response c.

The correct answer is: may be acceptable as long as the psychologist clarifies her roles with the court and the defense.

How well did you know this?
1
Not at all
2
3
4
5
Perfectly
78
Q

Dr. Passe is hired by a company to evaluate an employee for promotion into a high-level position. The company manager tells Dr. Passe that the employee was evaluated six years ago prior to his last promotion, and the company wants Dr. Passe to make a recommendation based on those results without an additional evaluation. Dr. Passe is concerned that some of the information from the evaluation is no longer relevant. He should:
Select one:

A. comply with the manager’s request since it is up to her (the manager) to decide what is relevant.
B. comply with the manager’s request but warn her of any potential limitations of her recommendation.
C. comply with the manager’s request only if the employee agrees to the conditions.
D. discuss with the manager the possibility that the data are obsolete and that new data should be collected.

A

This question is pretty straightforward. It is unethical to base decisions on obsolete data.

a. Incorrect See explanation for response d.
b. Incorrect See explanation for response d.
c. Incorrect See explanation for response d.
d. CORRECT Psychologists have a responsibility to ensure that their recommendations are accurate, which would not be the case if they are based on obsolete data. Providing information about the possible limitations (response b) of a recommendation would be insufficient.

The correct answer is: discuss with the manager the possibility that the data are obsolete and that new data should be collected.

How well did you know this?
1
Not at all
2
3
4
5
Perfectly
79
Q

Dr. Looby receives a letter from the Ethics Committee stating that the Committee has received a complaint from a current client who is concerned about Dr. Looby’s office staff and procedures. The client has found two members of the staff to be consistently rude, and they have made comments to her that imply they have access to confidential information. Dr. Looby should:
Select one:

A. discuss the matter with the staff members.
B. set up an appointment with the client to discuss the matter.
C. send a letter of apology to the client and inform her that changes will be made.
D. respond to the Committee in writing.

A

This question is difficult to answer because none of the responses describes an entirely inappropriate action. However, only one response addresses APA requirements.

a. Incorrect See explanation for response d.
b. Incorrect See explanation for response d.
c. Incorrect See explanation for response d.
d. CORRECT This response addresses the requirements of APA’s Ethics Code and Rules and Procedures - i.e., both documents require psychologists to cooperate with the Ethics Committee. Specifically, Part II, Standard 8.2 of the Rules and Procedures requires psychologists to respond to charges of unethical conduct personally and Standard 9 requires full cooperation “in a timely fashion.” How Dr. Looby responds to the client’s charges - e.g., by talking to the client, discussing the matter with the staff - depends on the situation.

The correct answer is: respond to the Committee in writing.

How well did you know this?
1
Not at all
2
3
4
5
Perfectly
80
Q

A 13-year old client tells his therapist that he wants to kill himself and that he has access to his father’s gun collection. The therapist believes the boy’s threat is a serious one and tells him that she will have to contact his parents. The boy gets very angry and says he thought they had agreed that everything he said in therapy was confidential. If the therapist contacts the boy’s parents, she has:
Select one:

A. acted ethically only if the limits of confidentiality were discussed with the boy at the beginning of therapy.
B. acted ethically because a breach of confidentiality is justified in this situation.
C. acted unethically if confidentiality was, in fact, guaranteed to the boy at the outset of therapy.
D. acted unethically if she contacts the boy’s parents without his consent.

A

In this situation, a breach of confidentiality is both ethical and legal.

a. Incorrect See explanation for response b.
b. CORRECT A psychologist must take appropriate action whenever a client is a danger to him/herself. Contacting the boy’s parents would be an appropriate action.
c. Incorrect See explanation for response b.
d. Incorrect See explanation for response b.

The correct answer is: acted ethically because a breach of confidentiality is justified in this situation.

How well did you know this?
1
Not at all
2
3
4
5
Perfectly
81
Q

During his first therapy session, an Asian American client tells his non-Asian therapist that he would prefer to see an Asian psychologist. The therapist’s best course of action would be to:
Select one:

A. discuss the issue with the client and make a referral if, after the discussion, the client says he still wants to see an Asian therapist.
B. tell the client that she’s had experience working with members of minority groups and suggest that they see each other for at least two or three more sessions.
C. realize that the client’s preference is a manifestation of resistance and tell him that she thinks it would be best if they saw each other for a few sessions before a referral is made.
D. tell the client that it would be discriminatory, and therefore unethical, for her to make a referral to another therapist on the basis of race alone.

A

This question is fairly simple: It is never ethical to coerce a client into remaining in therapy.

a. CORRECT This is the best course of action in this type of situation. The client will not benefit from treatment if he is not comfortable with the therapist, and it would be unethical to try to “talk him into” additional sessions.
b. Incorrect See explanation for response a.
c. Incorrect See explanation for response a.
d. Incorrect See explanation for response a.

The correct answer is: discuss the issue with the client and make a referral if, after the discussion, the client says he still wants to see an Asian therapist.

How well did you know this?
1
Not at all
2
3
4
5
Perfectly
82
Q

The police arrive at your office with an arrest warrant for one of your clients. They tell you that they need the client’s most recent address and telephone number. You should:
Select one:

A. provide them with the information they request.
B. make a copy of the warrant for your files before providing them with the information they request.
C. tell them you will have to get a signed waiver from the client before you can release the information.
D. not give them any information about the client.

A

An arrest warrant does not constitute an exception to the therapist-client privilege.

a. Incorrect See explanation for response d.
b. Incorrect See explanation for response d.
c. Incorrect See explanation for response d.
d. CORRECT In order to provide any information about the client (including the fact that he/she is a client) would require either a signed release from the client or a court-order.

The correct answer is: not give them any information about the client.

How well did you know this?
1
Not at all
2
3
4
5
Perfectly
83
Q

At the end of her first therapy session, a middle-aged woman tells you she is presently receiving treatment from a psychiatrist. You, as an ethical psychologist, should:
Select one:

A. inform the woman that it would be unethical to continue seeing her while she is seeing another professional.
B. call the psychiatrist to inform him of the situation and obtain his permission to continue therapy with her.
C. discuss the issues related to the woman’s relationship with the psychiatrist during your next session with her.
D. allow the woman to decide when to terminate treatment with the psychiatrist.

A

Standard 10.04 of the Ethics Code states, “In deciding whether to offer or provide services to those already receiving mental health services elsewhere, psychologists carefully consider the treatment issues and the potential client’s/patient’s welfare. Psychologists discuss these issues with the client/patient or another legally authorized person on behalf of the client/patient in order to minimize the risk of confusion and conflict, consult with the other service providers when appropriate, and proceed with caution and sensitivity to the therapeutic issues.”

a. Incorrect Accepting the woman for treatment in this situation is not prohibited by the Ethics Code.
b. Incorrect Obtaining the permission of the psychiatrist to see the woman is not required by the Ethics Code and would violate the client’s confidentiality if it is done without her consent.
c. CORRECT This response is most consistent with the requirements of Standard 10.04.
d. Incorrect This is less consistent than answer c with Standard 10.04.

The correct answer is: discuss the issues related to the woman’s relationship with the psychiatrist during your next session with her.

How well did you know this?
1
Not at all
2
3
4
5
Perfectly
84
Q

Which of the following statements is most consistent with the requirements of the APA’s Ethics Code with regard to the use of deception in research?
Select one:

A. Psychologists may not deceive participants about possible risks that may affect their willingness to participate.
B. Psychologists are not required to disclose any possible risks if they have taken all possible steps to reduce those risks.
C. Psychologists must provide an explanation of the use of any deception in an experiment immediately after participation.
D. Psychologists may not use deception in research.

A

Answer A is correct. Standard 8.07 (Deception in Research) states that “

(a) Psychologists do not conduct a study involving deception unless they have determined that the use of deceptive techniques is justified by the study’s significant prospective scientific, educational, or applied value and that effective nondeceptive alternative procedures are not feasible.
(b) Psychologists do not deceive prospective participants about research that is reasonably expected to cause physical pain or severe emotional distress.
(c) Psychologists explain any deception that is an integral feature of the design and conduct of an experiment to participants as early as is feasible, preferably at the conclusion of their participation, but no later than at the conclusion of the data collection, and permits participants to withdraw their data.”

The correct answer is: Psychologists may not deceive participants about possible risks that may affect their willingness to participate.

How well did you know this?
1
Not at all
2
3
4
5
Perfectly
85
Q

“Vicarious liability” is most likely to be of concern when a psychologist is acting in the role of:
Select one:

A. collaborator.
B. consultant.
C. advocate.
D. supervisor.

A

The doctrine of vicarious liability (“respondeat superior”) applies to settings in which a psychologist has ethical and legal responsibility for the services provided by another person.

a. Incorrect See explanation for response d.
b. Incorrect See explanation for response d.
c. Incorrect See explanation for response d.
d. CORRECT Ethical and legal responsibility is most likely to be an issue in a supervisory relationship in which there is a large discrepancy between the skills, knowledge, etc. of the supervisor and those of the supervisee. In this situation, the supervisor is in a position of authority or control. (Vicarious liability can also be an issue in employer-employee relationships.)

The correct answer is: supervisor.

How well did you know this?
1
Not at all
2
3
4
5
Perfectly
86
Q

Nadler’s (1988) systems model of planned change proposes that, to successfully implement change, managers must attend to four factors. One of these is informal organizational elements which consist of:
Select one:

A. external (environmental) driving and restraining forces.
B. the employees’ knowledge, skills, and expectations.
C. the organization’s implicit beliefs, values, and behaviors.
D. the organization’s structures, processes, and methods.

A

The four factors identified by Nadler (1988) are informal organizational elements, formal organizational elements, individuals (employees and managers), and tasks.

a. Incorrect See explanation for response c.
b. Incorrect See explanation for response c.
c. CORRECT The informal organizational elements include implicit beliefs, values, and behaviors (e.g., communication patterns, leadership approach, and work-related norms).
d. Incorrect See explanation for response c.

The correct answer is: the organization’s implicit beliefs, values, and behaviors.

How well did you know this?
1
Not at all
2
3
4
5
Perfectly
87
Q

Hollander’s (1960) concept of “idiosyncrasy credits” is useful for understanding why:
Select one:

A. a person exerts less effort when working as a group member than when working alone.
B. a person acts in a way that is opposite of what is requested or expected.
C. a group member is allowed to deviate from group norms without receiving disapproval from other group members.
D. a group member adopts the group decision even when he/she disagrees with it.

A

Even if you’re unfamiliar with Hollander’s (1960) concept, the word “idiosyncrasy” may have helped you identify the correct answer to this question.

a. Incorrect This answer describes “social loafing.”
b. Incorrect This answer describes “psychological reactance.”
c. CORRECT According to Hollander, group members are allowed to deviate from group norms without negative consequences in certain circumstances - for example, when the member has a history of conformity or is serving as the leader of the group.
d. Incorrect This answer is not relevant to idiosyncrasy credits.

The correct answer is: a group member is allowed to deviate from group norms without receiving disapproval from other group members.

How well did you know this?
1
Not at all
2
3
4
5
Perfectly
88
Q

Studies investigating the relationship between job performance and job satisfaction have most consistently found that:
Select one:

A. correlations between job satisfaction and performance are positive, and coefficients are usually moderate to high.
B. correlations between satisfaction and performance are negative, although negative coefficients are usually low.
C. correlations between job satisfaction and performance are positive, although coefficients are usually low.
D. correlations between job satisfaction and performance are negative, and negative coefficients are usually quite high.

A

Contrary to what might be expected, a strong relationship between job satisfaction and performance has not been supported by the research.

a. Incorrect While correlations between job satisfaction and performance are usually positive, coefficients are not usually large.
b. Incorrect Correlations between job satisfaction and performance are usually positive, not negative.
c. CORRECT Correlations between job satisfaction and performance are usually positive but small. Vroom (1964), for example, reported an average correlation coefficient of .14.
d. Incorrect Correlations between job satisfaction and performance are usually positive, not negative.

The correct answer is: correlations between job satisfaction and performance are positive, although coefficients are usually low.

How well did you know this?
1
Not at all
2
3
4
5
Perfectly
89
Q

The Taylor-Russell tables will indicate that a new selection test is most likely to be helpful in choosing job applicants who will be satisfactory employees when:
Select one:

A. the proportion of current employees who are considered satisfactory is less than 25%.
B. there are only a few job openings and many applicants for the job.
C. the majority of applicants have jobs skills in the moderate (average) range.
D. the test has adequate “differential validity.”

A

Use of the Taylor-Russell tables estimates the percent of employees who will be satisfactory if a new selection technique is used, and requires three pieces of information - the test’s validity coefficient, the selection ratio, and the base rate.

a. Incorrect The proportion of current employees who are considered successful is the “base rate.” A new selection test will be most useful when the base rate is moderate (around 50%).
b. CORRECT All other things being equal, a new selection test will be most useful for selecting job applicants when the selection ratio is low (i.e., there are few jobs and many applicants).
c. Incorrect This is not a factor that is considered when using the Taylor-Russell tables to evaluate the usefulness of a selection test.
d. Incorrect This is not a factor that is considered when using the Taylor-Russell tables.

The correct answer is: there are only a few job openings and many applicants for the job.

How well did you know this?
1
Not at all
2
3
4
5
Perfectly
90
Q

Overtraining is most associated with:
Select one:

A. peak performance.
B. enhanced transfer of training.
C. a drug-like “high.”
D. decreased motivation and performance.

A

Note that this question is asking about overtraining, not overlearning. The term overtraining appears in the sports psychology literature; overlearning is used in the learning and training literature.

a. Incorrect This would be the best answer if the question were asking about overlearning.
b. Incorrect This also better fits overlearning.
c. Incorrect This is not associated with overtraining or overlearning.
d. CORRECT In the sports psychology literature, overtraining (staleness) refers to excessive training and the resulting physical and psychological symptoms. The undesirable effects of overtraining include premature fatigue during exercise, reduced performance, emotional instability, decreased motivation, and increased susceptibility to infection.

The correct answer is: decreased motivation and performance.

How well did you know this?
1
Not at all
2
3
4
5
Perfectly
91
Q

A colleague of yours, an industrial psychologist, designs a study to investigate the relationship between job satisfaction and mental and physical health. Based on your knowledge of the research in this area, you tell him that:
Select one:

A. there is no consistent relationship between job satisfaction and mental or physical health.
B. there is a positive relationship between job satisfaction and mental and physical health.
C. there is a positive relationship between job satisfaction and mental health but no relationship between job satisfaction and physical health.
D. there is a positive relationship between job satisfaction and physical health but no relationship between job satisfaction and mental health.

A

Even if you are unfamiliar with the research in this area, it seems logical that job dissatisfaction might be either a cause or effect of poor physical and/or mental health.

a. Incorrect See explanation for response b.
b. CORRECT Several studies have found a positive relationship between job satisfaction and both mental and physical health. Note, however, that the research has not demonstrated that job satisfaction causes mental/physical health or vice-versa, only that they are correlated.
c. Incorrect See explanation for response b.
d. Incorrect See explanation for response b.

The correct answer is: there is a positive relationship between job satisfaction and mental and physical health.

92
Q

In the context of training, “identical elements” refers to the match between:
Select one:

A. the expectations of trainees about the potential benefits of a training program and its actual benefits.
B. the information and skills addressed in the training program and the information and skills required by the job.
C. the goals of the training program and the personal goals of the participants in the program.
D. the goals of the training program and the program outcomes desired by management.

A

The concept of “identical elements” was introduced by Thorndike and Woodworth (1901).

a. Incorrect See explanation for response b.
b. CORRECT The principle of identical elements proposes that the more similar the training and performance situations are with regard to the presented stimuli and required responses, the greater the transfer of training. Similarity includes both physical similarity (e.g., the similarity of the tasks and environment) and psychological similarity (i.e., the degree to which trainees attach similar meanings to the training and performance situations).
c. Incorrect See explanation for response b.
d. Incorrect See explanation for response b.

The correct answer is: the information and skills addressed in the training program and the information and skills required by the job.

93
Q

According to Maslow’s need hierarchy theory, once a person’s physiological needs have been satisfied, that person will be motivated by his/her __________ needs.
Select one:

A. social
B. safety
C. existence
D. esteem

A

Maslow’s need hierarchy theory predicts that humans have five basic needs that emerge in a hierarchical order.

a. Incorrect See explanation for response b.
b. CORRECT The five needs, in order, are: physiological, safety, social, esteem, and self-actualization. Need hierarchy theory predicts that physiological needs are the first needs to act as motivators; once they are satisfied, safety needs become the source of motivation; and so on.
c. Incorrect Existence is not one of the needs identified by Maslow.
d. Incorrect Esteem needs are not motivators until physiological, safety, and social needs have been satisfied.

The correct answer is: safety

94
Q

A psychologist is hired to develop a selection test that will be used to assist in the hiring of management trainees. The psychologist’s first step in developing the selection test will be conducting a:
Select one:

A. needs assessment.
B. job analysis.
C. job evaluation.
D. formative evaluation.

A

For the exam, you want to be familiar with all of the procedures listed in the answers to this question. Additional information about them is provided in the Industrial-Organizational Psychology chapter of the written study materials.

a. Incorrect A needs assessment (needs analysis) is a first step in designing training programs.
b. CORRECT To develop a test that will be used to assist in the selection of job applicants, it would be necessary to know the duties and skills needed to successfully perform the job. Therefore, the first step in developing the test will be a job analysis.
c. Incorrect A job evaluation is conducted specifically to determine the relative worth of jobs in order to set salaries and wages.
d. Incorrect A formative evaluation is conducted while a training program is being developed to determine if any modifications to the program are required.

The correct answer is: job analysis.

95
Q

When using a Behaviorally-Anchored Rating Scale (BARS), scale points are “anchored” with:
Select one:

A. empirically-derived weights.
B. job dimensions.
C. criterion outcomes.
D. critical incidents.

A

In a Behaviorally-Anchored Rating Scale (BARS), the points on the scale are “anchored” with descriptions of specific job behaviors.

a. Incorrect See explanation for response d.
b. Incorrect See explanation for response d.
c. Incorrect See explanation for response d.
d. CORRECT Another name for behavioral anchors is critical incidents.

The correct answer is: critical incidents.

96
Q

A college student receives a high score on the realistic scale of Holland’s Self-Directed Search but a low score on all other scales. This is an example of which of the following?
Select one:

A. congruence
B. vocational identity
C. differentiation
D. individuation

A

Holland’s theory of career decision-making incorporates several constructs including congruence, identity, and differentiation.

a. Incorrect As defined by Holland, congruence refers to the degree of consistency between an individual’s expressed and assessed interests.
b. Incorrect Vocational identity refers to the clarity and stability of the individual’s goals and self-perceptions.
c. CORRECT Differentiation refers to the distinctiveness of the individual’s profile on the Self-Directed Search. An individual’s profile is highly differentiated when he/she obtains a high score on one scale and low scores on all other scales.
d. Incorrect Individuation is not one of Holland’s constructs.

The correct answer is: differentiation

97
Q

According to Dawis, England, and Lofquist’s (1964) theory of work adjustment, the probability that a person will be forced out of the work environment is inversely related to his/her:
Select one:

A. satisfaction.
B. satisfactoriness.
C. job commitment.
D. organizational commitment.

A

The theory of work adjustment predicts that job tenure is a joint function of satisfaction and satisfactoriness. Satisfaction refers to the correspondence between an employee’s vocational needs and values and the reinforcement systems provided by the job, while satisfactoriness is a function of the correspondence between an employee’s abilities and the ability requirements of the job.

a. Incorrect According to this theory, the probability that a person will voluntarily leave the work environment is inversely related to his/her satisfaction.
b. CORRECT Knowing that satisfactoriness is the extent to which an employee is able to perform a job would have helped you identify this as the correct answer. When an employee cannot perform the job because his/her abilities do not meet ability requirements of the job, the employee is likely to be fired.
c. Incorrect See explanation above.
d. Incorrect See explanation above.

The correct answer is: satisfactoriness.

98
Q

According to Herzberg’s “two-factor theory,” a disgruntled blue-collar worker is most likely to say he is satisfied with his job if he is:
Select one:

A. given a raise.
B. given fewer tasks to do.
C. put on a piece-rate pay system.
D. given more responsibility.

A

Herzberg’s two-factor theory divides job components into two types: hygiene factors and motivator factors.

a. Incorrect For Herzberg, money is a hygiene factor and, consequently, can produce dissatisfaction when it is perceived to be inadequate but does not contribute to satisfaction or motivation, even when it is perceived to be adequate.
b. Incorrect From Herzberg’s perspective, this would not lead to greater satisfaction.
c. Incorrect This is not consistent with two-factor theory.
d. CORRECT According to two-factor theory, satisfaction and motivation are increased by providing an employee with motivator factors; i.e., with greater responsibility, autonomy, challenge, etc.

The correct answer is: given more responsibility.

99
Q

In addition to identifying “bounded rationality” as an impediment to rational decision-making, Herbert Simon is known for his work on:
Select one:

A. artificial intelligence.
B. job burnout.
C. groupthink.
D. organizational culture.

A

Although Herbert Simon is probably best known for his work on individual decision-making, he was also a pioneer in the field of artificial intelligence.

a. CORRECT Simon believed there are two main goals in using and studying computers: (1) to augment human intelligence and (2) to help understand how humans think.
b. Incorrect See explanation above.
c. Incorrect See explanation above.
d. Incorrect See explanation above.

The correct answer is: artificial intelligence.

100
Q

According to Fiedler’s contingency model, which of the following is most important for maximizing a supervisor’s ability to effectively lead his or her employees?
Select one:

A. Each employee is highly motivated.
B. Each employee is very proficient at his/her job.
C. The leader has good relationships with his/her employees.
D. The leader is highly experienced.

A

Fiedler’s contingency model identifies three characteristics of the work situation that influence a leader’s ability to lead: leader-member relations, task structure, and the leader’s position power.

a. Incorrect See explanation for response c.
b. Incorrect See explanation for response c.
c. CORRECT Of the three situational characteristics, Fiedler considered leader-member relations to be most critical. According to Fiedler, factors that affect the quality of the relationship between the leader and members are the degree to which the leader perceives members to be similar to him/her and trusts and respects them and the degree to which members like, trust, and respect the leader and are loyal to him/her.
d. Incorrect See explanation for response c.

The correct answer is: The leader has good relationships with his/her employees.

101
Q

When a rater’s ratings on a criterion measure are biased by his or her knowledge of the ratee’s performance on the predictor, this is referred to as:
Select one:

A. criterion irrelevance.
B. criterion unreliability.
C. criterion contamination.
D. criterion deficiency.

A

Criterion contamination occurs when a criterion measure is “contaminated” by an extraneous variable.

a. Incorrect Relevance refers to the measure’s significance in terms of the ultimate criterion.
b. Incorrect A criterion measure is unreliable when scores are affected to a large degree by measurement error.
c. CORRECT Contamination often occurs when a rater’s knowledge of a ratee’s predictor performance biases ratings on the criterion measure.
d. Incorrect A criterion is deficient when it does not measure important aspects of the ultimate criterion.

The correct answer is: criterion contamination.

102
Q

In the context of expectancy theory, “valence” refers to:
Select one:

A. degree of effort.
B. level of performance.
C. need strength.
D. subjective worth of outcomes.

A

Expectancy theory distinguishes between three types of beliefs that contribute to motivation - expectancy, instrumentality, and valence.

a. Incorrect See explanation for response d.
b. Incorrect See explanation for response d.
c. Incorrect See explanation for response d.
d. CORRECT In the context of expectancy theory, valence refers to the value of available outcomes to the individual.

The correct answer is: subjective worth of outcomes.

103
Q

Research on working women suggests that combining work and family roles is most associated with:
Select one:

A. stress-related illness.
B. reduced involvement in family roles.
C. enhanced self-esteem.
D. low job commitment.

A

This topic was addressed in a review of the research on women and work (S. D. Phillips and A. R. Imhoff, Women and career development: A decade of research, Annual Review of Psychology, 48, 31-59, 1997).

a. Incorrect See explanation for response c.
b. Incorrect See explanation for response c.
c. CORRECT Empirical research suggests that, overall, dual roles for women have positive effects on self-esteem and other aspects of well-being.
d. Incorrect See explanation for response c.

The correct answer is: enhanced self-esteem.

104
Q

Research on Total Quality Management suggests that, when it fails, this is often because:
Select one:

A. the manager didn’t adapt his/her style to the demands of the situation.
B. the employees were not given adequate training.
C. the employees were not sufficiently involved in problem-solving and decision-making.
D. the manager often had to “satisfice” rather than “optimize” because of limited resources.

A

Knowing that TQM emphasizes employee involvement would have helped you choose the correct response to this question.

a. Incorrect See explanation for response c.
b. Incorrect See explanation for response c.
c. CORRECT TQM is a theory of management that emphasizes customer satisfaction, employee involvement, and continuous change. Studies looking at TQM failures have found that, despite the goal of commitment to employee involvement, employees often do not participate fully in problem-solving and decision-making.
d. Incorrect See explanation for response c.

The correct answer is: the employees were not sufficiently involved in problem-solving and decision-making.

105
Q

As defined in the EEOC Uniform Guidelines, “unfairness” is a problem when:
Select one:

A. different groups obtain consistently different scores on the predictor but similar scores on the criterion.
B. different groups obtain consistently different scores on the criterion but similar scores on the predictor.
C. the predictor has significantly different validity coefficients for different groups.
D. the predictor is equally invalid for all groups.

A

Unfairness is one of the possible causes of adverse impact.

a. CORRECT A test is “unfair” when it has a similar validity coefficient for members of two (or more) groups and members of the groups have similar criterion performance, but members of one group consistently score lower on the predictor than members of the other group(s). In this situation, if the same predictor cutoff is used for all individuals, there may be adverse impact for members of the low-scoring group.
b. Incorrect See explanation above.
c. Incorrect This describes differential validity.
d. Incorrect See explanation above.

The correct answer is: different groups obtain consistently different scores on the predictor but similar scores on the criterion.

106
Q

A low selection ratio and a moderate base rate are useful for:
Select one:

A. maximizing incremental validity.
B. reducing the likelihood of adverse impact.
C. increasing discriminant validity.
D. reducing measurement error.

A

For the exam, you want to have the terms selection ratio and base rate linked to the concept of incremental validity.

a. CORRECT A predictor is most likely to increase decision-making accuracy (i.e., have good incremental validity) when there are many applicants to choose from (a low selection ratio) and when the current technique produces a moderate number of correct decisions (moderate base rate).
b. Incorrect Selection ratio and base rate are not directly related to adverse impact.
c. Incorrect Discriminant validity refers to low correlations between measures that assess different traits. It is used as evidence of construct validity.
d. Incorrect Selection ratio and base rate are not relevant to the extent of measurement error.

The correct answer is: maximizing incremental validity.

107
Q

An employer interested in increasing the creativity of workers would be best advised to do all of the following except:
Select one:

A. provide supportive supervision.
B. let workers know that their work will be critically evaluated.
C. hire employees who score high on a measure of openness to experience.
D. increase the physical distance between workers in the workplace.

A

Research investigating the impact of various personal and contextual characteristics on the creativity of employees was recently reviewed by C. Shalley, J. Zhou, and G. Oldham [The effects of personal and contextual effects on creativity: Where should we go from here?, Journal of Management, 30(6), 933-958, 2004].

a. Incorrect The research has confirmed that supportive supervision has a positive effect on employee creativity.
b. CORRECT Although the expectation that one’s work will be evaluated in a nonjudgmental way (i.e., that evaluations will be developmental rather than judgmental) has a positive impact on creativity, the expectation that one’s work will be critically evaluated tends to diminish creativity.
c. Incorrect Of the five personality factors, openness to experience has been found to be most strongly related to creativity.
d. Incorrect Reducing the overall density of the work environment has been linked to improvements in worker creativity.

The correct answer is: let workers know that their work will be critically evaluated.

108
Q

Which of the following is the most commonly used criterion-measure in organizational settings?
Select one:

A. supervisor ratings
B. standardized performance tests
C. quantitative measures of production
D. assessment centers

A

Although some jobs lend themselves to direct (objective) criterion measures, most require that subjective measures be used.

a. CORRECT Supervisor ratings continue to be the most frequently used methods for assessing employees, probably because they are the easiest and quickest way of evaluating employees.
b. Incorrect Performance tests are more likely to be used as predictors than criteria.
c. Incorrect Quantitative indices (e.g., number of units produced) are unavailable for many jobs.
d. Incorrect Assessment centers are costly in terms of time and money and are used in large organizations for the purpose of managerial placements and promotions.

The correct answer is: supervisor ratings

109
Q

In terms of initiating structure and consideration, the two core dimensions of leadership, research in organizations suggests that:
Select one:

A. consistent with gender stereotypes, female leaders are higher in consideration, while male leaders are higher in initiating structure.
B. contrary to gender stereotypes, female leaders are higher in initiating structure, while males are higher in consideration.
C. contrary to gender stereotypes, males and females do not differ substantially in terms of consideration or initiating structure.
D. contrary to gender stereotypes, males are higher than females in both consideration and initiating structure.

A

A considerable body of research has addressed the core dimensions of leadership and, more recently, gender differences on these dimensions.

a. Incorrect See explanation for response c.
b. Incorrect See explanation for response c.
c. CORRECT Although male and female leaders differ in terms of decision-making style, overall, their leadership styles do not show consistent discrepancies.
d. Incorrect See explanation for response c.

The correct answer is: contrary to gender stereotypes, males and females do not differ substantially in terms of consideration or initiating structure.

110
Q

When asked to recall an accident you witnessed two weeks ago, which of the following will be involved?
Select one:

A. procedural memory
B. semantic memory
C. representational memory
D. episodic memory

A

Different experts categorize the types of memory in different ways. One frequently cited method divides memory into procedural and declarative memory with declaritive memory including semantic and episodic (autobiographical) memories.

a. Incorrect Procedural memory stores information about how to do things (e.g., ride a bicycle, play tennis).
b. Incorrect Semantic memory stores information about language, facts, and concepts.
c. Incorrect Some authors use the term “representational memory” to refer to memories for general knowledge, common sense, and skills.
d. CORRECT Episodic memory is a record of personally experienced events.

The correct answer is: episodic memory

111
Q

Underlying the technique known as stress inoculation is the assumption that:
Select one:

A. exposure to stressful situations reduces the stress reaction as the result of habituation.
B. exposure to stressful situations leads to catharsis, which results in reduced reactivity.
C. successful coping with stress in the present strengthens the ability to cope with stress in the future.
D. expecting exposure to a stressful stimulus reduces its impact.

A

Stress inoculation is based on the assumption that it is possible to “inoculate” a person against future stress.

a. Incorrect See explanation for response c.
b. Incorrect See explanation for response c.
c. CORRECT When using this technique, a person acquires a variety of skills to help him/her cope with the current problem as well as with future problems. The assumption is that successful coping with stress in the present will reduce its impact in the future.
d. Incorrect See explanation for response c.

The correct answer is: successful coping with stress in the present strengthens the ability to cope with stress in the future.

112
Q

Thorndike’s law of effect predicts that:
Select one:

A. distributed practice is more effective than massed practice when learning a complex skill.
B. transfer of training is maximized when the learning and performance situations share similar characteristics.
C. a behavior is more likely to be repeated when it is followed by a reinforcing (satisfying) consequence.
D. a behavior is more likely to occur when it is preceded by a cue that signals that reinforcement will follow the behavior.

A

Knowing that Thorndike’s law of effect contributed to Skinner’s theory of operant conditioning would have helped you identify the correct answer to this question.

a. Incorrect Although this is true, it is not predicted by the law of effect.
b. Incorrect This answer refers to the theory of identical elements.
c. CORRECT Thorndike developed several basic laws of learning, the most important of which is the law of effect, which predicts that any response that is followed by “a satisfying state of affairs” is likely to be repeated.
d. Incorrect This answer refers to stimulus control - i.e., a behavior is under stimulus control when it occurs in the presence of cues that signal that reinforcement will follow the behavior but not in the presence of cues that signal that reinforcement will not be provided.

The correct answer is: a behavior is more likely to be repeated when it is followed by a reinforcing (satisfying) consequence.

113
Q

Which of the following statements best illustrates Beck’s “cognitive triad”?
Select one:

A. If I don’t get an “A” in this class, my life will be ruined.
B. That’s the story of my life - I never do anything right.
C. I saw my wife with a man at a restaurant yesterday - she must be having an affair.
D. No one is coming to my party - everyone must think I’m a nerd.

A

Beck’s cognitive triad predicts that people with depression have negative thoughts about themselves, the future, and the world.

a. Incorrect This statement is a better example of Ellis’s basic irrational beliefs. A person with depression would probably assume that it will be impossible to get an A.
b. CORRECT This statement is most illustrative of Beck’s cognitive triad - i.e., it reflects a negative view of one’s own ability and the belief that negative events will always occur.
c. Incorrect See explanation above.
d. Incorrect See explanation above.

The correct answer is: That’s the story of my life - I never do anything right.

114
Q

Treisman and Gelade’s (1980) feature integration theory would be of most interest to a psychologist conducting research on:
Select one:

A. the impact of arousal on performance.
B. the difference between echoic and iconic memory.
C. visual attention.
D. forgetting.

A

Knowing that the full name of feature integration theory is “feature integration theory of attention” would have helped you identify the correct answer to this questions.

a. Incorrect See explanation for response c.
b. Incorrect See explanation for response c.
c. CORRECT According the feature integration theory, the perception of objects involves two stages - preattentive in which the basic aspects of an object are perceived in parallel (e.g., edges, size, color) and attentive in which these features are “glued” together into a coherent whole through a serial process that depends on focal attention.
d. Incorrect See explanation for response c.

The correct answer is: visual attention.

115
Q

As defined by Aaron Beck, overgeneralization, personalization, and catastrophizing are:
Select one:

A. the “cognitive triad.”
B. cognitive distortions.
C. cognitive schemata.
D. automatic thoughts.

A

Aaron Beck describes several cognitive phenomena that contribute to depression, anxiety, and other disorders.

a. Incorrect The cognitive triad refers specifically to the cognitive profile underlying depression (i.e., negative views of oneself, the world, and the future).
b. CORRECT The cognitive distortions (errors in reasoning) identified by Beck include overgeneralization, personalization, and catastrophizing.
c. Incorrect Schemata are ways of organizing and interpreting experiences that develop in early childhood and may be latent until later in life when they are activated by stress.
d. Incorrect Automatic thoughts are repetitive, automatic self-statements that are elicited by certain stimuli and that are associated with strong emotions.

The correct answer is: cognitive distortions.

116
Q

A psychologist is consulted by a medical doctor who is concerned about a patient in a pain clinic. The patient constantly asks for his pain medication and the nurses, who get tired of his barrage of requests, give him too much medication. To get the man to cut down on his requests, the psychologist is most likely to recommend:
Select one:

A. giving him a sedative to keep him quiet.
B. giving him his medication after every twenty requests.
C. giving him his medication once every four hours.
D. giving him higher but less frequent doses of medication.

A

The patient is currently on a variable ratio schedule of reinforcement. That is, he is being reinforced with pain medication after a variable amount of responses (the requests). Behaviors reinforced on a variable ratio schedule are the most difficult to extinguish or reduce, since the relationship between responding and reinforcement is unpredictable (i.e., the man in this case never knows when his requests are going to cause the nurses to break down and give him the medication, so he might as well keep trying).

a. Incorrect Since psychologists are not M.D.s, this is not the type of recommendation a psychologist is likely to make.
b. Incorrect This represents a fixed ratio schedule, in which reinforcement occurs after a fixed number of responses. While behaviors reinforced on a fixed ratio schedule are easier to reduce than those reinforced on a variable ratio schedule, those reinforced on a fixed interval schedule (response c) are even easier to extinguish. (Also, this schedule could result in giving the man too much medication.)
c. CORRECT This represents a fixed interval schedule, in which reinforcement is provided after a fixed interval of time (in this case, once every four hours). Compared to behaviors reinforced on the other intermittent schedules, behaviors reinforced on a fixed interval schedule are the easiest to reduce or extinguish, since the frequency of response is not connected to the delivery of reinforcement.
d. Incorrect This strategy would represent reinforcement of an undesirable behavior and is thus more likely to maintain the behavior than reduce or eliminate it.

The correct answer is: giving him his medication once every four hours.

117
Q

The phenomenon known as “spontaneous recovery” confirms the hypothesis that the loss of a conditioned response represents which of the following?
Select one:

A. decay of memory traces
B. stimulus discrimination
C. habituation
D. inhibition

A

After a classically conditioned response is extinguished, the subsequent presentation of the conditioned stimulus often produces a weak conditioned response. For example, a dog’s conditioned response of salivating to a bell will often, after it has been extinguished, reappear when the dog later hears the sound of a bell. This phenomenon is known as “spontaneous recovery.”

a. Incorrect If the loss of a conditioned response represented a decay of memory traces, the response would have to be relearned. In other words, spontaneous recovery, or immediate reappearance of the conditioned response upon the reapplication of the conditioned stimulus, would not occur.
b. Incorrect Stimulus discrimination refers to an organism’s ability to respond to a specific stimulus with a conditioned response, but not to similar stimuli. It has nothing to do with spontaneous recovery.
c. Incorrect Habituation has not been used as an explanation for spontaneous recovery.
d. CORRECT The fact that an extinguished conditioned response can reappear upon the reapplication of the conditioned stimulus suggests that the response is never lost or forgotten but is merely suppressed or inhibited.

The correct answer is: inhibition

118
Q

In Aaron Beck’s cognitive-behavioral therapy, interpretations and assumptions underlying a client’s problems are:
Select one:

A. jointly explored for their origins by the therapist and client during the initial stages of therapy.
B. identified and actively challenged or refuted by the therapist.
C. viewed as “testable hypotheses.”
D. viewed as “incongruent with reality.”

A

A key characteristic of Beck’s approach is its reliance on “collaborative empiricism.”

a. Incorrect In CBT, the focus is on the here-and-now rather than on past.
b. Incorrect See explanation for response c.
c. CORRECT In Beck’s cognitive-behavioral therapy, the therapist and client are co-investigators who approach the client’s problems in a scientific manner. For example, the client’s interpretations and assumptions are conceptualized and treated as testable hypotheses.
d. Incorrect See explanation for response c.

The correct answer is: viewed as “testable hypotheses.”

119
Q

Which of the following illustrates the use of negative reinforcement?
Select one:

A. A child’s usual allowance is reduced by a specific amount each time he misbehaves.
B. A child is allowed to watch television for one hour when he finishes his homework.
C. Restrictions on a child’s activities are removed each time he helps with household chores.
D. Restrictions on a child’s activities are imposed each time he argues with his sister.

A

Negative reinforcement occurs when the removal of a stimulus following a response increases the occurrence of that response.

a. Incorrect This is an example of negative punishment.
b. Incorrect This is an example of positive reinforcement.
c. CORRECT In this situation, restrictions are removed following a behavior in order to increase the behavior. This is referred to as negative reinforcement.
d. Incorrect This is also an example of negative punishment; i.e., something is being taken away following a behavior, apparently in order to decrease the behavior.

The correct answer is: Restrictions on a child’s activities are removed each time he helps with household chores.

120
Q

Craik and Lockhart’s (1972) levels of processing model of memory:
Select one:

A. describes memory in terms of a “dual store.”
B. implies that elaborative rehearsal is more effective than maintenance rehearsal.
C. implies that the duration of rehearsal is more important that the depth of rehearsal.
D. focuses on the biological correlates of the short- and long-term memory.

A

Craik and Lockart (1972) proposed the levels of processing model as an alternative to the three-store model of memory. See the Learning Theory chapter for additional information about this model.

a. Incorrect See explanation for response b.
b. CORRECT According to the levels of processing model, the semantic level is the deepest level of processing and produces the best recall. Elaborative rehearsal involves encoding material semantically (in terms of meaning).
c. Incorrect See explanation for response b.
d. Incorrect See explanation for response b.

The correct answer is: implies that elaborative rehearsal is more effective than maintenance rehearsal.

121
Q

Subjects learn a task while under the influence of a CNS stimulant. Three days later, half of the subjects are tested while under the influence of the drug; the other half are tested while drug-free. Subjects who have been given the drug perform better on the task during testing. This result is predicted by which of the following?
Select one:

A. state dependent learning
B. proactive facilitation
C. drug-dependence effect
D. priming

A

Remember – when it doubt, pick the response that “sounds right”!

a. CORRECT As its name implies, state dependent learning refers to the fact that recall is best when the individual is in the same state (e.g., drug-induced or emotional state) during learning and recall.
b. Incorrect This is a “made up” term.
c. Incorrect This is another “made up” term.
d. Incorrect Priming occurs when a memory is triggered by a cue (e.g., remembering the word priming when given the letters “prm”).

The correct answer is: state dependent learning

122
Q

Relaxation training seems to be less effective than biofeedback for treating which of the following disorders?
Select one:

A. chemotherapy-induced nausea and vomiting
B. tension headaches
C. lower back pain
D. Raynaud’s disease

A

For many of the disorders that have been treated with relaxation training and biofeedback, the two techniques have been found to have beneficial effects but not to differ substantially in the degree of their effectiveness.

a. Incorrect Relaxation has been found helpful in some cases of chemotherapy-induced nausea and vomiting; biofeedback is not a recommended treatment for this problem.
b. Incorrect Relaxation and biofeedback are about equally effective for tension headaches.
c. Incorrect Relaxation and biofeedback appear to be about equally effective for lower back pain.
d. CORRECT For Raynaud’s disease, temperature biofeedback has been found to be superior to relaxation training.

The correct answer is: Raynaud’s disease

123
Q

In classical conditioning, ________ occurs when human or animals subjects are unable to form an association between a new neutral stimulus and a unconditioned stimulus because the new neutral stimulus provides redundant information (i.e. it provides the same information as the original conditioned stimulus).
Select one:

A. overshadowing
B. blocking
C. experimental neurosis
D. reciprocal inhibition

A

For the exam, you want to be familiar with the terms listed in the answers to this question. These are described in the Learning Theory chapter of the written study materials.

a. Incorrect See explanation for response b.
b. CORRECT Blocking occurs when an association between a CS and a US has been made and, subsequently, the presence of the CS blocks an association being made between a new neutral stimulus and the US when the CS and the new neutral stimulus are presented together prior to the US.
c. Incorrect See explanation for response b.
d. Incorrect See explanation for response b.

The correct answer is: blocking

124
Q

Aaron Beck’s interactions with therapy clients is best described as:
Select one:

A. confrontative.
B. consensual.
C. covert.
D. collaborative.

A

Aaron Beck initially developed cognitive therapy (CT) as a treatment for depression. His therapeutic approach is described as “collaborative empiricism.”

a. Incorrect Ellis, the founder of rational emotive behavior therapy, is known for his confrontative approach when working with therapy clients.
b. Incorrect The term “consensual” is vague and not the best description of Beck’s approach.
c. Incorrect Beck does not rely on covert techniques when working with clients.
d. CORRECT In therapy, Beck treats the client as a colleague and works together (collaboratively) with the client to, for example, identify therapy goals and investigate the client’s assumptions.

The correct answer is: collaborative.

125
Q

The behavioral technique known as response cost makes use of which of the following to alter a person’s behavior?
Select one:

A. positive punishment
B. negative punishment
C. positive reinforcement
D. negative reinforcement

A

When using response cost, a stimulus is removed each time the target behavior is performed in order to reduce or eliminate that behavior.

a. Incorrect Positive punishment involves applying a stimulus following a behavior to reduce that behavior.
b. CORRECT Negative punishment involves removing a stimulus following a behavior to reduce that behavior. In the case of response cost, the stimulus is something that the individual finds reinforcing such as money or computer privileges.
c. Incorrect Positive reinforcement involves applying a stimulus following a behavior to increase that behavior.
d. Incorrect Negative reinforcement involves removing a stimulus following a behavior to increase that behavior.

The correct answer is: negative punishment

126
Q

The research has confirmed that working memory is negatively affected by increasing age. According to Baddeley (1986), this decline is due primarily to:
Select one:

A. reduced efficiency of sensory register.
B. reduced capacity of short-term memory.
C. the susceptibility of older adults to proactive and retroactive interference.
D. the negative impact of aging on the central executive.

A

Baddeley (1986) proposed that working memory consists of three components: a visuospatial sketchpad that maintains visual images, an auditory/phonological loop that maintains auditory/verbal information, and a central executive that directs attention and controls and coordinates the visuospatial and auditory/phonological subsystems. Being familiar with these three components would have helped you identify the correct answer to this question.

a. Incorrect See explanation for response d.
b. Incorrect See explanation for response d.
c. Incorrect See explanation for response d.
d. CORRECT Research by Baddeley and others has confirmed that tasks that depend on the central executive - i.e., tasks requiring the reorganization of information or the coordination of information in sensory and auditory channels - are the ones that are most adversely affected by increasing age. Additional information on Baddeley’s model is provided in the Learning Theory chapter of the written study materials.

The correct answer is: the negative impact of aging on the central executive.

127
Q

When using aversion therapy to eliminate a client’s shoe fetish, an electric shock or other unpleasant stimulus acts as a(n):
Select one:

A. negative punishment.
B. negative reinforcement.
C. conditioned stimulus.
D. unconditioned stimulus.

A

To answer this question, you need to know that aversion therapy is a type of counterconditioning that is used to eliminate undesirable behaviors by pairing the stimulus associated with the undesirable behavior with a stimulus that produces an unpleasant reaction.

a. Incorrect Knowing that aversion therapy is a type of classical conditioning (counterconditioning) would have helped you eliminate answers a and b since punishment and reinforcement are associated with operant conditioning.
b. Incorrect See explanation for response a.
c. Incorrect When using aversion therapy to eliminate a shoe fetish, the shoe is the conditioned stimulus (CS).
d. CORRECT The shoe is the conditioned stimulus and is paired with electric shock or other stimulus that naturally produces an unpleasant reaction so that, eventually, the shoe also produces the unpleasant reaction. The electric shock or other stimulus that naturally produces an unpleasant reaction is the unconditioned stimulus (US).

The correct answer is: unconditioned stimulus.

128
Q

Charging telephone customers for using directory assistance to obtain phone numbers in order to decrease the usage of directory assistance is an application of which of the following?
Select one:

A. Premack Principle
B. overcorrection
C. negative reinforcement
D. response cost

A

In this situation, money is being taken away following a behavior in order to decrease that behavior. In operant terms, this is referred to as negative punishment.

a. Incorrect The Premack Principle is a type of positive reinforcement.
b. Incorrect Overcorrection is a form of positive punishment. It involves applying a penalty (usually correcting the consequences of the behavior) following a behavior in order to decrease that behavior.
c. Incorrect Negative reinforcement is used to increase, not decrease, a behavior.
d. CORRECT Response cost is a type of negative punishment. It involves removing a specific stimulus (e.g., money) following a behavior to decrease that behavior.

The correct answer is: response cost

129
Q

A husband consistently says loving things to his wife when she talks about sports but ignores her whenever she complains about her job. The husband’s behavior is most similar to which of the following techniques?
Select one:

A. overcorrection
B. chaining
C. Premack Principle
D. differential reinforcement

A

The husband’s behavior reflects a combination of positive reinforcement and extinction.

a. Incorrect Overcorrection is involves a combination of restitution (having the individual “fix” the negative consequences of his/her behavior) and positive practice (having the individual practice alternative behaviors).
b. Incorrect Chaining is used to establish complex behavior chains.
c. Incorrect When using the Premack Principle, a high frequency behavior is used as a reinforcer for a low frequency behavior.
d. CORRECT Differential reinforcement involves reinforcing desirable behavior while ignoring undesirable behavior. Although the husband’s reaction to his wife isn’t exactly the same as the differential reinforcement procedure, this answer best describes what he is doing since it combines positive reinforcement and extinction.

The correct answer is: differential reinforcement

130
Q

According to the Yerkes-Dodson law, learning and performance are maximized when:
Select one:

A. level of arousal is low.
B. level of arousal is moderate.
C. elaborative rehearsal is employed.
D. encoding specificity is high.

A

Yerkes and Dodson (1908) were interested in the relationship between level of arousal and the efficiency of learning and performance.

a. Incorrect See explanation for response b.
b. CORRECT These investigators found an inverted-U shape relationship between arousal and performance with maximum performance being associated with moderate levels of arousal.
c. Incorrect See explanation for response b.
d. Incorrect See explanation for response b.

The correct answer is: level of arousal is moderate.

131
Q

With regard to the serial position effect, the primacy effect is to __________ as the recency effect is to __________.
Select one:

A. sensory memory; short-term memory
B. short-term memory; long-term memory
C. long-term memory; short-term memory
D. remote long-term memory; recent long-term memory

A

The serial position effect is used to support the distinction between long- and short-term memory stores. It occurs when people are asked to recall a list of words immediately after reading the list, and they recall words in the beginning of the list (primacy effect) and end of the list (recency effect) better than words in the middle of the list.

a. Incorrect See explanation for response c.
b. Incorrect See explanation for response c.
c. CORRECT The primacy effect apparently occurs because words at the beginning of the list have been transferred to long-term memory, while the recency effect is due to the fact that the words at the end of the list are still in short-term memory.
d. Incorrect See explanation for response c.

The correct answer is: long-term memory; short-term memory

132
Q

According to _____, gender role development involves developing conceptual frameworks (schemas) of masculinity and femininity as the result of sociocultural experiences.
Select one:

A. Bem
B. Kohlberg
C. Freud
D. Mischel

A

For the exam, you want to be familiar with the gender role development theories of the four individuals listed in the answers to this question. These are described in the Lifespan Development chapter of the written study materials.

a. CORRECT Knowing that Bem’s theory is known as “gender schema theory” would have helped you identify this as the correct answer.
b. Incorrect See explanation for response a.
c. Incorrect See explanation for response a.
d. Incorrect See explanation for response a.

The correct answer is: Bem

133
Q

A securely attached infant:
Select one:

A. may or may not become upset when the parent leaves the room; is comforted by a stranger; and seeks contact with the parent on reunion.
B. becomes upset when the parent leaves the room; is unlikely to be comforted by a stranger; and seeks comfort from the parent on reunion.
C. becomes upset when the parent leaves the room; is comforted by a stranger; and may or may not seek contact with the parent on reunion.
D. becomes upset when the parent leaves the room; seeks comfort from a stranger if upset; and resumes play when the parent returns.

A

As described in the Lifespan Development chapter of the written study materials, research by Ainsworth and her colleagues identified four main patterns of attachment: secure, insecure/avoidant, insecure/ambivalent, and disorganized/disoriented.

a. Incorrect See explanation for response b.
b. CORRECT Securely attached children become upset when a parent leaves the room, are not comforted by a stranger, and, on reunion with the parent, seek contact and comfort but gradually return to play.
c. Incorrect See explanation for response b.
d. Incorrect See explanation for response b.

The correct answer is: becomes upset when the parent leaves the room; is unlikely to be comforted by a stranger; and seeks comfort from the parent on reunion.

134
Q

Which of the following aspects of memory is most negatively impacted by increasing age in adulthood?
Select one:

A. sensory memory
B. secondary memory
C. memory span
D. implicit memory

A

The effects of increasing age vary for different aspects of memory.

a. Incorrect Sensory memory is relatively unaffected by increasing age.
b. CORRECT Secondary memory is the aspect of long-term memory that contains newly acquired information. The studies have consistently found that secondary memory is the type of memory that is most negatively affected by increasing age.
c. Incorrect Memory span (also known as primary memory) is an aspect of short-term memory and is relatively unaffected by increasing age.
d. Incorrect Implicit memory (memory that does not require conscious effort) is less affected than explicit memory by increasing age.

The correct answer is: secondary memory

135
Q

In response to the cries of an infant:
Select one:

A. only the infant’s mother exhibits an increase in heart rate and blood pressure.
B. only the infant’s caregivers and older siblings exhibit an increase in heart rate and blood pressure.
C. only experienced parents exhibit an increase in heart rate and blood pressure.
D. most adults (both parents and non-parents) exhibit an increase in heart rate and blood pressure.

A

Research has found that parents and non-parents all respond physiologically to an infant’s cries.

a. Incorrect See explanation for response d.
b. Incorrect See explanation for response d.
c. Incorrect See explanation for response d.
d. CORRECT Most adults have an automatic physiological response to an infant’s cries, with the pain cry producing the srongest response.

The correct answer is: most adults (both parents and non-parents) exhibit an increase in heart rate and blood pressure

136
Q

Research has found maternal depression to be a risk factor for the emotional, behavioral, and cognitive development of children. For example, there is evidence that toddlers of mothers with severe depression tend to be:
Select one:

A. excessively acquiescent.
B. hypervigilant and anxious.
C. antagonistic and belligerent.
D. passively noncompliant.

A

The studies have shown that mothers with chronic and severe depression are less engaged with their children and have problems setting limits and that these parenting behaviors are associated with certain child outcomes.

a. Incorrect See explanation for response d.
b. Incorrect See explanation for response d.
c. Incorrect See explanation for response d.
d. CORRECT Several studies have confirmed that the toddlers of mothers with chronic and severe depressive symptoms are more likely than other children to have adjustment problems. For example, there is evidence that children of severely depressed mothers have low levels of independence and tend to be passively noncompliant (i.e., they ignore requests rather than more actively refuse to comply). See, e.g., L. Kuczynski & G. Kochanska, Development of children’s noncompliance strategies from toddlerhood to age 5, Developmental Psychology, 26(3), 398-408, 1990.

The correct answer is: passively noncompliant.

137
Q

In females with ____________, part or all of one X chromosome is missing.
Select one:

A. Mullerian syndrome
B. Fragile X syndrome
C. Cushing syndrome
D. Turner syndrome

A

Females with Turner syndrome do not develop secondary sex characteristics during puberty. They are typically short with a thick, webbed neck and are usually infertile due to a lack of normal ovaries. Many have mild learning problems.

a. Incorrect Persistent Mullerian duct syndrome is a congenital disorder that is caused by a lack of anti-Mullerian hormone or receptors for this hormone.
b. Incorrect Fragile X syndrome is a hereditary condition that causes an intellectual disability and other cognitive impairments as well as certain physical and behavioral abnormalities.
c. Incorrect Cushing syndrome is due to malfunctioning of the adrenal cortex.
d. CORRECT Females with this disorder have an XO chromosome pattern (i.e., one X chromosome with no other functioning sex chromosome).

The correct answer is: Turner syndrome

138
Q

Authoritative parenting, which has been associated with the most positive outcomes for children and adolescents, is best described as:
Select one:

A. high in control and high in acceptance.
B. high in control but low in responsiveness.
C. low in control and high in acceptance.
D. low in control and low in responsiveness.

A

Authoritative parents recognize that they have more knowledge and skills than their children but also view the rights of parent and child as reciprocal.

a. CORRECT You may have had some trouble with this one because we ordinarily think of parental control in a negative sense. However, authoritative parents do exert control over their children but do so through the use of reasoning and placing high (but fair) demands on their children for high performance. The research indicates that, during adolescence, authoritative parenting is associated with better school performance and better social adjustment.
b. Incorrect See explanation for response a.
c. Incorrect See explanation for response a.
d. Incorrect See explanation for response a.

The correct answer is: high in control and high in acceptance.

139
Q

With regard to language acquisition, a child’s use of his/her knowledge of the meaning of words to make inferences about their grammatical type or structure is referred to as:
Select one:

A. semantic bootstrapping.
B. prosodic bootstrapping.
C. syntactic bootstrapping.
D. morphological bootstrapping.

A

This is one of those “distant galaxy” questions but considering the meaning of the words listed in the answers may have helped you recognize the correct response.

a. CORRECT Semantic refers to “meaning,” and semantic bootstrapping is a term used in the field of psycholinguistics to describe how children use their knowledge of the meaning of words to infer their syntactical category which, in turn, allows them to understand syntactical rules and construct grammatically correct sentences.
b. Incorrect Prosodic bootstrapping refers to using the prosody (pitch, rhythm, etc.) of an utterance to make deductions about syntax.
c. Incorrect Syntactic bootstrapping refers to using syntactical information to make deductions about a word’s meaning.
d. Incorrect Morphological bootstrapping refers to using knowledge about morphemes to deduce the syntax or meaning of a word - e.g., deducing that a word is an action word (verb) because it ends in “ing.”

The correct answer is: semantic bootstrapping.

140
Q

Huntington’s disease, Marfan’s syndrome, and Von Willebrand’s disease are due to:
Select one:

A. an autosomal dominant gene.
B. an autosomal recessive gene.
C. an X-linked dominant gene.
D. an X-linked recessive gene.

A

The three disorders listed in the question are all genetic disorders.

a. CORRECT All three disorders are caused by an autosomal dominant gene, which means that they occur in the presence of only one gene on a chromosome that is not a sex (X or Y) chromosome.
b. Incorrect See explanation above.
c. Incorrect See explanation above.
d. Incorrect See explanation above.

The correct answer is: an autosomal dominant gene.

141
Q

Offense history has been found to be a good predictor of re-offense among juvenile offenders. Of the offense history factors, which is least predictive of re-offense?
Select one:

A. age of first contact with the law
B. length of first incarceration
C. age at commencement of the first offense
D. number of prior arrests

A

This is a difficult question because the correct answer is inconsistent with the axiom that “past behavior is the best predictor of future behavior.”

a. Incorrect The correlations reported in the literature for various risk factors and re-offense were summarized by C. Schwalbe, S. Day, & M. Fraser [Risk assessment in the juvenile justice system, retrieved from: http://www.unc.edu/ncjcp/Riskassessmentreview3.htm]. They report an average correlation of -.341 between age at first contact with the law and re-offense.
b. Incorrect These authors report an average correlation of .187 between length at first incarceration and re-offense.
c. Incorrect The average correlation obtained by Schwalbe et al. for age of commencement of first offense and re-offense is -.346.
d. CORRECT Schwalbe and colleagues report an average correlation of only .058 between number of prior arrests and re-offense.

The correct answer is: number of prior arrests

142
Q

Research has found that the effects of sexual abuse for a child depend on several factors. Which of the following most accurately describes a finding of these studies?
Select one:

A. The effects are less severe when the abuse was committed by a family member.
B. The effects are less severe when the abuse was committed by a stranger.
C. The younger the child is at the time of the initial abuse, the less severe the effects.
D. The younger the age of the perpetrator, the more severe the effects.

A

Although many individuals exhibit long-term adverse effects of childhood sexual abuse, certain conditions have been linked with better outcomes. See the Lifespan Development chapter for additional information on the risk factors and consequences associated with child abuse.

a. Incorrect The effects of abuse are usually worse when the perpetrator is a family member.
b. CORRECT Abuse by a stranger is associated with less severe effects.
c. Incorrect Younger age at the time of initial abuse is associated with more damaging outcomes.
d. Incorrect The older the age of the perpetrator, the more severe the effects for the victim.

The correct answer is: The effects are less severe when the abuse was committed by a stranger.

143
Q

Research on the effects of age on second-language learning has shown that the ability to achieve native-like pronunciation of a second language is:
Select one:

A. more likely when exposure begins in early or middle childhood.
B. more likely when exposure begins in adolescence.
C. more likely when exposure begins in early adulthood.
D. unrelated to the age when exposure begins.

A

Research on the relationship between age and second-language learning is very inconsistent and has not led to a consensus of opinion; and, perhaps, the best conclusion is that the relationship is mediated by a number of factors.

a. CORRECT While there is evidence that adolescents and adults initially make faster progress in acquiring a second language (especially with regard to syntax and morphology), individuals exposed to a second language during childhood are more likely to speak with a native accent. In other words, pronunciation is the one aspect of language learning for which there is fairly consistent evidence that “younger is better.”
b. Incorrect See explanation for response a.
c. Incorrect See explanation for response a.
d. Incorrect See explanation for response a.

The correct answer is: more likely when exposure begins in early or middle childhood.

144
Q

Patterson and his colleagues focus on the family in describing the origins of aggressiveness in children. Specifically, they relate high aggressiveness in children to:
Select one:

A. attachment insecurity in infancy.
B. parental permissiveness and overindulgence.
C. parental rejection and lack of warmth.
D. coercive exchanges between parent-child.

A

Patterson’s coercive family interaction model of aggression in children focuses on parent behaviors but also recognizes the role of coercive exchanges between parents and their children in which both act aggressively in order to alter the behavior of the other. Over time, this pattern tends to escalate in intensity.

a. Incorrect See explanation for response d.
b. Incorrect See explanation for response d.
c. Incorrect See explanation for response d.
d. CORRECT According to Patterson and his colleagues, aggressiveness in children is initiated by parental modeling and reinforcement. Over time, however, as children acquire aggressive behaviors, the aggressive interactions between parents and their children increase in intensity.

The correct answer is: coercive exchanges between parent-child.

145
Q

Which of the following best describes Kohlberg’s claim about the relationship between moral judgment and moral action?
Select one:

A. There is a strong relationship between moral judgment and moral action at all stages of moral development.
B. The relationship between moral judgment and moral action is strongest at the lower stages of moral development.
C. The relationship between moral judgment and moral action is strongest at the higher stages of moral development.
D. There is no predictable relationship between moral judgment and moral action at any stage of moral development.

A

Kohlberg distinguished between moral judgment and moral action, and his theory of moral development focuses on moral judgment.

a. Incorrect See explanation for response c.
b. Incorrect See explanation for response c.
c. CORRECT Kohlberg proposed that there is a greater correspondence between moral judgment and action at the higher stages of moral development because the higher stages employ more stable and generalizable standards.
d. Incorrect See explanation for response c.

The correct answer is: The relationship between moral judgment and moral action is strongest at the higher stages of moral development.

146
Q

The gross motor milestones for children _____ years of age include pedaling a tricycle, kicking a stationary ball forward, throwing a ball overhand, going up stairs using alternate feet, and running without falling.
Select one:

A. two
B. three
C. four
D. five

A

For the exam, you want to be familiar with major developmental milestones. (Note that, because reported milestones for different ages vary somewhat in the literature, it’s important to consider all of the milestones listed and not over-focus on any single milestone.)

a. Incorrect Gross motor milestones for two-year-old children include climbing up on furniture, walking alone, walking up and down stairs one at a time, and catching a large ball using hands and chest.
b. CORRECT The motor skills listed in this question are gross motor milestones for three-year-old children.
c. Incorrect Gross motor milestones for four-year-old children include hopping on one foot, standing on one leg, running to kick a ball, bouncing a ball, and going down stairs using alternate feet.
d. Incorrect Gross motor milestones for five-year-old children include riding a scooter or bicycle, jumping rope, hopping on alternate feet, jumping over low obstacles, and catching a ball with both hands and arms bent.

The correct answer is: three

147
Q

Alcohol consumption by a pregnant woman is likely to have the most adverse effects on her baby’s prenatal development when the woman drinks during:
Select one:

A. the first trimester.
B. the second trimester.
C. the third trimester.
D. the first or the third trimester.

A

Alcohol consumption by a woman during pregnancy can have detrimental effects on the developing fetus, with the severity and extent of the effects depending on the amount of alcohol consumed and the time during which it is consumed.

a. CORRECT There does not appear to be any safe time for alcohol consumption during pregnancy. However, the worst effects overall (i.e., for many different aspects of development) are associated with drinking during the first trimester.
b. Incorrect See explanation for response a.
c. Incorrect See explanation for response a.
d. Incorrect See explanation for response a.

The correct answer is: the first trimester.

148
Q

Generally speaking, the psychological adjustment of children with chronic illness is most dependent on:
Select one:

A. peer acceptance.
B. family attitudes toward the illness.
C. the child’s personality type.
D. the child’s level of functional disability.

A

If you are unfamiliar with the research in this area, the best approach to picking an answer would be to select the most general response.

a. Incorrect Although peer acceptance is important for children and adolescents with chronic illness, it is not as important as the level of disability.
b. Incorrect Family factors have been linked to outcomes in chronic illness but “family attitudes toward the illness” is too specific to be the correct response.
c. Incorrect The research has not linked outcome to particular personality types.
d. CORRECT A number of studies have linked severity of the illness to outcome. However, some authors argue that it is not just the illness severity but the level of functional disability that determines the impact of the illness on the child (see, e.g., W. T. Garrison and S. McQuiston, Chronic illness during childhood and adolescence, Newbury Park, CA: Sage, 1989).

The correct answer is: the child’s level of functional disability.

149
Q

Research conducted in the past 15 to 20 years on parents’ gender-related stereotypes of their offspring has found that:
Select one:

A. parents perceive boys and girls differently within 24 hours after the childs birth.
B. parents do not perceive boys and girls differently until several weeks or months after the child’s birth.
C. mothers (but not fathers) perceive boys and girls differently but not until several months after the child’s birth.
D. parents no longer perceive boys and girls differently during the first 12 to 24 months after the childs birth.

A

One theory of gender differences in behavior is that they can be traced to the different gender-related perceptions that parents have of their offspring, which affects how boys and girls are treated.

a. CORRECT Based on the results of their study of parents’ perceptions of male and female newborns during the first hours to days following birth, K. H. Karraker, D. A. Vogel, and M. A. Lake conclude that, while gender-related stereotypes have declined in the past few decades, they still exist. For example, they found that parents of newborn girls tend to describe them as less strong, more delicate, and more feminine than do parents of newborn boys (Parents’ gender-stereotyped perceptions of newborns: The eye of the beholder revisited, Sex Roles, 33 (9-10), 667-701, 1995).
b. Incorrect Karraker et al. found that differences in perceptions were apparent within the first few hours following the birth of the child, and this result is consistent with earlier studies.
c. Incorrect While earlier research found that fathers exhibited more stereotypes than mothers, Karraker et al.’s review found no significant differences between fathers and mothers.
d. Incorrect The results of the Karraker et al. study suggest that gender stereotypes have declined but have not entirely disappeared.

The correct answer is: parents perceive boys and girls differently within 24 hours after the childs birth.

150
Q

Research suggests that, in comparison to popular children, less popular children tend to be all of the following except:
Select one:

A. less intelligent.
B. less physically attractive.
C. less friendly and sociable.
D. less cooperative.

A

Not surprisingly, research has shown that characteristics that are valued in the culture are the characteristics most associated with popularity during childhood.

a. Incorrect Unpopular children are generally less intelligent than popular children.
b. Incorrect Unpopular children tend to be less physically attractive than popular children.
c. CORRECT Unpopular children are not necessarily less friendly or sociable than popular children (Ascher and Hymel, 1981; Gottman, 1977).
d. Incorrect Unpopular children do tend to be less cooperative than popular children.

The correct answer is: less friendly and sociable.

151
Q
Researchers interested in the effects of aging on autobiographical memory use the term "reminiscence bump" to describe the tendency of adults aged 65 and older to have the largest number of memories for events that occurred when they were between the ages of:
Select one:
A. 10 and 30.
B. 5 and 15.
C. 25 and 45. 
D. 40 and 55.
A

Older adults typically report the largest number of memories for events that occurred in the ten year period prior to being evaluated. The next largest number of memories is for events that occurred during adolescence and early adulthood.

a. CORRECT Older adults recall recent and remote events more frequently than intermediate events. The term “reminiscence bump” is used to refer to the larger number of memories for events that occurred during adolescence and early adulthood.
b. Incorrect See explanation above.
c. Incorrect See explanation above.
d. Incorrect See explanation above.

152
Q

In a research study, a child sits in front of a three-dimensional model of a mountain scene while another person sits on the opposite side of the mountain scene. The child is then asked to first choose a photograph that depicts the scene that he or she has observed and then to choose the picture that depicts what the other person observed. Being familiar with the research on Piaget’s stages of cognitive development, you predict that children 3 to 5 years of age will most likely:
Select one:

A. pick the correct picture for their own point of view and pick the same (incorrect) picture for the point of view of the other person.
B. pick the correct picture for their own point of view and pick a different (but incorrect) picture for the point of view of the other person.
C. pick the correct pictures for their own point of view and the point of view of the other person.
D. pick incorrect pictures for their own point of view and the point of view of the other person.

A

This question describes a study that is very similar to the one conducted by Piaget, which confirmed that preoperational children are egocentric - i.e., they have trouble understanding another person’s point of view.

a. CORRECT This response is consistent with the results of research conducted by Piaget and others, which found that preoperational children were usually unable to recognize that another person’s view of the mountain differed from their own. (Note, however, that some recent research suggests that young children’s perspective-taking may be affected by the type of task or other variables.)
b. Incorrect See explanation for response a.
c. Incorrect See explanation for response a.
d. Incorrect See explanation for response a.

The correct answer is: pick the correct picture for their own point of view and pick the same (incorrect) picture for the point of view of the other person.

153
Q

A child who has just progressed into Kohlberg’s conventional level of moral development has been caught stealing from other children at school. The child’s parents would be best advised to do which of the following to discourage the child’s stealing?
Select one:

A. Tell her that she will receive a present at the end of each week that she does not steal.
B. Help her understand how the children she steals from feel about having their things taken.
C. Tell her that stealing is against the law and morally wrong.
D. Have her personally return the stolen items to their owners.

A

Answer B is correct. Stage 3 is the first stage in Kohlberg’s conventional level of moral development. It is referred to as the “Good Boy/Good Girl” stage and is characterized by a strong belief in the Golden Rule. Children in this stage are cognitively able to put themselves in the place of others and, therefore, consider the feelings of others when making moral judgments. Consequently, this approach would be most effective with a child who is in this stage.

a. Incorrect - This approach would probably be most effective for a child with pre-conventional morality.
c. Incorrect - Law and authority (“right” and “wrong”) and morally based judgments are for children in post-conventional morality.
d. Incorrect - Although this approach may be beneficial, it is not addressed by Kohlberg’s theory.

The correct answer is: Help her understand how the children she steals from feel about having their things taken.

154
Q

A young child exhibiting an insecure-disorganized attachment pattern reacts to the return of his/her mother in the Strange Situation with disorganized, conflicted, and, in some cases, self-injurious behaviors. This attachment pattern has been linked to parental abuse and neglect and, more recently, with:
Select one:

A. elevated levels of cortisol in the child during the events of the Strange Situation.
B. lower-than-normal levels of cortisol in the child during the events of the Strange Situation.
C. elevated levels of cortisol in the child only when his/her mother returns during the Strange Situation.
D. elevated levels of cortisol in the mother (but not the child) during the events of the Strange Situation.

A

Research has linked attachment patterns to different levels of neurohormones and brain activity.

a. CORRECT The levels of cortisol, a stress hormone released by the adrenal gland, differ for children with secure and insecure attachment to their primary caregivers. Specifically, while securely attached children show an increase in cortisol levels when the caregiver leaves the Strange Situation, the level of cortisol decreases when the caregiver returns. In contrast, for children exhibiting insecure-disorganized attachment, cortisol levels remain elevated throughout the course of the Strange Situation. See, e.g., M. Main and J. Solomon, Procedures for identifying infants as disorganized/disoriented during the Ainsworth Strange Situation, in M. T. Greenberg, D. Cicchetti, and E. M. Cummings (Eds.), Attachment in the preschool years: Theory, research and intervention (pp. 121-160), Chicago, University of Chicago Press, 1990.
b. Incorrect See explanation above.
c. Incorrect See explanation above.
d. Incorrect See explanation above.

The correct answer is: elevated levels of cortisol in the child during the events of the Strange Situation.

155
Q

Jerome Kagan has related the basic temperament quality of inhibition (degree of approach or withdrawal in new situations) to which of the following?
Select one:

A. prenatal development
B. intellectual capacity
C. caregiver behaviors
D. CNS reactivity

A

Although other investigators have identified several basic temperament qualities, Kagan focuses on behavioral inhibition and describes it as the key determinant of later personality.

a. Incorrect See explanation for response d.
b. Incorrect See explanation for response d.
c. Incorrect See explanation for response d.
d. CORRECT Infants with a high degree of behavioral inhibition also exhibit high CNS activity (especially in the amygdala and hypothalamus) in new situations. See J. Kagan et al., The temperamental quality of inhibition and lack of inhibition, in M. Lewis and S. M. Miller (Eds.), Handbook of developmental psychopathology, New York, Plenum Press, 1990, pages 219-226.

The correct answer is: CNS reactivity

156
Q

Research investigating the outcomes of parenting style for adolescents has found that the strength of the relationship between authoritative parenting and adolescent scholastic achievement is moderated most by which of the following variables?
Select one:

A. gender
B. ethnicity
C. age
D. socioeconomic status

A

This issue was investigated by L. Steinberg and his colleagues in their study of the outcomes of authoritative parenting (Impact of parenting practices on adolescent achievement: Authoritative parenting, school involvement, and encouragement to succeed, Child Development, 63, 1266-1281, 1992).

a. Incorrect See explanation for response b.
b. CORRECT Research by Steinberg et al. found that an authoritative parenting style is predictive of successful school performance in adolescence. However, while predictive accuracy remains the same regardless of gender, socioeconomic status, or age (younger versus older adolescent), the effects of authoritativeness are moderated by culture/ethnicity. Specifically, the relationship between authoritative parenting and academic achievement is much weaker for African-American adolescents, apparently because they are more strongly influenced by their peers than by their parents.
c. Incorrect See explanation for response b.
d. Incorrect See explanation for response b.

The correct answer is: ethnicity

157
Q

A 47-year old politician teaching his aides the essentials of dirty tricks tells them, “Nothing is wrong unless you get caught.” One of his aides, a developmental psychologist, comments to another aide that the politician is apparently at the __________ level of moral development.
Select one:

A. preconventional
B. conventional
C. postconventional
D. instrumental

A

Kohlberg’s levels of moral development are preconventional morality (punishment-obedience orientation and instrumental-relativist orientation), conventional morality (good boy/nice girl orientation and law and order orientation), and postconventional morality (legalistic orientation and universal ethical principles orientation).

a. CORRECT Individuals at the punishment-obedience level of preconventional morality believe an action is moral to the degree that it does not result in punishment. Although this stage is characteristic of children aged 4 to 10, it seems that this politician is also at this level.
b. Incorrect Conventional morality, which is typical of individuals aged 10-13, is characterized by moral judgments motivated by a desire to be liked (good boy/nice girl orientation) or a desire to avoid censure by authority (law and order orientation).
c. Incorrect Postconventional morality, a level which is not reached by most adults, is characterized by a concern for maintaining the social order (legalistic orientation) or meeting the criteria set by one’s own conscience (universal Ethical Principles orientation).
d. Incorrect The instrumental-relativist stage, in which moral judgments are based on a desire to satisfy one’s own needs, is the second stage of the preconventional level.

The correct answer is: preconventional

158
Q

An infant with secure attachment will most likely respond to a stranger in the Strange Situation in which of the following ways?
Select one:

A. The infant will be friendly toward the stranger when mother is present but wary of the stranger when mother is absent.
B. The infant will be wary of the stranger when mother is present but friendly toward the stranger when mother is absent.
C. The infant will be friendly toward the stranger when mother is either present or absent.
D. The infant will be wary of the stranger when mother is either present or absent.

A

The Strange Situation was developed by Ainsworth and her colleagues (1978) to evaluate attachment in infants and consists of several events including the mother being alone with her infant, the mother and a stranger being with the infant, and the infant being alone with the stranger.

a. CORRECT Ainsworth and her colleagues found that securely attached infants were often friendly toward the stranger when mother was present but tended to be wary of the stranger when mother was absent, although some of these infants were somewhat comforted by the overtures of the stranger when separated from their mothers.
b. Incorrect This answer does not describe the behavior associated with any of the three attachment patterns initially identified by Ainsworth.
c. Incorrect In the Strange Situation, most infants with insecure-avoidant attachment reacted to the stranger in much the same way that they reacted to their mother and some were friendlier toward the stranger than their mother.
d. Incorrect Infants with insecure-ambivalent attachment were wary of the stranger when mother was either present or absent and were not calmed by the stranger when separated from their mothers.

The correct answer is: The infant will be friendly toward the stranger when mother is present but wary of the stranger when mother is absent.

159
Q
Most children are not physically ready for toilet training until which age?
Select one:
A. 9 to 12 months
B. 12 to 18 months
C. 20 to 24 months 
D. 30 to 34 months
A

If you’ve had any experience with young children, you may have been able to answer this question. Note that it’s asking about MOST children.

a. Incorrect See explanation for response c.
b. Incorrect See explanation for response c.
c. CORRECT The experts seem to vary somewhat with regard to this issue, but most consider 20 to 24 months a good time to begin toilet training.
d. Incorrect See explanation for response c.

The correct answer is: 20 to 24 months

160
Q

One of the most common impairments following brain injury is memory loss. Although the nature of the disturbance depends on the locus and extent of the damage, most often it involves:
Select one:

A. an inability to recall information stored in remote long-term memory.
B. profound deficits in memory for events that occurred in the days to weeks prior to the incident that caused the injury.
C. spotty deficits in memory for both recent and past events.
D. problems related to retaining new information.

A

Not surprisingly, brain injury typically affects memory, at least temporarily.

a. Incorrect See explanation for response d.
b. Incorrect See explanation for response d.
c. Incorrect See explanation for response d.
d. CORRECT The exact nature of the memory disturbance depends on the location and severity of the injury. However, new learning is most commonly impaired, and the severity of the injury is often measured by the length of this post-traumatic (anterograde) amnesia. Additional information on memory loss following head trauma is provided in the Physiological Psychology and Psychopharmacology chapter of the written study materials.

The correct answer is: problems related to retaining new information.

161
Q

A person with damage to the hippocampus and adjacent areas in the temporal lobes will most likely demonstrate which of the following?
Select one:

A. inability to form long-term memories about facts and events
B. impairments in working memory
C. inability to acquire a classically-conditioned response
D. loss of memory for remote personal events

A

The hippocampus and adjacent areas in the temporal lobe are involved in memory.

a. CORRECT The effects of lesions in these areas of the brain were demonstrated in the case of H.M. who exhibited profound impairments in the ability to consolidate declarative memories (i.e., following ablation of these areas to control H.M.’s epilepsy, he was unable to form new memories of facts and events).
b. Incorrect See explanation above.
c. Incorrect See explanation above.
d. Incorrect See explanation above.

The correct answer is: inability to form long-term memories about facts and events

162
Q

Tacrine (Cognex), donepezil (Aricept), and galantamine (Reminyl) exert their beneficial effects on memory for patients with Alzheimer’s dementia by:
Select one:

A. slowing acetylcholine depletion.
B. slowing dopamine depletion.
C. increasing the production of serotonin.
D. increasing the production of norepinephrine.

A

Memory loss associated with Alzheimer’s dementia has been linked to lower-than-normal levels of acetylcholine (ACh).

a. CORRECT The drugs listed in the question are cholinesterase inhibitors, which means that they inhibit enzymes that break down unused ACh and thereby increase levels of this neurotransmitter in the brain.
b. Incorrect See explanation above.
c. Incorrect See explanation above.
d. Incorrect See explanation above.

The correct answer is: slowing acetylcholine depletion.

163
Q

The onset of puberty in humans occurs when certain cells in the __________ secrete gonadotropin-releasing hormones.
Select one:

A. tectum
B. thalamus
C. hippocampus
D. hypothalamus

A

At puberty, the gonads (testes and ovaries) produce their hormones, which, in turn, are responsible for physical sexual maturation.

a. Incorrect See explanation for response d.
b. Incorrect See explanation for response d.
c. Incorrect See explanation for response d.
d. CORRECT Secretion of the gonadotropin-releasing hormones by the hypothalamus stimulates production and release of the gonadotropic hormones by the pituitary gland, which then stimulate the gonads to release the sex hormones.

The correct answer is: hypothalamus

164
Q

Following a stroke, a 71-year-old woman exhibits loss of vision in her right visual field. Most likely, this impairment is the result of damage to the:
Select one:

A. visual cortex in her left hemisphere.
B. visual cortex in her right hemisphere.
C. left posterior temporal lobe. Incorrect
D. right posterior temporal lobe.

A

Not surprisingly, vision is mediated by the visual cortex.

a. CORRECT Most of the body’s functions–including vision–are mediated by the opposite side of the brain. Therefore, loss of vision in the right visual field would be caused by damage in the left hemisphere.
b. Incorrect See explanation for response a.
c. Incorrect See explanation for response a.
d. Incorrect See explanation for response a.

The correct answer is: visual cortex in her left hemisphere.

165
Q

Your new client, Annabelle A., reports that she has felt fidgety and anxious and has been unable to sleep for the past few weeks. She also complains of heart palpitations, weight loss, and a reduced ability to pay attention to her schoolwork. Despite the cold weather, she is wearing a cotton dress without a coat or sweater. Annabelle’s symptoms are most suggestive of which of the following?
Select one:

A. hypomania
B. Raynaud’s disease
C. hyperthyroidism
D. hypoglycemia

A

Hyperthyroidism is characterized by speeded-up metabolism, elevated body temperature, weight loss, general excitability, nervousness, insomnia, and accelerated heart rate.

a. Incorrect A person with hypomania does display excitability and nervousness but does not exhibit such physical signs as increased body temperature and heart rate.
b. Incorrect This alternative is clearly a distractor. Raynaud’s disease involves restricted blood flow to the fingers or toes and is characterized by pallor, paresthesias, and pain.
c. CORRECT The client’s symptoms are characteristic of hyperthyroidism which is caused by an overactive thyroid gland.
d. Incorrect Hypoglycemia is characterized by hunger, dizziness, and depression.

The correct answer is: hyperthyroidism

166
Q

Which of the following is least likely to be an initial symptom of multiple sclerosis?
Select one:

A. blurred or double vision
B. fatigue that worsens in the afternoon
C. impaired memory and attention
D. heaviness, weakness, or loss of sensation in the legs

A

Multiple sclerosis (MS) is caused by demyelination of nerve fibers in the central nervous system. Although its symptoms vary from individual to individual, the progression of symptoms usually follows a predictable pattern.

a. Incorrect Visual problems are common initial symptoms of MS.
b. Incorrect Fatigue (especially fatigue that worsens in the afternoon) is a common initial symptom of this disorder.
c. CORRECT It is estimated that up to 70% of people with MS develop some type of cognitive dysfunction, and this most often involves impairments in attention, recent memory, and information processing speed. While some of these individuals experience cognitive problems in the early stages of the disorder, the majority do not do so until they have had the disease for an extended period. Therefore, of the answers given, this one is the best response.
d. Incorrect Sensory and motor problems are initial signs of MS and most often affect the legs.

The correct answer is: impaired memory and attention

167
Q

Neuroleptic malignant syndrome is characterized by:
Select one:

A. ataxia, cardiac arrhythmia, and delirium.
B. headache, tremor, and confusion.
C. nausea and vomiting, loss of coordination, and seizures.
D. muscle rigidity, hyperthermia, and stupor.

A

Neuroleptic malignant syndrome (NMS) is a rare, idiosyncratic reaction to neuroleptic drugs that may be caused by dopamine blockage in the basal ganglia.

a. Incorrect These are symptoms of tricyclic overdose.
b. Incorrect These symptoms are characteristic of 5-HT toxicity syndrome.
c. Incorrect These symptoms suggest lithium toxicity.
d. CORRECT These are the core symptoms of NMS. Other symptoms include painful joints, mutism, tachycardia, and urinary retention.

The correct answer is: muscle rigidity, hyperthermia, and stupor.

168
Q

Combining an MAOI with certain cold, cough, sinus, or allergy medication can produce which of the following?
Select one:

A. neuroleptic malignant syndrome
B. a hypertensive crisis
C. agranulocytosis
D. tardive dyskinesia

A

Combining an MAOI with certain other drugs or certain foods can produce potentially fatal side effects.

a. Incorrect See explanation for response b.
b. CORRECT An MAOI in combination with certain drugs can produce a hypertensive crisis (noradrenergic syndrome), which involves a severe headache, diaphoresis, elevated blood pressure, neck stiffness, and neuromuscular agitation; or a hyperpyrexic crisis (serotonin syndrome), which is characterized by hyperthermia, tachycardia, nausea, shivering, restlessness, confusion, and insomnia.
c. Incorrect See explanation for response b.
d. Incorrect See explanation for response b.

The correct answer is: a hypertensive crisis

169
Q

Synesthesia is best conceptualized as:
Select one:

A. a learned phenomenon.
B. the result of imagination.
C. a physical process in the brain.
D. a manifestation of psychosis.

A

Synesthesia occurs when two or more senses are automatically and involuntarily joined so that stimulation of one sense automatically elicits another (e.g., the sound of a voice elicits a specific color).

a. Incorrect See explanation for response c.
b. Incorrect See explanation for response c.
c. CORRECT Although the cause of synesthesia is not clear, it is believed to be due to brain functioning. One theory attributes it to unusually dense connections between sensory regions of the brain.
d. Incorrect See explanation for response c.

The correct answer is: a physical process in the brain.

170
Q

Tactile sensation and visuospatial functions are mediated primarily by the:
Select one:

A. frontal lobe.
B. occipital lobe.
C. parietal lobe.
D. temporal lobe.

A

For the licensing exam, you want to be familiar with the primary functions of each lobe of the cerebral cortex. These are described in the Physiological Psychology and Pharmacology chapter of the written study materials.

a. Incorrect The frontal lobe mediates motor functions, language production, personality, and executive cognitive functions.
b. Incorrect The occipital lobe is responsible for vision and visual perception.
c. CORRECT The parietal lobe receives tactile input and mediates visuospatial functions, reading, and calculation.
d. Incorrect The temporal lobe is responsible for audition, language comprehension, and memory.

The correct answer is: parietal lobe.

171
Q

The anterior cingulate cortex is believed to be involved in:
Select one:

A. color vision.
B. spatial memory.
C. emotional reactions to pain.
D. the physiological component of anxiety.

A

The anterior cingulate cortex is located in the limbic cortex on the medial side of the cerebral hemispheres.

a. Incorrect See explanation for response c.
b. Incorrect See explanation for response c.
c. CORRECT A number of studies have confirmed that exposure to painful stimuli activates the anterior cingulate cortex. However, this structure does not appear to be involved in the perception of pain but, instead, in the emotional reaction to painful stimuli.
d. Incorrect See explanation for response c.

The correct answer is: emotional reactions to pain.

172
Q

As defined in Lazarus’s (1991) cognitive appraisal theory, __________ refers to a person’s evaluation of the resources he or she has to cope with a situation that has been identified as stressful.
Select one:

A. primary appraisal
B. secondary appraisal
C. tertiary appraisal
D. re-appraisal

A

Cognitive appraisal theory (Lazarus, 1991) predicts that cognitive appraisal is an essential component of emotional experience and distinguishes between three types of appraisal: primary appraisal, secondary appraisal, and re-appraisal.

a. Incorrect Primary appraisal refers to a person’s evaluation of the situation as positive, stressful, or irrelevant to his/her well-being.
b. CORRECT Secondary appraisal refers to a person’s appraisal of the resources available to cope with a stressful situation (e.g., social support, material resources, level of energy).
c. Incorrect Tertiary appraisal is not part of Lazarus’s theory.
d. Incorrect Re-appraisal occurs when the person monitors the situation and, when necessary, changes his/her primary and/or secondary appraisal.

The correct answer is: secondary appraisal

173
Q

Prolonged exposure to stress may lead to chronic __________ by the adrenal glands.
Select one:

A. suppression of cortisol release
B. secretion of cortisol
C. suppression of oxytocin release
D. secretion of oxytocin

A

Knowing that cortisol is secreted by the adrenal cortex would have helped you answer this question.

a. Incorrect See explanation for response b.
b. CORRECT Cortisol and the other corticosteroids are collectively known as the “stress hormones.” Chronic stress leads to chronic secretion of these hormones, which can cause a number of problems including increased susceptibility to infection.
c. Incorrect Oxytocin is secreted by the pituitary gland and, in females, controls uterine contractions and milk production.
d. Incorrect See explanation above.

The correct answer is: secretion of cortisol

174
Q

Up to 50% or more of individuals who experience a minor brain injury develop postconcussional syndrome. The majority of these individuals fully recover within __________. However, for those whose symptoms last for more than __________, symptoms may be permanent.
Select one:

A. one to three weeks; six months
B. one to two months; nine months
C. one to three months; one year
D. three to six months; two years

A

Postconcussional syndrome is a pattern of somatic and psychological symptoms that occur in up to 50% or more of individuals who experience a minor brain injury.

a. Incorrect See explanation for answer c.
b. Incorrect See explanation for answer c.
c. CORRECT Most individuals who develop minor symptoms following a brain injury fully recover within one to three months after the injury. A minority of individuals continues to experience symptoms for more than one year and, for these individuals, symptoms may be lifelong. See, e.g., E. L. Legome, R. Alt, and T. Wu, May 13, 2013, Postconcussive syndrome. Retrieved from http://emedicine.medscape.com/article/828904-overview.

d. Incorrect See explanation for answer c.
The correct answer is: one to three months; one year

175
Q

Data from the National Health Care Survey indicate that, in general, central nervous system medications are most often prescribed for patients in which of the following ambulatory care settings?
Select one:

A. primary care offices
B. medical specialty offices
C. hospital outpatient departments
D. hospital emergency departments

A

Data on ambulatory medical care visits in the United States is collected yearly by the National Center on Health Statistics and released in several reports [S. Raofi and S. M. Schappert, Medication therapy in ambulatory medical care: United States, 2003-04, Vital Health Stat, 13(163), 2006].

a. CORRECT The surveys have consistently shown that, overall, central nervous system medications are most frequently provided, prescribed, or continued at ambulatory care visits in primary care offices, followed by medical specialty offices. There are a few exceptions to this general conclusion for specific types of CNS medications, however - e.g., antipsychotics and antimanics are more often provided or prescribed in medical specialty offices, which include the offices of psychiatrists and neurologists.
b. Incorrect See explanation for response a.
c. Incorrect See explanation for response a.
d. Incorrect See explanation for response a.

The correct answer is: primary care offices

176
Q

Which of the following is a type of nonfluent aphasia that involves deficits in both receptive and expressive language?
Select one:

A. conduction aphasia
B. global aphasia
C. anomic aphasia
D. transcortical motor aphasia

A

The various types of aphasia are sometimes categorized in terms of fluency of speech. A person with nonfluent aphasia has slow, labored speech, while a person with fluent aphasia has well-articulated fluent speech that is devoid of meaning.

a. Incorrect Conduction aphasia is a type of fluent aphasia that involves phonemic paraphasias (substitutions of one phoneme for another) and impaired speech repetition with normal auditory comprehension.
b. CORRECT Global aphasia is a type of nonfluent aphasia that involves deficits in comprehension, severe anomia (inability to recall the names of familiar objects), and impaired speech repetition.
c. Incorrect Anomic aphasia is a type of fluent aphasia that is characterized by anomia with normal comprehension and speech repetition.
d. Incorrect Transcortical motor aphasia is a type of nonfluent aphasia with normal auditory comprehension and speech repetition.

The correct answer is: global aphasia

177
Q

Central achromatopsia is caused by bilateral lesions in the occipitotemporal region of the brain and is characterized by an inability to:
Select one:

A. name colors.
B. distinguish between different hues.
C. associate a color with an object.
D. perceive variations in brightness.

A

A number of visual impairments are associated with damage to the occipital lobe.

a. Incorrect The inability to name colors despite an intact ability to discriminate between them is referred to color anomia.
b. CORRECT For people with central achromatopsia, everything looks grey or “washed out.”
c. Incorrect The inability to associate a color with an object is referred to as color agnosia. A person with this disorder would, for example, consider a pink hippopotamus to be plausible.
d. Incorrect Achromatopsia does not involve an inability to perceive variations in brightness.

The correct answer is: distinguish between different hues.

178
Q

A young six-year-old boy is having severe behavioral problems. He is physically aggressive, shouting and misbehaving constantly. There is no prior history of the child having any kind of behavioral problems and there has been no trauma in the child’s past. Of the following, which is the most likely cause of the boy’s behavior?
Select one:

A. The passing on of certain genes from both parents causing this behavior in their son
B. A brain tumor growing that is pushing on various parts of the brain which in turn is affecting the child’s behavior
C. The child could be experiencing developmental difficulties that are causing him to become frustrated and lash out at those around him
D. None of the above

A

The correct answer is B. Of the four possible answers B is most likely.

Answer A is not likely, as it is difficult to demonstrate cause and effect with genetics regarding behavior on a consistent basis.

Answer C is incorrect because developmental is too broad of a descriptor, and it could be referring to something like autism spectrum disorder or simply growing pains.

The correct answer is: A brain tumor growing that is pushing on various parts of the brain which in turn is affecting the child’s behavior

179
Q

Hemiplegia affecting the right side of the body suggests damage to the:
Select one:

A. motor cortex in the left hemisphere.
B. motor cortex in the right hemisphere.
C. somatosensory cortex in the left hemisphere.
D. somatosensory cortex in the right hemisphere.

A

To answer this question, you have to know that hemiplegia refers to partial or complete paralysis on one side of the body and that the right side of the body is controlled by the left side of the brain.

a. CORRECT Damage to the motor cortex could result in paralysis. Since the paralysis is on the right side of the body, this implies that the damage is to the motor cortex in the left hemisphere.
b. Incorrect See explanation above.
c. Incorrect See explanation above.
d. Incorrect See explanation above.

The correct answer is: motor cortex in the left hemisphere.

180
Q

Which of the following is true about the prescription of anxiolytic drugs for older adults?
Select one:

A. A lower-than-normal dose is usually indicated because, in older adults, these drugs have a shorter half-life.
B. A lower-than-normal dose is usually indicated because, in older adults, these drugs have a longer half-life.
C. A higher-than-normal dose is usually indicated because, in older adults, these drugs have a shorter half-life.
D. A higher-than-normal dose is usually indicated because, in older adults, these drugs have a longer half-life.

A

In the context of psychopharmacology, half-life refers to the time it takes for the body to eliminate half of the given dose of a drug (i.e., for plasma levels to drop by 50%). For a number of drugs, increasing age increases the half-life of the drug because of an age-related decrease in metabolism or reduction in renal functioning.

a. Incorrect See explanation for response b.
b. CORRECT Because the half-life of the anxiolytic drugs increases with increasing age, these drugs must be administered in lower doses to older adults to avoid toxicity. In addition, anxiolytics with a long half-life (e.g., diazepam, flurazepam) should usually be avoided.
c. Incorrect See explanation for response b.
d. Incorrect See explanation for response b.

The correct answer is: A lower-than-normal dose is usually indicated because, in older adults, these drugs have a longer half-life.

181
Q

Research on the etiology of Tourette’s syndrome has linked it to excessive activity of dopamine receptors in the:
Select one:

A. amygdala.
B. septum.
C. medulla oblongata.
D. caudate nucleus.

A

Tourette’s syndrome involves involuntary tics and, not surprisingly, the area of the brain associated with automatic movements has been implicated in this disorder.

a. Incorrect See explanation for response d.
b. Incorrect See explanation for response d.
c. Incorrect See explanation for response d.
d. CORRECT The caudate nucleus mediates automatic movement, and supersensitivity of dopamine receptors in this structure has been linked to Tourette’s syndrome.

The correct answer is: caudate nucleus.

182
Q

Which of the following conditions is not accurately described?
Select one:

A. ALS is a progressive peripheral nerve disorder that causes muscle weakness, tremor, and spasticity.
B. Myasthenia gravis is a neuromuscular autoimmune disorder that produces weakness in the eye and facial muscles, difficulty swallowing, and slurred speech.
C. Multiple sclerosis is a musculoskeletal syndrome that produces muscular pain, stiffness, and spasms and is often accompanied by fatigue, gastrointestinal disorders, and depression.
D. Huntingtons disease is a hereditary disease that causes involuntary jerky movements, impaired memory and judgment, and depression.

A

The conditions described in the answers to this question are neurological disorders that cause impairment in sensory and/or motor functioning. Note that the question is asking which condition is not accurately described.
a. Incorrect This answer correctly describes ALS (amyotrophic lateral sclerosis), which is also known as Lou Gehrig’s disease and is a degenerative disease that eventually leads to loss of muscle control and paralysis.

b. Incorrect The term myasthenia gravis means “grave muscle weakness.” This disorder can affect any of the body’s muscles, but the muscles that control the eyes, facial expression, chewing, and swallowing are ordinarily most affected.
c. CORRECT The symptoms presented in this answer describe fibromyalgia rather than multiple sclerosis (MS). MS is caused by demyelination of nerve fibers in the brain and spinal cord and is believed to be due to an abnormal immune system response. Its primary symptoms include impairments in sensory and motor functioning (e.g., paresthesias, shooting pain in the back and limbs, problems with balance, and muscle weakness); deficits in memory, reasoning, and concentration; sexual dysfunction; and vision abnormalities.
d. Incorrect This answer accurately describes Huntington’s disease, which produces a combination of motor, cognitive, and psychiatric symptoms.

The correct answer is: Multiple sclerosis is a musculoskeletal syndrome that produces muscular pain, stiffness, and spasms and is often accompanied by fatigue, gastrointestinal disorders, and depression.

183
Q

Research investigating the physiological mechanisms associated with emotion have found that the eyeblink reflex is:
Select one:

A. potentiated when the individual is viewing an unpleasant stimulus but inhibited when the individual is viewing a pleasant stimulus.
B. inhibited when the individual is viewing an unpleasant stimulus but potentiated when the individual is viewing a pleasant stimulus.
C. potentiated when the individual is viewing a neutral stimulus but inhibited when the individual is viewing an unpleasant or pleasant stimulus.
D. unaffected by the type of stimulus being viewed by an individual.

A

The eyeblink reflex has been used to investigate the role of central and peripheral factors in a number of phenomena including motivational and emotional states and certain disorders (e.g., Panic Disorder, PTSD, and Schizophrenia).

a. CORRECT The research has confirmed that the startle reflex is affected by an individual’s emotional state; and, more specifically, that a negative emotional state is associated with an enhanced startle reflex, while a positive emotional state is associated with a reduced startle reflex. See, e.g., M. M. Bradley, B. N. Cuthbert, and P. J. Lang, Pictures as prepulses: Attention and emotion in startle modification, Psychophysiology, 30, 541-545, 1993.
b. Incorrect See explanation for response a.
c. Incorrect See explanation for response a.
d. Incorrect See explanation for response a.

The correct answer is: potentiated when the individual is viewing an unpleasant stimulus but inhibited when the individual is viewing a pleasant stimulus.

184
Q

The left hemisphere of the cerebral cortex is dominant for speech and language functions:
Select one:
A. for most left-handers but few right-handers.
B. for nearly all left-handers and many right-handers.
C. for nearly all right-handers and the majority of left-handers.
D. for nearly all right-handers but a small minority of left-handers.

A

The left hemisphere is dominant for language for most people, regardless of handedness.

a. Incorrect See explanation for response c.
b. Incorrect See explanation for response c.
c. CORRECT For most right-handers and the majority of left-handers, the left hemisphere is dominant for language. While some left-handers have right hemisphere dominance, others show a lack of dominance of either hemisphere.
d. Incorrect See explanation for response c.

The correct answer is: for nearly all right-handers and the majority of left-handers.

185
Q

Intelligence (IQ) tests are to brainstorming and other measures of creativity as:
Select one:

A. convergent thinking is to divergent thinking.
B. divergent thinking is to convergent thinking.
C. disjunctive thinking is to conjunctive thinking.
D. conjunctive thinking is to disjunctive thinking.

A

Intelligence tests require an examinee to produce the correct response, while brainstorming and other measures of creativity require the examinee to come up with many ideas or solutions.

a. CORRECT Tasks that require an examinee to derive a single correct answer are assessing convergent thinking, while tasks that require the examinee to generate as many ideas as possible are evaluating divergent thinking.
b. Incorrect See explanation for response b.
c. Incorrect Disjunctive (declarative) and conjunctive (cumulative) are two of the mental processes identified by E. Jaques’s (1994) theory about intelligence in the context of the management of work. (Note that you’re not likely to encounter Jaques’s theory on the licensing exam and these terms were included here as “distractors” only.)
d. Incorrect See explanation for response c.

The correct answer is: convergent thinking is to divergent thinking.

186
Q

If a researcher wanted to collect data quickly, within a modest budget and from a large group of people, which approach would be most appropriate?
Select one:

A. Multi-informant reports
B. Semi-structured interviews
C. Psychophysiological measures
D. Self-report methods

A

The correct answer is D. This information may include: personality traits, moods, thoughts, attitudes, preferences, and behaviors. The main advantages of self-report methods are quick production and scoring, and low cost. Answers A, B, and C are incorrect as they tend to be more time consuming and expensive than self-report methods.

The correct answer is: Self-report methods

187
Q

Raven Colored Progressive Matrices would be useful for:
Select one:

A. assessing the intelligence of a 9-year-old non-English speaking child.
B. assessing the intelligence of an 18-year-old who has a learning disability.
C. assessing the memory of a 74-year-old with early signs of dementia.
D. assessing the intelligence of a 45-year-old who has Broca’s aphasia.

A

Raven Colored Progressive Matrices (RCPM) is a 36-item test of intelligence that uses figural test stimuli presented in a 6-option multiple-choice format.

a. CORRECT Because it is a nonverbal test, the RCPM is considered appropriate for non-English speaking individuals as well as individuals with speech deficits. The test is designed for children ages 5 through 11 years.
b. Incorrect See explanation for response a.
c. Incorrect See explanation for response a.
d. Incorrect See explanation for response a.

The correct answer is: assessing the intelligence of a 9-year-old non-English speaking child.

188
Q

Zajonc’s confluence model predicts that children’s intellectual growth can be either enhanced or hindered by their immediate family circumstances. Specifically, Zajonc considers __________ to be a critical factor in intellectual development.
Select one:

A. birth order
B. parental marital status
C. parenting style
D. culture

A

Zajonc argues that intellectual development is influenced by the intellectual environment of the home, especially as it relates to family configuration.

a. CORRECT Zajonc’s confluence model predicts that children born earlier (e.g., firstborns) tend to do slightly better on IQ and academic tests than their younger siblings due to differences in the family environment. See the Psychological Assessment chapter for additional information on the confluence model.
b. Incorrect See explanation for response a.
c. Incorrect See explanation for response a.
d. Incorrect See explanation for response a.

The correct answer is: birth order

189
Q

Scores on the Symbol Search, Coding, and Cancellation subtests are used to derive a score on which WAIS-IV Index?
Select one:

A. Working Memory
B. Perceptual Reasoning
C. Verbal Comprehension
D. Processing Speed

A

Answer D is correct. The WAIS-IV provides scores on the four Indexes listed in the answers to this question. The Processing Speed Index consists of two core subtests (Symbol Search and Coding) and one supplemental subtest (Cancellation).

Answer A: The Working Memory Index consists of two core subtests (Digit Span and Arithmetic) and one supplemental subtest (Letter-Number Sequencing).

Answer B: The Perceptual Reasoning Index consists of three core subtests (Block Design, Matrix Reasoning, and Visual Puzzles) and two supplemental subtests (Figure Weights and Picture Completion).

Answer C: The Verbal Comprehension Index consists of three core subtests (Vocabulary, Similarities, and Information) and one supplemental subtest (Comprehension).

The correct answer is: Processing Speed

190
Q

Examinees with Mild Cognitive Impairment, Major Depression, or Traumatic Brain Injury are likely to achieve the highest score on which of the following WAIS-IV Indexes?
Select one:

A. Perceptual Reasoning
B. Processing Speed
C. Verbal Comprehension
D. Working Memory

A

Answer C is correct: The WAIS-IV Technical Manual reports that, of the four Indexes, examinees with Mild Cognitive Impairment, Major Depression, or Traumatic Brain Injury received the highest score on the Verbal Comprehension Index.

The correct answer is: Verbal Comprehension

191
Q

Which of the following approaches was used to derive the “Big Five” personality traits?
Select one:

A. theoretical
B. empirical criterion keying
C. lexical
D. multitrait-multimethod matrix

A

The Big Five personality traits - i.e., conscientiousness, extroversion/introversion, openness to experience, emotional stability, and agreeableness - were originally identified through factor analyses of personality traits included in the dictionary.

a. Incorrect See explanation for response c.
b. Incorrect See explanation for response c.
c. CORRECT The lexical approach is atheoretical and is based on the assumption that all socially-relevant personality traits have been incorporated into language. For the Big Five, this initially involved identifing the personality traits listed in the dictionary.
d. Incorrect See explanation for response c.

The correct answer is: lexical

192
Q

On the MMPI-2, an examinee whose highest clinical scale scores are on Scales 4 and 9 is most likely to exhibit:
Select one:

A. apathy, depression, and psychosomatic complaints.
B. egocentrism and grandiosity.
C. depression and hypersensitivity to criticism.
D. antisocial behavior and impulsivity.

A

Knowing the names of Clinical Scales 4 and 9 may have helped you identify the correct answer to this question: Scale 4 is the Psychopathic Deviate scale, and Scale 9 is the Hypomania scale.

a. Incorrect Apathy, depression, and psychosomatic complaints are characteristic of several codes including 23/32. (Scale 2 is the Depression Scale and Scale 3 is the Hysteria scale.)
b. Incorrect Egocentrism and grandiosity are associated with the 90/09 code. (Scale 9 is the Hypomania scale and 0 is the Social Introversion scale.)
c. Incorrect Depression and hypersensitivity to criticism are associated with elevations on several scales include Scale 2 (Depression) and Scale 6 (Paranoia).
d. CORRECT The 49/94 code (highest scores on Scales 4 and 9) is associated with antisocial behaviors, sex offenses, impulsivity, and alcohol and drug abuse.

The correct answer is: antisocial behavior and impulsivity.

193
Q

The Wide-Range Achievement Test is:
Select one:

A. a brief achievement test that measures reading comprehension, spelling, and arithmetic computation.
B. a brief achievement test that measures the basic content domains (e.g., English, science, math) taught in elementary and junior high school.
C. a measure of academic achievement that is used to predict high school grades.
D. a measure of academic achievement that is used to predict undergraduate grades.

A

The Wide-Range Achievement Test (WRAT) is a brief achievement test that can be administered to children, adolescents, and adults. The most recent version is the WRAT-4, which was published in 2006.

a. CORRECT The WRAT assesses achievement in reading, spelling, and arithmetic only.
b. Incorrect See explanation above.
c. Incorrect See explanation above.
d. Incorrect See explanation above.

The correct answer is: a brief achievement test that measures reading comprehension, spelling, and arithmetic computation.

194
Q

Kahneman and Tversky’s (1979) notion of __________ predicts a tendency for a loss of a particular magnitude to seem more aversive to an individual than a gain of the same magnitude seems attractive.
Select one:

A. status quo bias
B. psychological reactance
C. loss aversion
D. gain/loss theory

A

Kahneman and Tversky’s notion of loss aversion is part of their cumulative prospect theory and refers to the tendency to weigh losses more heavily than gains.

a. Incorrect The status quo bias refers to the tendency to maintain the status quo because the disadvantages of changing are perceived to be greater than the advantages.
b. Incorrect Psychological reactance occurs when an attempt at social control causes a person to feel a loss of freedom and, in response, reacts in a way that is the opposite of what is requested or desired.
c. CORRECT According to Kahneman and Tversky’s notion of loss aversion, people experience losses more intensely than gains of the same magnitude and, consequently, are unlikely to take risks. See D. Kahneman & A. Tversky, Prospect theory: An analysis of decisions under risk, Econometrika, 47, 263-291, 1979.
d. Incorrect Gain/loss theory predicts that a complement will be valued more by a person when it comes from a critic than from someone who has bestowed compliments in the past.

The correct answer is: loss aversion

195
Q

A listener is most likely to process a persuasive message peripherally (i.e., use the “peripheral route”) if she is:
Select one:

A. bored.
B. in a good mood.
C. in a bad mood.
D. emotionally unstable.

A

This question is asking about the elaboration likelihood model, which is described in the Social Psychology chapter of the written study materials.

a. Incorrect See explanation for response b.
b. CORRECT According to this model, whether a person uses the central route (carefully considers an argument) or peripheral route (relies on heuristic criteria) depends on several factors including the nature of the message and the person’s mood. With regard to the latter, a person is more likely to rely on the peripheral route when he/she is in a good mood.
c. Incorrect See explanation for response b.
d. Incorrect See explanation for response b.

The correct answer is: in a good mood.

196
Q

The “sleeper effect” (Hovland & Weiss, 1952) is useful for understanding:
Select one:

A. why people might reject a persuasive message immediately after it is delivered by a low credible communicator but accept the message six weeks later.
B. why people are more likely to accept a persuasive message that they accidentally overheard than a message that was intentionally delivered to them.
C. why a decision-maker chooses the first solution that meets minimum criteria of acceptability rather than obtaining the information needed to choose the optimal solution.
D. why group decisions are often more extreme (riskier or more conservative) than the decision each group member would make alone.

A

The sleeper effect refers to the delayed impact of a persuasive message when the message is accompanied by a discounting cue (e.g., is delivered by a low credible communicator).

a. CORRECT C. I. Hovland and W. Weiss found that, over time, the persuasive impact of a message delivered by a highly credible communicator diminished while the persuasive impact of the same message delivered by a low credible communicator increased. They referred to the latter phenomenon as the “sleeper effect.”
b. Incorrect Although accidental (versus intentional) messages are often more readily accepted, this phenomenon is not referred to as the sleeper effect.
c. Incorrect This answer refers to the bounded rationality model of decision making which states that decision-makers often “satisfice” rather than “optimize” due to limited time and resources.
d. Incorrect This answer describes group polarization.

The correct answer is: why people might reject a persuasive message immediately after it is delivered by a low credible communicator but accept the message six weeks later.

197
Q

Byrne’s law of attraction emphasizes the role of _________ as an essential determinant of our attraction to others.
Select one:

A. physical attractiveness
B. reciprocal liking
C. propinquity
D. attitude similarity

A

All of the factors listed in the answers to this question have been linked to interpersonal attraction, but Byrne’s law of attraction focuses on one of them.

a. Incorrect See explanation for response d.
b. Incorrect See explanation for response d.
c. Incorrect See explanation for response d.
d. CORRECT According to Byrne, people who have similar attitudes to us reinforce our perceptions of the world and, consequently, are associated with positive feelings that increase our attraction to those people.

The correct answer is: attitude similarity

198
Q

__________ theory predicts that we tend to like others whose initially negative feelings toward us change to positive feelings more than we like those who have positive feelings toward us from the beginning.
Select one:

A. Self-verification
B. Gain-loss
C. Social exchange
D. Balance

A

For the exam, you want to be familiar with all of the theories listed in the answers to this question. Additional information on these theories is provided in the Social Psychology chapter of the written study materials.

a. Incorrect Self-verification theory predicts that people prefer others who provide them with accurate information about themselves (i.e., with information that is consistent with their self-evaluations).
b. CORRECT Gain-loss theory predicts that we tend to like others whose initial negative feelings toward us change to positive feeling more than those who are consistently positive and, conversely, that we tend to dislike others whose initially positive feelings toward us change to negative feelings more than those who have negative feelings toward us from the beginning.
c. Incorrect Social exchange theory focuses on the rewards and costs of a relationship and predicts that we are more likely to remain in a relationship when the rewards exceed the costs of doing so.
d. Incorrect Balance theory uses the concept of cognitive consistency to explain attitude change and focuses on the relationships between three entities - the person, another person, and a third person or an idea, event, or object.

The correct answer is: Gain-loss

199
Q

Advocates for prayer in school hope to persuade other members of the community to support their position. According to social judgment theory, other community members are most likely to be convinced if the discrepancy between their initial position on this issue and the position of the advocates of school prayer:
Select one:

A. is low.
B. is moderate.
C. is high.
D. varies from member to member.

A

Social judgment theory considers initial level of discrepancy to be an important determinant of attitude change.

a. Incorrect See explanation for response b.
b. CORRECT According to social judgment theory, a person’s potential for attitude change can be described in terms of three dimensions or latitudes: latitude of acceptance, latitude of non-commitment, and latitude of rejection. When an attitude is within the person’s latitude of acceptance, that person already agrees with the position, so no attitude change occurs; when it is within the person’s latitude of rejection, the person is unlikely to change his/her attitude. Therefore, the greatest attitude change occurs when the attitude is within the person’s latitude of non-commitment, which occurs when there is a moderate discrepancy between the person’s initial position and the position of the communication.
c. Incorrect See explanation for response b.
d. Incorrect See explanation for response b.

The correct answer is: is moderate.

200
Q

The tendency to overestimate personality traits as the cause of behavior and underestimate the influence of situational factors is referred to as:
Select one:

A. the halo bias.
B. the discounting principle.
C. the self-serving bias.
D. the fundamental attribution bias.

A

This question is asking about the tendency to make dispositional (versus situational) attributions.

a. Incorrect See explanation for response d.
b. Incorrect See explanation for response d.
c. Incorrect See explanation for response d.
d. CORRECT This tendency is referred to as the fundamental attribution bias. In most of the literature, it is described as occurring when an observer makes an attribution about the behavior of another person.

The correct answer is: the fundamental attribution bias.

201
Q

Sherif (1935) used the autokinetic effect to study which of the following phenomena?
Select one:

A. conformity
B. affiliation
C. psychological reactance
D. locus of control

A

The apparent movement of a stationary light in a dark room is referred to as the autokinetic effect.

a. CORRECT Sherif (1935) used the autokinetic effect to study conformity. He found that estimates of movement varied widely when a participant made an estimate alone but conformed to the estimates of other people when a participant was asked to provide an estimate after hearing the estimates of others.
b. Incorrect See explanation for response a.
c. Incorrect See explanation for response a.
d. Incorrect See explanation for response a.

The correct answer is: conformity

202
Q

In team sports, the “home advantage” phenomenon has most consistently been linked to:
Select one:

A. the demoralization of the opposing team.
B. the impact of a supportive home audience.
C. the players expectations.
D. the players familiarity with the home court.

A

If you’re a sports fan, you probably know that the home advantage refers to the tendency of sports teams to win more often when they are playing on their home court.

a. Incorrect See explanation for response b.
b. CORRECT Several explanations for the home advantage have been proposed. However, the most consistently supported explanation is that it is due to the social support that fans give the home team. See, e.g., K. S. Courneya and A. V. Carron, The home advantage in sports competitions: A literature review, Journal of Sport & Exercise Psychology, 14, 13-27, 1992.
c. Incorrect See explanation for response b.
d. Incorrect See explanation for response b.

The correct answer is: the impact of a supportive home audience.

203
Q

The unavailability of preventive health services in minority communities, a lack of quality health care in close proximity to these communities, and the delivery of substandard health care services to members of these communities is best attributed to which of the following?
Select one:

A. subtle racism
B. institutional racism
C. personally mediated racism
D. internalized racism

A

Several types of racism have been distinguished. For example, C. P. Jones identifies three levels of racism - institutional, personally mediated, and internalized [Levels of racism: A theoretic framework and a gardener’s tale, American Journal of Public Health, 90(8), 1212-1216, 2000].

a. Incorrect Subtle racism refers to a less blatant (more covert) form of racism, which some experts contend has replaced “old-fashioned” (overt) prejudice and discrimination. The term subtle racism is used most often to describe the beliefs, attitudes, and actions of individuals (rather than institutions).
b. CORRECT Institutional racism refers to denial or restriction of material conditions (e.g., access to health care) and access to power to members of minority groups.
c. Incorrect Personally mediated racism refers to prejudice and discrimination at the individual level.
d. Incorrect As defined by Jones, internalized racism refers to “acceptance by members of the stigmatized races of negative messages about their own abilities and intrinsic worth” (p. 1213).

The correct answer is: institutional racism

204
Q

When conducting a one-way ANOVA, the F-ratio is calculated by:
Select one:

A. dividing the mean square between (MSB) by the mean square within (MSW).
B. dividing the mean square within (MSW) by the mean square between (MSB).
C. dividing the mean square total (MST) by the mean square between (MSB).
D. dividing the mean square total (MST) by the mean square within (MSW).

A

When using the one-way ANOVA to determine if an independent variable has had a significant effect on a dependent variable, an F-ratio is calculated by dividing the mean square between (MSB) by the mean square within (MSW).

a. CORRECT MSB provides an estimate of variability due to treatment plus error, while MSW provides an estimate of variability due to error only. When the independent variable has had an effect, MSB will be larger than MSW and the F-ratio will be larger than 1.0, and the larger the F-ratio, the more likely that the effect of the independent variable is statistically significant.
b. Incorrect See explanation for answer a.
c. Incorrect See explanation for answer a.
d. Incorrect See explanation for answer a.

The correct answer is: dividing the mean square between (MSB) by the mean square within (MSW).

205
Q

When using the analysis of variance, calculation of the F-ratio involves:
Select one:

A. comparing group means directly to each other.
B. comparing each group mean to a total mean.
C. comparing true (experimental) variability to error variability.
D. comparing between-group variability to within-group variability.

A

The F-ratio is a measure of treatment effects plus measurement error divided by a measure of measurement error only.

a. Incorrect Unlike the t-test, the ANOVA compares means indirectly rather than directly.
b. Incorrect Some of the calculations for the ANOVA do involve comparing group means to a “grand mean,” but this is not what is done in deriving the F-ratio itself.
c. Incorrect This doesn’t describe the F-ratio.
d. CORRECT This is the best description of the F-ratio since the numerator is a measure of between-group variability, while the denominator is a measure of within-group variability.

The correct answer is: comparing between-group variability to within-group variability.

206
Q

In a normal distribution of test scores, approximately ____% of scores are more than two standard deviations above the mean.
Select one:

A. 2.5
B. 5
C. 7.5
D. 10

A

Knowing that, in a normal distribution, approximately 95% of scores fall between the scores that are -2 and +2 standard deviations from the mean would have helped you identify the correct answer to this question.

a. CORRECT About 95% of scores fall between the scores that are -2 and +2 standard deviations from the mean. This means that about 2.5% of scores are less than the score that is 2 standard deviations below the mean and about 2.5% are more than the score that is 2 standard deviations above the mean.
b. Incorrect See explanation for response a.
c. Incorrect See explanation for response a.
d. Incorrect See explanation for response a.

The correct answer is: 2.5

207
Q

A factorial ANOVA is used to statistically analyze data when:
Select one:

A. the study includes two or more dependent variables.
B. there are more than two levels of a single independent variable.
C. the study includes two or more independent variables.
D. the effects of a confounding variable must be statistically removed.

A

A factorial ANOVA is used to analyze data when a factorial design is used and the dependent variable is measured on an interval or ratio scale.

a. Incorrect A MANOVA (multivariate analysis of variance) would be the appropriate test in this situation.
b. Incorrect A one-way ANOVA would be the appropriate test in this situation.
c. CORRECT A research design is “factorial” when it includes two or more independent variables. The factorial ANOVA analyzes the main effects of each independent variable as well as their interaction(s).
d. Incorrect An ANCOVA (analysis of covariance) would be used in this situation.

The correct answer is: the study includes two or more independent variables.

208
Q

A multiple regression equation yields a predicted criterion score for an examinee based on the examinee’s scores on the predictors included in a test battery. When computing a multiple regression equation, each test is weighted:
Select one:

A. in direct proportion to its correlation with the criterion and in inverse proportion to its correlation with the other predictors in the test battery.
B. in inverse proportion to its correlation with the criterion and in direct proportion to its correlation with the other predictors in the test battery.
C. in direct proportion to its correlation with the criterion and with the other predictors in the test battery.
D. in inverse proportion to its correlation with the criterion and with the other predictors in the test battery.

A

When choosing tests for a test battery that will be used to predict status on a criterion, each test should correlate highly with the criterion but not with the other tests in the battery in order to provide the most useful (nonredundant) information.

a. CORRECT By computing the multiple regression equation so that each test is weighted in direct proportion to its correlation with the criterion and in inverse proportion to its correlation with other tests, the test with the highest criterion-related validity and the least amount of overlap (correlation) with the other tests will be given the largest weight, while the test with the lowest criterion-related validity and the most overlap with the other tests will be given the smallest weight.
b. Incorrect See explanation for response a.
c. Incorrect See explanation for response a.
d. Incorrect See explanation for response a.

The correct answer is: in direct proportion to its correlation with the criterion and in inverse proportion to its correlation with the other predictors in the test battery.

209
Q

Which of the following would be most useful for combining the results of several research studies that each evaluated the effectiveness of the same treatment for a particular disorder?
Select one:

A. p-value
B. kappa coefficient
C. coefficient alpha
D. effect size

A

To combine data from different studies (which may have different sample sizes, different outcome measures, etc.), the data must be converted to a common metric.

a. Incorrect Although the p-value indicates the level of significance (probability that the null hypothesis is false), it is not as useful as an effect size for combining the results of different studies.
b. Incorrect The kappa coefficient is used to evaluate inter-rater reliability.
c. Incorrect Coefficient alpha is used to assess internal consistency reliability.
d. CORRECT An effect size is the index that is calculated when conducting a meta-analysis. It converts data from different studies to a common metric so that the results of the studies can be combined and compared.

The correct answer is: effect size

210
Q

A distribution of scores is ____________ when it is more “peaked” than a normal distribution.
Select one:

A. platykurtic
B. leptokurtic
C. mesokurtic
D. endokurtic

A

For the exam, you want to know the difference between platykurtic and leptokurtic.

a. Incorrect A platykurtic distribution is flatter than a normal distribution.
b. CORRECT A leptokurtic distribution is more “peaked” than a normal distribution – i.e., a larger proportion of scores are in the middle of the distribution.
c. Incorrect A normal distribution is mesokurtic.
d. Incorrect Endokurtic is a “made-up” term.

The correct answer is: leptokurtic

211
Q

When conducting a one-way ANOVA to compare the effects of four different diets on weight loss, an F-ratio of _____ suggests that there may be a significant difference between the types of diets?
Select one:

A. 15.5
B. 0.9
C. 0
D. -1.5

A

The F-ratio is calculated by dividing the mean square between by the mean square within. Mean square between is a measure of treatment effects plus error, while mean square within in a measure of error only.

a. CORRECT A treatment effect is suggested when the numerator of the F ratio (mean square between) is larger than the denominator (mean square within) - i.e., when the F value is greater than +1.0.
b. Incorrect See explanation for response a.
c. Incorrect See explanation for response a.
d. Incorrect See explanation for response a.

The correct answer is: 15.5

212
Q

Adding a constant to every score in a distribution of scores will:
Select one:

A. increase the distribution’s mean.
B. increase the distribution’s standard deviation.
C. increase the distribution’s mean and standard deviation.
D. not increase the distribution’s mean or standard deviation.

A

As noted in the Statistics and Research Design chapter of the written study materials, adding or subtracting a constant to each score changes the mean and multiplying or dividing by a constant changes both the mean and the standard deviation.

a. CORRECT Adding a constant only affects the mean and other measures of central tendency. It does not affect the variability (spread) of scores in the distribution.
b. Incorrect See explanation for response a.
c. Incorrect See explanation for response a.
d. Incorrect See explanation for response a.

The correct answer is: increase the distribution’s mean.

213
Q

The ANCOVA enables an experimenter to:
Select one:

A. statistically remove error variance that is attributable to a known extraneous variable.
B. determine if the effects of an extraneous variable moderate the relationship between the IV and DV.
C. simultaneously assess more than one hypothesis about the IV’s effects on the DV.
D. test the effects of an IV on more than one DV without increasing the experimentwise error rate.

A

Answer A is correct: The ANCOVA (analysis of covariance) is one of several forms of the analysis of variance that are described in the Statistics and Research Design chapter of the written study materials and that you want to be familiar with for the licensing exam. The ANCOVA is used to statistically remove the effects of an extraneous variable (i.e., error variance) so that it is easier to detect the effects of the IV on the DV.

Answer B: This doesn’t describe the ANCOVA as well as response a.

Answer C: This isn’t why an ANCOVA is used.

Answer D: This describes the MANOVA (multivariate analysis of variance).

The correct answer is: statistically remove error variance that is attributable to a known extraneous variable.

214
Q

A developmental psychologist finds that low birthweight is associated with a higher risk for ADHD for boys but not for girls. In this situation, gender is a(n):
Select one:

A. mediator variable.
B. moderator variable.
C. feedback variable.
D. outcome variable.

A

In the situation described in the question, gender is moderating the relationship between birthweight and risk for ADHD.

a. Incorrect A mediator variable is responsible for an observed relationship between an independent variable and a dependent (outcome) variable. In other words, the independent variable affects the mediator variable which, in turn, affects the dependent variable.
b. CORRECT In this situation, gender is a moderator variable because it affects (moderates) the strength of the relationship between the independent and dependent variables - i.e., between birthweight and risk for ADHD.
c. Incorrect “Feedback variable” is a term used in engineering and is not relevant to the situation described in this question.
d. Incorrect “Outcome variable” is another name for dependent variable and criterion.

The correct answer is: moderator variable.

215
Q

Structural equation modeling (SEM) is used to:
Select one:

A. assess the length of time to the occurrence of a critical event.
B. evaluate the causal (predictive) influences of multiple latent factors.
C. determine, for a sample of people, how many types the sample represents.
D. identify natural subgroupings among a collection of observations.

A

Structural equation modeling is a multivariate technique that is used to test causal hypotheses about the relationships among a set of factors.

a. Incorrect Survival analysis is used to evaluate the length of time to a critical event (e.g., relapse, promotion).
b. CORRECT Of the answers given, this one best describes the purpose of structural equation modeling: It allows a researcher to identify the underlying (latent) factors that relate to a set of measured variables and the nature of the causal relationships between those factors. Additional information on SEM is provided in the Statistics and Research Design chapter of the written study materials.
c. Incorrect Q-technique factor analysis is used to identify types of people in a sample of people.
d. Incorrect Cluster analysis is used to identify homogeneous subgroups in a heterogeneous collection of observations.

The correct answer is: evaluate the causal (predictive) influences of multiple latent factors.

216
Q

A normal distribution of raw scores has a mean of 106 and a standard deviation of 10. In this distribution, which of the following scores is equivalent to a raw score of 126?
Select one:

A. a percentile rank of 98
B. a T-score of 55
C. a z-score of +2.5
D. a stanine score of 6

A

In the distribution described in this question, a raw score of 126 is two standard deviations above the mean.

a. CORRECT In a normal distribution, a percentile rank of 98 is about two standard deviations above the mean.
b. Incorrect A T-score of 55 is one-half standard deviation above the mean.
c. Incorrect Z-scores are interpreted directly in terms of standard deviation units, which means that a z-score of +2.5 is 2-1/2 standard deviations above the mean.
d. Incorrect A stanine score of 6 is between the mean and the score that is one standard deviation above the mean.

The correct answer is: a percentile rank of 98

217
Q

In a factor matrix, the factor loading for Test A and Factor II is .70. This means that:
Select one:

A. 70% of variability in Test A is accounted for by Factor II.
B. 49% of variability in Test A is accounted for by Factor II.
C. only 70% of variability in Test A is accounted for by the factor analysis.
D. only 49% of variability in Test A is accounted for by the factor analysis.

A

A factor loading is the correlation coefficient for a test and an identified factor.

a. Incorrect See explanation for response b.
b. CORRECT A factor loading can be interpreted by squaring it to obtain a measure of shared variability. When the factor loading is .70, this means that 49% (.70 squared) of variability in the test is accounted for by the factor.
c. Incorrect This is not an appropriate interpretation.
d. Incorrect Some variability in Test A is probably also accounted for by the other identified factor(s), so this is not the best answer.

The correct answer is: 49% of variability in Test A is accounted for by Factor II.

218
Q

To construct the 68% confidence interval for an examinee’s obtained test score, you would need the examinee’s score and:
Select one:

A. the test’s mean.
B. the range of test scores.
C. the standard error of measurement.
D. the standard error of estimate.

A

To construct a confidence interval around an obtained test score, you need the standard error of measurement (which is calculated from the test’s standard deviation and reliability coefficient).

a. Incorrect The mean is not needed to construct a confidence interval.
b. Incorrect The standard deviation is one of the elements needed to calculate the standard error of measurement but, by itself, cannot be used to construct a confidence interval.
c. CORRECT To construct a 68% confidence interval, you add and subtract one standard error of measurement to and from the examinee’s obtained test score.
d. Incorrect The standard error of estimate is used to construct a confidence interval around a predicted criterion score.

The correct answer is: the standard error of measurement.

219
Q

A screening test for a disorder that has a very low base rate in the population is known to have an overall accuracy rate of 98%. When using this test to identify individuals in the general population who have the disorder, it’s important to keep in mind that the test will produce:
Select one:

A. very few false positives but very many true positives.
B. very few true negatives but very many false negatives.
C. a larger number of false negatives than false positives.
D. a larger number of false positives than false negatives.

A

This is a difficult question, but you may have been able to identify the correct answer if you know that a low base rate means that there are very few people in the population who have the disorder, which implies that the most likely predictive error will be to falsely identify those who do not have the disorder as having it.

a. Incorrect See explanations for answers c and d.
b. Incorrect See explanations for answers c and d.
c. Incorrect To understand why this answer is incorrect (and answer d is correct), assume that the base rate for the disorder is 1% and that you test a random sample of 10,000 people with the screening test. In this situation, 100 people will have the disease, and the test (which has a 98% accuracy rate) will correctly identify 98 of them - i.e., 98 will be true positives and 2 will be false negatives.
d. CORRECT Continuing with the example, when the base rate for the condition is 1%, 9,900 of the 10,000 people tested will not have the disease, and the test (which has a 98% accuracy rate) will correctly identify 9,702 of them - i.e., 9,702 will be true negatives and the remaining 198 will be false positives. In other words, there will be more false positives than false negatives - and this will be true whenever the predictor has a high accuracy rate and the base rate is less than 50%.

The correct answer is: a larger number of false positives than false negatives.

220
Q

An item discrimination (D) index of 0 for an item included in an academic achievement test indicates that:
Select one:

A. the item was answered correctly by only a few students.
B. the item was answered correctly by more low-achieving students than by high-achieving students.
C. the item was answered correctly by more high-achieving students than by low-achieving students.
D. the item was answered correctly by the same number of low- and high-achieving students.

A

The item discrimination index indicates the extent to which a test item discriminates between examinees who obtain high versus low scores on the entire test or on an external criterion and ranges in value from +1.0 to -1.0.

a. Incorrect See explanation for response d.
b. Incorrect A negative item discrimination index indicates that the item was answered correctly by more low-achieving students than by high-achieving students.
c. Incorrect A positive item discrimination index indicates that the item was answered correctly by more high-achieving students than by low-achieving students.
d. CORRECT The item discrimination index (D) is calculated by subtracting the percent of examinees in the low-scoring group who answered the item correctly from the percent of examinees in the high-scoring group who answered the item correctly. D is equal to 0 only when these percents are the same.

The correct answer is: the item was answered correctly by the same number of low- and high-achieving students.

221
Q

When item response theory has been used as the basis for test construction, an examinee’s score on the test provides information about his/her:
Select one:

A. future status on an external criterion.
B. status on a latent trait or ability.
C. performance relative to other examinees.
D. performance relative to performance on related measures.

A

Item response theory differs from classical test theory in several ways, including the interpretation of an examinee’s scores.

a. Incorrect See explanation for response a.
b. CORRECT One of the primary characteristics of item response theory is that it is based on the assumption that “the performance of an examinee on a test item can be explained (or predicted) by a set of factors called ‘traits,’ ‘latent traits,’ or ‘abilities’” [D. H. Henard, Item response theory, in L. G. Grimm and P. R. Yarnold (eds.), Reading and understanding more multivariate statistics, Washington, DC, APA, 2000].
c. Incorrect See explanation for response a.
d. Incorrect See explanation for response a.

The correct answer is: status on a latent trait or ability.

222
Q

To evaluate the impact of an educational program on each student’s mastery of the information presented, which of the following would be most useful?
Select one:

A. norm-referenced scores
B. criterion-referenced scores
C. standard scores
D. ipsative scores

A

To determine if a student has benefited from an educational program, you would most likely want to determine how much of the information presented in the program has been retained and/or to what degree participation in the program has improved the individual’s performance on a task.

a. Incorrect Norm-referenced scores (e.g., standard scores, percentile ranks) tell you how well an examinee is doing compared to other examinees.
b. CORRECT Criterion-referenced scores tell you how well an examinee did in absolute terms (e.g., how many questions he/she answered correctly) and, therefore, would be most useful for the purpose described in the question.
c. Incorrect Standard scores are a type of norm referenced score.
d. Incorrect Ipsative scores indicate the relative strengths of the different characteristics measured by a test for the individual and would be less useful than criterion-referenced scores for the purpose described in the question.

The correct answer is: criterion-referenced scores

223
Q

All other things being equal, which of the following types of tests would be expected to have the lowest reliability?
Select one:

A. 7-response multiple choice
B. 3-response multiple choice
C. true-false
D. free recall

A

A test’s reliability is an index of the degree to which scores on the test are free from random error (chance factors) and indicative of examinees’ true scores. The greater the probability that an examinee can answer an item correctly by guessing (i.e., by chance), the lower a test’s reliability.

a. Incorrect The probability that an examinee can guess a correct answer on a 7-response multiple choice test is 14% (1 in 7).
b. Incorrect The probability that an examinee can guess a correct answer on a 3-response multiple choice exam is 33% (1 in 3).
c. CORRECT On a true-false test, the probability that the correct answer can be selected by chance is 50% (1/2). In other words, there is a higher probability that an examinee can guess a correct answer on a true-false test than on the other tests, and thus, an examinee’s score on a true-false test will reflect, to a greater degree, error rather than his/her true score.
d. Incorrect The probability that an examinee can guess a correct answer on a free recall test is close to zero.

The correct answer is: true-false

224
Q

Assuming that the following scores are all from the same normal distribution of scores, which of the following lists the scores in order from lowest to highest?
Select one:

A. z-score of +1.0, percentile rank of 70, T-score of 80
B. z-score of +.75, percentile rank of 84, T-score of 65
C. z-score of +1.25, percentile rank of 95, T-score of 55
D. z-score of +.50, percentile rank of 98, T-score of 60

A

For the exam, you want to be familiar with the relationship between z-scores, percentile ranks, and T-scores. For a review of this information, see the section on “Test Score Interpretation” in the Test Construction chapter of the AATBS written study materials.

a. Incorrect Converting the scores to standard deviation units would have helped you identify the correct answer to this question. A z-score of +1.0 is one standard deviation above the mean, a percentile rank of 70 is below one standard deviation above the mean, and a T-score of 80 is three standard deviations above the mean.
b. CORRECT A z-score of +.75 is 3/4ths of a standard deviation above the mean; a percentile rank of 84 is one standard deviation above the mean; and a T-score of 65 is 1-1/2 standard deviations above the mean. Therefore, this answer lists the scores in order from lowest to highest.
c. Incorrect A z-score of +1.25 is 1-1/4 standard deviations above the mean, a percentile rank of 95 is more than one standard deviation above the mean, and a T score of 55 is one-half standard deviation above the mean.
d. Incorrect A z-score of +.50 is one-half standard deviation above the mean, a percentile rank of 98 is two standard deviations above the mean, and a T-score of 60 is one standard deviation above the mean.

The correct answer is: z-score of +.75, percentile rank of 84, T-score of 65

225
Q

When using the multitrait-multimethod matrix to evaluate the construct validity of a newly developed test, a __________ coefficient provides evidence of the test’s divergent (discriminant) validity.
Select one:

A. large monotrait-heteromethod
B. large heterotrait-monomethod
C. small monotrait-heteromethod
D. small heterotrait-monomethod

A

The answers to this question list two of the four types of correlation coefficients included in a multitrait-multimethod matrix.

a. Incorrect The monotrait-heteromethod coefficient indicates the correlation of two different methods that measure the same trait. A large monotrait-heteromethod coefficient provides evidence of a test’s convergent validity. For example, if the test being validated is a self-rating scale of self-esteem and it has a high correlation with a previously validated parent rating scale of self-esteem, this would suggest that the test being validated is actually measuring self-esteem.
b. Incorrect The heterotrait-monomethod coefficient indicates the correlation between two similar methods that measure different traits. A large heterotrait-monomethod coefficient indicates that a test has a lack of divergent validity. For example, if the test being validated is a self-rating scale of self-esteem and it has a high correlation with a self-rating scale of neuroticism, this would suggest that the measure of self-esteem may be measuring something other than self-esteem since these two traits should not correlate.
c. Incorrect A small monotrait-heteromethod coefficient would suggest that the test lacks convergent validity.
d. CORRECT A small heterotrait-monomethod coefficient provides evidence of a test’s divergent validity - i.e., it shows that test does not correlate with a measure of another trait that it should not correlate with.

The correct answer is: small heterotrait-monomethod